Home
Browse
Create
Search
Log in
Sign up
Upgrade to remove ads
Only $2.99/month
Final/HESI
Mometrix
Get Quizlet's official HESI A2 - 1 term, 1 practice question, 1 full practice test
Preview
STUDY
Flashcards
Learn
Write
Spell
Test
PLAY
Match
Gravity
Terms in this set (1757)
1. A nurse reviews the urinalysis of a client and notes the presence of glucose. Which action should the nurse take?
a. Document findings and continue to monitor the client.
b. Contact the provider and recommend a 24-hour urine test.
c. Review the client's recent dietary selections. d. Perform a capillary artery glucose assessment.
ANS: D
Glucose normally is not found in the urine. The normal renal threshold for glucose is about 220 mg/dL, which means that a person whose blood glucose is less than 220 mg/dL will not have glucose in the urine. A positive finding for glucose on urinalysis indicates high blood sugar. The most appropriate action would be to perform a capillary artery glucose assessment. The client needs further evaluation for this abnormal result; therefore, documenting and continuing to monitor is not appropriate. Requesting a 24-hour urine test or reviewing the client's dietary selections will not assist the nurse to make a clinical decision related to this abnormality.
A nurse caring for a client with sickle cell disease (SCD) reviews the client's laboratory work. Which finding should the nurse report to the provider?
a. Creatinine: 2.9 mg/dL
b. Hematocrit: 30%
c. Sodium: 147 mEq/L
d. White blood cell count: 12,000/mm3
a. Creatinine: 2.9 mg/dL
1. A client is receiving an infusion of tissue plasminogen activator (t-PA). The nurse assesses the client to be disoriented to person, place, and time. What action by the nurse is best?
a. Assess the client's pupillary responses.
b. Request a neurologic consultation.
c. Stop the infusion and call the provider.
d. Take and document a full set of vital signs.
ANS: C
A change in neurologic status in a client receiving t-PA could indicate intracranial hemorrhage. The nurse should stop the infusion and notify the provider immediately. A full assessment, including pupillary responses and vital signs, occurs next. The nurse may or may not need to call a neurologist.
DIF: Applying/Application REF: 768
KEY: Coronary artery disease| neurologic system| critical rescue| Rapid Response Team| thrombolytic agents MSC: Integrated Process: Nursing Process: Implementation
NOT: Client Needs Category: Physiological Integrity: Pharmacological and Parenteral Therapies
The nurse manager for the medical-surgical unit is making staff assignments. Which client will be most appropriate to assign to a newly graduated RN who has completed a 6-week unit orientation?
A. Client with chronic hypothyroidism and dementia who takes levothyroxine (Synthroid) daily
B. Client with follicular thyroid cancer who has vocal hoarseness and difficulty swallowing
C. Client with Graves' disease who is experiencing increasing anxiety and diaphoresis
D. Client with hyperparathyroidism who has just arrived on the unit after a parathyroidectomy
A.
The client with chronic hypothyroidism and dementia is the most stable of the clients described and would be most appropriate to assign to an inexperienced RN. A client with vocal hoarseness and difficulty swallowing is at higher risk for complications and requires close observation by a more experienced nurse. Increasing anxiety and diaphoresis in a client with Graves' disease can be an indication of impending thyroid storm, which is an emergency; this is not a situation to be managed by a newly graduated RN. A client who has just arrived on the unit after a parathyroidectomy requires close observation for bleeding and airway compromise and requires assessment by an experienced nurse.
1. A nurse is assessing a client who has suffered a nasal fracture. Which assessment should the nurse perform first?
a. Facial pain
b. Vital signs
c. Bone displacement
d. Airway patency
ANS: D
A patent airway is the priority. The nurse first should make sure that the airway is patent and then should determine whether the client is in pain and whether bone displacement or blood loss has occurred.
DIF: Applying/Application REF: 531
KEY: Trauma| medical emergencies
MSC: Integrated Process: Nursing Process: Implementation
NOT: Client Needs Category: Safe and Effective Care Environment: Management of Care
1. A nurse assesses a client with polycystic kidney disease (PKD). Which assessment finding should alert the nurse to immediately contact the health care provider?
a. Flank pain
b. Periorbital edema
c. Bloody and cloudy urine d. Enlarged abdomen
ANS: B
Periorbital edema would not be a finding related to PKD and should be investigated further. Flank pain and a distended or enlarged abdomen occur in PKD because the kidneys enlarge and displace other organs. Urine can be bloody or cloudy as a result of cyst rupture or infection.
1. A nurse working in a geriatric clinic sees clients with "cold" symptoms and rhinitis. Which drug would be appropriate to teach these clients to take for their symptoms?
a. Chlorpheniramine (Chlor-Trimeton)
b. Diphenhydramine (Benadryl)
c. Fexofenadine (Allegra) d. Hydroxyzine (Vistaril)
ANS: C
First-generation antihistamines are not appropriate for use in the older population. These drugs include chlorpheniramine, diphenhydramine, and hydroxyzine. Fexofenadine is a second-generation antihistamine.
A nursing student wants to know why clients with chronic obstructive pulmonary disease tend to be polycythemic. What response by the nurse instructor is best?
a.
It is due to side effects of medications for bronchodilation.
b.
It is from overactive bone marrow in response to chronic disease.
c.
It combats the anemia caused by an increased metabolic rate.
d.
It compensates for tissue hypoxia caused by lung disease.
d.
It compensates for tissue hypoxia caused by lung disease.
ANS: D
In response to hypoxia, more red blood cells are made so more oxygen can be carried and delivered to tissues. This is a physiologic process in response to the disease; it is not a medication side effect, the result of overactive bone marrow, or a response to anemia.
1. A nurse assesses clients on a cardiac unit. Which client should the nurse identify as being at greatest risk for the development of left-sided heart failure?
a. A 36-year-old woman with aortic stenosis
b. A 42-year-old man with pulmonary hypertension
c. A 59-year-old woman who smokes cigarettes daily
d. A 70-year-old man who had a cerebral vascular accident
ANS: A
Although most people with heart failure will have failure that progresses from left to right, it is possible to have left-sided failure alone for a short period. It is also possible to have heart failure that progresses from right to left. Causes of left ventricular failure include mitral or aortic valve disease, coronary artery disease, and hypertension. Pulmonary hypertension and chronic cigarette smoking are risk factors for right ventricular failure. A cerebral vascular accident does not increase the risk of heart failure.
DIF: Applying/Application REF: 679
KEY: Heart failure| health screening
MSC: IntegratedProcess:NursingProcess:Assessment
NOT: Client Needs Category: Safe and Effective Care Environment: Management of Care
A nurse cares for a client who is prescribed a drug that blocks a hormone's receptor site. Which therapeutic effect should the nurse expect?
a. Greater hormone metabolism
b. Decreased hormone activity
c. Increased hormone activity
d. Unchanged hormone response
ANS: B
Hormones cause activity in the target tissues by binding with their specific cellular receptor sites, thereby changing the activity of the cell. When receptor sites are occupied by other substances that block hormone binding, the cell's response is the same as when the level of the hormone is decreased.
The charge nurse is making client assignments for the medical-surgical unit. Which client will be best to assign to an RN who has floated from the pediatric unit?
A. Client in Addisonian crisis who is receiving IV hydrocortisone
B. Client admitted with syndrome of inappropriate antidiuretic hormone (SIADH) secondary to lung cancer
C. Client being discharged after a unilateral adrenalectomy to remove an adrenal tumor
D. Client with Cushing's syndrome who has elevated blood glucose and requires frequent administration of insulin
D.
An RN who works with pediatric clients would be familiar with glucose monitoring and insulin administration. A client in Addisonian crisis would best be monitored by an RN from the medical-surgical floor. Although the float RN could complete the admission history, the client with SIADH secondary to lung cancer might require teaching and orientation to the unit that a nurse more familiar with that area would be better able to provide. Discharge teaching specific to adrenalectomy should be provided by the RN who is regularly assigned to the medical-surgical floor and is more familiar with taking care of postoperative adult clients with endocrine disorders.
1. A nurse assesses a client who had a myocardial infarction and is hypotensive. Which additional assessment finding should the nurse expect?
a. Heart rate of 120 beats/min
b. Cool, clammy skin
c. Oxygen saturation of 90%
d. Respiratory rate of 8 breaths/min
A
When a client experiences hypotension, baroreceptors in the aortic arch sense a pressure decrease in the vessels. The parasympathetic system responds by lessening the inhibitory effect on the sinoatrial node. This results in an increase in heart rate and respiratory rate. This tachycardia is an early response and is seen even when blood pressure is not critically low. An increased heart rate and respiratory rate will compensate for the low blood pressure and maintain oxygen saturations and perfusion. The client may not be able to compensate for long, and decreased oxygenation and cool, clammy skin will occur later.
p. 801, Safe and Effective Care Environment
Which new assessment finding in a client with sickle cell disease who currently is in crises does the nurse report immediately to the health care provider?
A. Pain in the right hip with limited range of motion
B. Slow capillary refill in the toes of the right foot
C. Yellow appearance of the roof of the mouth
D. Facial drooping on the right side
Answer: D
Rationale: All current assessment findings are important. However, the pain in the hip, the slow capillary refill, and the yellow appearance of the roof of the mouth are related to the crises and are expected. The facial drooping as a new finding indicates the possibility of reduced brain perfusion and stroke. This new development requires immediate attention and intervention.
The charge nurse at an assisted-living facility receives report from an emergency department (ED) nurse about one of the resident clients. The client was sent to the ED with a fever, chills, muscle aches, and headache. The ED nurse reports the client's rapid influenza report came back from the laboratory positive for influenza A. What action by the nurse at the assisted-living facility is most appropriate?
A) Prepare to administer antibiotics.
B) Have the resident eat meals in his room.
C) Provide oseltamivir (Tamiflu) to the staff.
D) Arrange a follow-up chest x-ray in 2 weeks.
(Chp. 31; p. 587)
B) Have the resident eat meals in his room.
(Chp. 31; p. 587)
1. A nurse assesses clients on the medical-surgical unit. Which client is at greatest risk for the development of bacterial cystitis?
a. A 36-year-old female who has never been pregnant
b. A 42-year-old male who is prescribed cyclophosphamide
c. A 58-year-old female who is not taking estrogen replacement d. A 77-year-old male with mild congestive heart failure
ANS: C
Females at any age are more susceptible to cystitis than men because of the shorter urethra in women. Postmenopausal women who are not on hormone replacement therapy are at increased risk for bacterial cystitis because of changes in the cells of the urethra and vagina. The middle-aged woman who has never been pregnant would not have a risk potential as high as the older woman who is not using hormone replacement therapy.
1. A nurse assesses clients on the medical-surgical unit. Which client is at greatest risk for the development of bacterial cystitis?
a. A 36-year-old female who has never been pregnant
b. A 42-year-old male who is prescribed cyclophosphamide
c. A 58-year-old female who is not taking estrogen replacement
d. A 77-year-old male with mild congestive heart failure
C
Females at any age are more susceptible to cystitis than men because of the shorter urethra in women. Postmenopausal women who are not on hormone replacement therapy are at increased risk for bacterial cystitis because of changes in the cells of the urethra and vagina. The middle-aged woman who has never been pregnant would not have a risk potential as high as the older woman who is not using hormone replacement therapy.
61. The nurse is admitting a client with a diagnosis
of rule-out cancer of the larynx. Which
information should the nurse teach?
1. Demonstrate the proper method of gargling
with normal saline.
2. Perform voice exercises for 30 minutes three
(3) times a day.
3. Explain that a lighted instrument will be
placed in the throat to biopsy the area.
4. Teach the client to self-examine the larynx
monthly.
3. A laryngoscopy will be performed to
allow for visualization of the vocal cords
and to obtain a biopsy for pathological
diagnosis.
TEST-TAKING HINT: The test taker must understand
that, if the question states the client
is admitted to "rule out" a disease, then
diagnostic tests and procedures will be done
to determine if, in fact, the client has the diagnosis
that the HCP suspects.
The nurse is performing a hematologic assessment of an older adult patient. Which findings does the nurse identify as normal changes in the older adult?
(Select all that apply.)
a. Progressive loss of body hair
b. Thickened or discolored nails
c. Yellowing of the skin
d. Dryness of the skin
e. Ecchymosis
a, b, c, d
A client with unstable angina has received education about acute coronary syndrome. Which statement indicates that the client has understood the teaching?
"This is a big warning; I must modify my lifestyle or risk having a heart attack in the next year."
"Angina is just a temporary interruption of blood flow to my heart."
"I need to tell my wife I've had a heart attack."
"Because this was temporary, I will not need to take any medications for my heart."
"This is a big warning; I must modify my lifestyle or risk having a heart attack in the next year."
Among people who have unstable angina, 10% to 30% have a myocardial infarction within 1 year. Although anginal pain is temporary, it reflects underlying coronary artery disease (CAD), which requires attention, including lifestyle modifications. Unstable angina reflects tissue ischemia, but infarction represents tissue necrosis. Clients with underlying CAD may need medications such as aspirin, lipid-lowering agents, anti-anginals, or antihypertensives.
1. Problems in the hypothalamus that change the function of the anterior pituitary gland result in which condition?
a. Adenohypophysis
b. Panhypopituitarism
c. Primary pituitary dysfunction
d. Secondary pituitary dysfunction
d
Secondary pituitary dysfunction
1. A 22-year-old patient comes to the clinic for a wellness check-up. History reveals that the patient's parent has the autosomal-dominant form of polycystic kidney disease (PKD). Which vital sign suggests that the patient should be evaluated for PKD?
a. Pulse of 90 beats/min
b. Temperature of 99.6 F
c. Blood pressure of 136/88 mm Hg
d. Respiratory rate of 22/min
c
1. A nurse answers a call light and finds a client anxious, short of breath, reporting chest pain, and having a blood pressure of 88/52 mm Hg on the cardiac monitor. What action by the nurse takes priority?
a. Assess the client's lung sounds.
b. Notify the Rapid Response Team. c. Provide reassurance to the client. d. Take a full set of vital signs.
ANS: B
This client has manifestations of a pulmonary embolism, and the most critical action is to notify the Rapid Response Team for speedy diagnosis and treatment. The other actions are appropriate also but are not the priority.
1. The nurse is interviewing a patient who is newly admitted to the unit with a diagnosis of anemia. Which assessment findings does the nurse expect? (Select all that apply.)
a. Dyspnea on exertion
b. Systolic hypertension
c. Intolerance to heat
d. Concave appearance of the nails
e. Pallor of the ears
f. Headache
a, d, e, f
1. A nurse assesses a client's electrocardiograph tracing and observes that not all QRS complexes are preceded by a P wave. How should the nurse interpret this observation? a. The client has hyperkalemia causing irregular QRS complexes.
b. Ventricular tachycardia is overriding the normal atrial rhythm.
c. The client's chest leads are not making sufficient contact with the skin.
d. Ventricular and atrial depolarizations are initiated from different sites.
ANS: D
Normal rhythm shows one P wave preceding each QRS complex, indicating that all depolarization is initiated at the sinoatrial node. QRS complexes without a P wave indicate a different source of initiation of depolarization. This finding on an electrocardiograph tracing is not an indication of hyperkalemia, ventricular tachycardia, or disconnection of leads.
DIF: Understanding/Comprehension REF: 649
KEY: Cardiac electrical conduction
MSC: Integrated Process: Nursing Process: Analysis
NOT: Client Needs Category: Physiological Integrity: Physiological Adaptation
1. Which glands are parts of the endocrine system? (Select all that apply.)
a. Thyroid
b. Occipital
c. Parathyroid
d. Adrenal
e. Pituitary
a, c, d, e
Thyroid, Parathyroid, Adrenal, Pituitary
A client with unstable angina has received education about acute coronary syndrome. Which statement indicates that the client has understood the teaching?
a. "This is a big warning; I must modify my lifestyle or risk having a heart attack in the next year."
b. "Angina is just a temporary interruption of blood flow to my heart."
c. "I need to tell my wife I've had a heart attack."
d. "Because this was temporary, I will not need to take any medications for my heart."
a. "This is a big warning; I must modify my lifestyle or risk having a heart attack in the next year."
Among people who have unstable angina, 10% to 30% have a myocardial infarction within 1 year. Although anginal pain is temporary, it reflects underlying coronary artery disease (CAD), which requires attention, including lifestyle modifications. Unstable angina reflects tissue ischemia, but infarction represents tissue necrosis. Clients with underlying CAD may need medications such as aspirin, lipid-lowering agents, anti-anginals, or antihypertensives.
A nurse assesses clients for potential endocrine dysfunction. Which client is at greatest risk for a deficiency of gonadotropin and growth hormone?
a. A 36-year-old female who has used oral contraceptives for 5 years
b. A 42-year-old male who experienced head trauma 3 years ago
c. A 55-year-old female with a severe allergy to shellfish and iodine
d. A 64-year-old male with adult-onset diabetes mellitus
B (Gonadotropin and growth hormone are anterior pituitary hormones. Head trauma is a common cause of anterior pituitary hypofunction. The other factors do not increase the risk of this condition.)
1. The nurse is assessing a client with a diagnosis of pre-renal acute kidney injury (AKI). Which condition would the nurse expect to find in the client's recent history?
a. Pyelonephritis
b. Myocardial infarction
c. Bladder cancer d. Kidney stones
ANS: B
Pre-renal causes of AKI are related to a decrease in perfusion, such as with a myocardial infarction. Pyelonephritis is an intrinsic or intrarenal cause of AKI related to kidney damage. Bladder cancer and kidney stones are post-renal causes of AKI related to urine flow obstruction.
A client hospitalized with sickle cell crisis frequently asks for opioid pain medications, often shortly after receiving a dose. The nurses on the unit believe the client is drug seeking. When the client requests pain medication, what action by the nurse is best?
a. Give the client pain medication if it is time for another dose.
b. Instruct the client not to request pain medication too early.
c. Request the provider leave a prescription for a placebo.
d. Tell the client it is too early to have more pain medication.
a. Give the client pain medication if it is time for another dose.
1. A nurse assesses several clients who have a history of asthma. Which client should the nurse assess first?
a. A 66-year-old client with a barrel chest and clubbed fingernails
b. A 48-year-old client with an oxygen saturation level of 92% at rest
c. A 35-year-old client who has a longer expiratory phase than inspiratory phase
d. A 27-year-old client with a heart rate of 120 beats/min
ANS: D
Tachycardia can indicate hypoxemia as the body tries to circulate the oxygen that is available. A barrel chest is not an emergency finding. Likewise, a pulse oximetry level of 92% is not considered an acute finding. The expiratory phase is expected to be longer than the inspiratory phase in someone with airflow limitation.
DIF: Applying/Application REF: 552
KEY: Respiratory distress/failure| assessment/diagnostic examination
MSC: IntegratedProcess:NursingProcess:Assessment
NOT: Client Needs Category: Safe and Effective Care Environment: Management of Care
1. The home health nurse reads in the patient's chart that the patient has asymptomatic bacterial urinary tract infection (ABUTI). Which intervention will the nurse perform?
a. Obtain an order for urinalysis and urine culture and sensitivity.
b. Check the patient's medication list for appropriate antibiotic order.
c. Closely monitor for conditions that cause progression to acute infection.
d. Ask the patient when the ABUTI first started and when it was diagnosed.
c
2. A nurse assesses a client who has a nasal fracture. The client reports constant nasal drainage, a headache, and difficulty with vision. Which action should the nurse take next?
a. Collect the nasal drainage on a piece of filter paper.
b. Encourage the client to blow his or her nose.
c. Perform a test focused on a neurologic examination.
d. Palpate the nose, face, and neck.
ANS: A
The client with nasal drainage after facial trauma could have a skull fracture that has resulted in leakage of cerebrospinal fluid (CSF). CSF can be differentiated from regular drainage by the fact that it forms a halo when dripped on filter paper. The other actions would be appropriate but are not as high a priority as assessing for CSF. A CSF leak would increase the client's risk for infection.
DIF: Applying/Application REF: 532
KEY: Trauma| medical emergencies
MSC: Integrated Process: Nursing Process: Implementation
NOT: Client Needs Category: Physiological Integrity: Reduction of Risk Potential
p. 802, Ethical/Legal
A 27-year-old African-American man in sickle cell crisis is a patient on your unit. During report, one of the nurses from the previous shift mentions that she withheld the regularly scheduled IV opioid pain medication during the night because she had taken care of this patient a year ago and believes that he is a "drug seeker."
1. What is your first action?
Stop report and go and assess the patient for pain and administer the prescribed scheduled opioid. The pain during crisis is intense. Pain medication, including opioids, must be given on a scheduled basis and not as needed, and they should not be withheld.
an older client has a history of coronary artery disease. which modifiable risk factors will the nurse assess to guide the client's teaching plan? select all that apply.
A. older age
B. tobacco use
C. female
D. high-fat diet
E. family history
F. obesity
B, D, F
The nurse is performing a physical examination of a patient's thyroid gland. Precautions are taken in performing the correct technique because palpation can result in which occurrence?
a. Damage to the esophagus causing gastric reflux
b. Obstruction of the carotid arteries causing a stroke c. Pressure on the trachea and laryngeal nerve causing
hoarseness
d. Exacerbation of symptoms by releasing additional
thyroid hormone
d
2. A client received tissue plasminogen activator (t-PA) after a myocardial infarction and now is on an intravenous infusion of heparin. The client's spouse asks why the client needs this medication. What response by the nurse is best?
a. "The t-PA didn't dissolve the entire coronary clot."
b. "The heparin keeps that artery from getting blocked again."
c. "Heparin keeps the blood as thin as possible for a longer time."
d. "The heparin prevents a stroke from occurring as the t-PA wears off."
ANS: B
After the original intracoronary clot has dissolved, large amounts of thrombin are released into the bloodstream, increasing the chance of the vessel reoccluding. The other statements are not accurate. Heparin is not a "blood thinner," although laypeople may refer to it as such.
DIF: Understanding/Comprehension REF: 768
KEY: Coronary artery disease| thrombolytic agents| patient education
MSC: IntegratedProcess:Teaching/Learning
NOT: Client Needs Category: Physiological Integrity: Pharmacological and Parenteral Therapies
p. 1259, Physiological Integrity
What effect on circulating levels of sodium and glucose does the nurse expect in a client who has been taking an oral cortisol preparation for 2 years because of a respiratory problem?
A. Decreased sodium; decreased glucose
B. Decreased sodium; increased glucose
C. Increased sodium; decreased glucose
D. Increased sodium; increased glucose
Answer: D
Rationale: Any of the glucocorticoids have some mineralocorticoid activity and increase the reabsorption of sodium from the kidney tubules, thus increasing the serum sodium level. Cortisol also increases liver production of glucose (gluconeogenesis) and inhibits peripheral glucose uptake by the cells. Both of these actions increase blood glucose levels.
2. A patient with sickle cell crisis is admitted to the hospital. Which questions does the nurse ask the patient to elicit information about the cause of the current crisis? (Select all that apply.)
a. "Have you recently traveled on an airplane?"
b. "Have you ever had radiation therapy?"
c. "In the past 24 hours, has any activity made you short of breath?"
d. "Have you recently consumed alcohol or used recreational drugs?"
e. "Have you had any symptoms of infection, such as fever?"
a, c, d, e
62. The client is diagnosed with cancer of the larynx
and is to have radiation therapy to the area.
Which prophylactic procedure will the nurse
prepare the client for?
1. Removal of the client's teeth and fitting for
dentures.
2. Take antiemetic medications every four
(4) hours.
3. Wear sunscreen on the area at all times.
4. Placement of a nasogastric feeding tube.
1. The teeth will be in the area of radiation
and the roots of teeth are highly sensitive
to radiation, which results in root
abscesses. The teeth are removed and
the client is fitted for dentures prior to
radiation.
TEST-TAKING HINT: The test taker could eliminate
option "4" as a form of nutritional treatment,
not prophylaxis. The test taker must
recognize which anatomical structures would
lie within the radiation beam. The teeth of the
lower jaw definitely are within the port, and
the upper teeth possibly would be in range.
1. Which problems occur with acute kidney injury (AKI)? (Select all that apply.)
a. decreased peristalsis
b. Anemia
c. Metabolic acidosis
d. Hypokalemia
e. Peripehral edema
a, b, c, e
1. A patient has sustained a minor kidney injury. Which structure must remain functional in order to form urine from blood?
a. Medulla
b. Nephron
c. Calyx
d. Capsule
b
2. A nurse cares for a client with arthritis who reports frequent asthma attacks. Which action should the nurse take first?
a. Review the client's pulmonary function test results.
b. Ask about medications the client is currently taking.
c. Assess how frequently the client uses a bronchodilator.
d. Consult the provider and request arterial blood gases.
ANS: B
Aspirin and other nonsteroidal anti-inflammatory drugs (NSAIDs) can trigger asthma in some people. This results from increased production of leukotriene when aspirin or NSAIDs suppress other inflammatory pathways and is a high priority given the client's history. Reviewing pulmonary function test results will not address the immediate problem of frequent asthma attacks. This is a good intervention for reviewing response to bronchodilators. Questioning the client about the use of bronchodilators will address interventions for the attacks but not their cause. Reviewing arterial blood gas results would not be of use in a client between attacks because many clients are asymptomatic when not having attacks.
DIF: Applying/Application REF: 553
KEY: Respiratory distress/failure| medication
MSC: IntegratedProcess:NursingProcess:Assessment
NOT: Client Needs Category: Physiological Integrity: Pharmacological and Parenteral Therapies
Which statement about hematologic changes associated with aging is true?
a. The older adult has increased
blood volume.
b. The older adult has increased
levels of plasma proteins.
c. Platelet counts decrease with age.
d. Antibody levels and responses are lower and slower
in older adults.
d
The nurse is caring for a group of clients who have sustained myocardial infarction (MI). The nurse observes the client with which type of MI most carefully for the development of left ventricular heart failure?
Inferior wall
Anterior wall
Lateral wall
Posterior wall
Anterior wall
Due to the large size of the anterior wall, the amount of tissue infarction may be large enough to decrease the force of contraction, leading to heart failure. The client with an inferior wall MI is more likely to develop right ventricular heart failure. Clients with obstruction of the circumflex artery may experience a lateral wall MI and sinus dysrhythmias or a posterior wall MI and sinus dysrhythmias.
A nurse is teaching a client with diabetes mellitus who asks, "Why is it necessary to maintain my blood glucose levels no lower than about 60 mg/dL?" How should the nurse respond?
a. "Glucose is the only fuel used by the body to produce the energy that it needs."
b. "Your brain needs a constant supply of glucose because it cannot store it."
c. "Without a minimum level of glucose, your body does not make red blood cells."
d. "Glucose in the blood prevents the formation of lactic acid and prevents acidosis."
ANS: B
Because the brain cannot synthesize or store significant amounts of glucose, a continuous supply from the body's circulation is needed to meet the fuel demands of the central nervous system. The nurse would want to educate the client to prevent hypoglycemia. The body can use other sources of fuel, including fat and protein, and glucose is not involved in the production of red blood cells. Glucose in the blood will encourage glucose metabolism but is not directly responsible for lactic acid formation.
2. A marathon runner comes into the clinic and states "I have not urinated very much in the last few days." The nurse notes a heart rate of 110 beats/min and a blood pressure of 86/58 mm Hg. Which action by the nurse is the priority?
a. Give the client a bottle of water immediately.
b. Start an intravenous line for fluids.
c. Teach the client to drink 2 to 3 liters of water daily. d. Perform an electrocardiogram.
ANS: A
This athlete is mildly dehydrated as evidenced by the higher heart rate and lower blood pressure. The nurse can start hydrating the client with a bottle of water first, followed by teaching the client to drink 2 to 3 liters of water each day. An intravenous line may be ordered later, after the client's degree of dehydration is assessed. An electrocardiogram is not necessary at this time.
2. A client is admitted with a pulmonary embolism (PE). The client is young, healthy, and active and has no known risk factors for PE. What action by the nurse is most appropriate? a. Encourage the client to walk 5 minutes each hour.
b. Refer the client to smoking cessation classes.
c. Teach the client about factor V Leiden testing.
d. Tell the client that sometimes no cause for disease is found.
ANS: C
Factor V Leiden is an inherited thrombophilia that can lead to abnormal clotting events, including PE. A client with no known risk factors for this disorder should be referred for testing. Encouraging the client to walk is healthy, but is not related to the development of a PE in this case, nor is smoking. Although there are cases of disease where no cause is ever found, this assumption is premature.
A client is receiving rivaroxaban (Xarelto) and asks the nurse to explain how it works. What response by the nurse is best?
a.
"It inhibits thrombin."
b.
"It inhibits fibrinogen."
c.
"It thins your blood."
d.
"It works against vitamin K."
a.
"It inhibits thrombin."
ANS: A
Rivaroxaban is a direct thrombin inhibitor. It does not work on fibrinogen or vitamin K. It is not a "blood thinner," although many clients call anticoagulants by this name.
The nurse is caring for a group of clients who have sustained myocardial infarction (MI). The nurse observes the client with which type of MI most carefully for the development of left ventricular heart failure?
a. Inferior wall
b. Anterior wall
c. Lateral wall
d. Posterior wall
b. Anterior wall
Due to the large size of the anterior wall, the amount of tissue infarction may be large enough to decrease the force of contraction, leading to heart failure. The client with an inferior wall MI is more likely to develop right ventricular heart failure. Clients with obstruction of the circumflex artery may experience a lateral wall MI and sinus dysrhythmias or a posterior wall MI and sinus dysrhythmias.
Ch.61 Question 1 of 16
The nurse is assessing a client for endocrine dysfunction. Which comment by the client indicates a need for further assessment?
a. "I am worried about losing my job because of cutbacks."
b. "I don't have any patience with my kids. I lose my temper faster."
c. "I don't seem to have any stressors now."
d. "My weight has been stable these past few years."
b
"I don't have any patience with my kids. I lose my temper faster."
Many endocrine problems can change a client's behavior, personality, and psychological responses; the client stating that he or she has become short-tempered warrants further assessment. Worrying about losing a job is a normal concern but does not give any indication of a need for further assessment. The nurse will need to assess the client's claim that he or she has no stressors at present because the client's response does not provide enough information to make this determination; however, the client's statement about losing patience is the priority. Weight gain or loss may or may not be an indication of an endocrine disorder.
1. The nurse is performing a physical examination of a patient's thyroid gland. Precautions are taken in performing the correct technique because palpitation can result in which occurrence?
a. Damage to the esophagus causing gastric reflux
b. Obstruction of the carotid arteries causing a stroke
c. Pressure on the trachea and laryngeal nerve causing hoarseness
d. Exacerbation of symptoms by releasing additional thyroid hormone
d
a client weighing 174 lbs had thrombolytic therapy followed by a one-time dose of IV lovenox 30 mg. the physician prescribes lovenox 1 mg/kg subcutaneously after the IV administration. the nurse will give ____ mg of lovenox to the client.
79
A client in sickle cell crisis is dehydrated and in the emergency department. The nurse plans to start an IV. Which fluid choice is best?
a. 0.45% normal saline
b. 0.9% normal saline
c. Dextrose 50% (D50)
d. Lactated Ringer's solution
a. 0.45% normal saline
2. A nurse in a family practice clinic is preparing discharge instructions for a client reporting facial pain that is worse when bending over, tenderness across the cheeks, and postnasal discharge. What instruction will be most helpful?
a. "Ice packs may help with the facial pain."
b. "Limit fluids to dry out your sinuses."
c. "Try warm, moist heat packs on your face."
d. "We will schedule you for a computed tomography scan this week."
ANS: C
This client has rhinosinusitis. Comfort measures for this condition include breathing in warm steam, hot packs, nasal saline irrigations, sleeping with the head elevated, increased fluids, and avoiding cigarette smoke. The client does not need a CT scan.
2. A nurse reviews the laboratory findings of a client with a urinary tract infection. The laboratory report notes a "shift to the left" in a client's white blood cell count. Which action should the nurse take?
a. Request that the laboratory perform a differential analysis on the white blood cells.
b. Notify the provider and start an intravenous line for parenteral antibiotics.
c. Collaborate with the unlicensed assistive personnel (UAP) to strain the client's
urine for renal calculi.
d. Assess the client for a potential allergic reaction and anaphylactic shock.
ANS: B
An increase in band cells creates a "shift to the left." A left shift most commonly occurs with urosepsis and is seen rarely with uncomplicated urinary tract infections. The nurse will be administering antibiotics, most likely via IV, so he or she should notify the provider and prepare to give the antibiotics. The shift to the left is part of a differential white blood cell count. The nurse would not need to strain urine for stones. Allergic reactions are associated with elevated eosinophil cells, not band cells.
A nurse is caring for an 89-year-old client admitted with pneumonia. He has an IV of normal saline running at 100 mL/hr and antibiotics that were initiated in the emergency department 3 hours ago. He has oxygen at 2 liters/nasal cannula. What assessment finding by the nurse indicates that goals for a priority diagnosis have been met for this client?
A) The client is alert and oriented to person, place, and time.
B) Blood pressure is within normal limits and client's baseline.
C) Skin behind the ears demonstrates no redness or irritation.
D) Urine output has been >30 mL/hr per Foley catheter.
(Chp. 31; p. 593)
A) The client is alert and oriented to person, place, and time.
(Chp. 31; p. 593)
p. 802, Ethical/Legal
A 27-year-old African-American man in sickle cell crisis is a patient on your unit. During report, one of the nurses from the previous shift mentions that she withheld the regularly scheduled IV opioid pain medication during the night because she had taken care of this patient a year ago and believes that he is a "drug seeker."
2. How should you approach your colleague?
After you have seen to your patient's comfort, remind her that unless there is a physiologic danger, such as severe respiratory depression, she is legally required to administer the prescribed opioid regardless of her beliefs. Determine whether she assessed the patient for pain and what, for this episode, not the patient's behavior a year ago, convinced her that the pain is not real.
2. A nurse cares for a client who has a heart rate averaging 56 beats/min with no adverse symptoms. Which activity modification should the nurse suggest to avoid further slowing of the heart rate?
a. "Make certain that your bath water is warm."
b. "Avoid straining while having a bowel movement."
c. "Limit your intake of caffeinated drinks to one a day."
d. "Avoid strenuous exercise such as running."
ANS: B
Bearing down strenuously during a bowel movement is one type of Valsalva maneuver, which stimulates the vagus nerve and results in slowing of the heart rate. Such a response is not desirable in a person who has bradycardia. The other instructions are not appropriate for this condition.
DIF: Applying/Application REF: 663 KEY: Functional ability MSC: Integrated Process: Nursing Process: Implementation
NOT: Client Needs Category: Physiological Integrity: Basic Care and Comfort
1. A nurse assesses a client with hyperthyroidism who is prescribed lithium carbonate. Which assessment finding should alert the nurse to a side effect of this therapy?
a. Blurred and double vision
b. Increased thirst and urination
c. Profuse nausea and diarrhea
d. Decreased attention and insomnia
B. Lithium antagonizes antidiuretic hormone and can cause symptoms of diabetes insipidus. This manifests with increased thirst and urination. Lithium has no effect on vision, gastric upset, or level of consciousness.
2. A nurse reviews the laboratory findings of a client with a urinary tract infection. The laboratory report notes a shift to the left in a clients white blood cell count. Which action should the nurse take?
a. Request that the laboratory perform a differential analysis on the white blood cells.
b. Notify the provider and start an intravenous line for parenteral antibiotics.
c. Collaborate with the unlicensed assistive personnel (UAP) to strain the clients urine for renal calculi.
d. Assess the client for a potential allergic reaction and anaphylactic shock.
B
An increase in band cells creates a shift to the left. A left shift most commonly occurs with urosepsis and is seen rarely with uncomplicated urinary tract infections. The nurse will be administering antibiotics, most likely via IV, so he or she should notify the provider and prepare to give the antibiotics. The shift to the left is part of a differential white blood cell count. The nurse would not need to strain urine for stones. Allergic reactions are associated with elevated eosinophil cells, not band cells.
2. A nurse assesses a client in an outpatient clinic. Which statement alerts the nurse to the possibility of left-sided heart failure?
a. "I have been drinking more water than usual."
b. "I am awakened by the need to urinate at night."
c. "I must stop halfway up the stairs to catch my breath."
d. "I have experienced blurred vision on several occasions."
ANS: C
Clients with left-sided heart failure report weakness or fatigue while performing normal activities of daily living, as well as difficulty breathing, or "catching their breath." This occurs as fluid moves into the alveoli. Nocturia is often seen with right-sided heart failure. Thirst and blurred vision are not related to heart failure.
DIF: Understanding/Comprehension REF: 682
KEY: Heart failure| assessment/diagnostic examination
MSC: IntegratedProcess:NursingProcess:Assessment
NOT: Client Needs Category: Health Promotion and Maintenance
A nurse cares for a client who is prescribed a drug that blocks a hormone's receptor site. Which therapeutic effect should the nurse expect?
a. Greater hormone metabolism
b. Decreased hormone activity
c. Increased hormone activity
d. Unchanged hormone response
ANS: B
Hormones cause activity in the target tissues by binding with their specific cellular receptor sites, thereby changing the activity of the cell. When receptor sites are occupied by other substances that block hormone binding, the cell's response is the same as when the level of the hormone is decreased.
3. A patient is scheduled to undergo diagnostic testing for sickle cell anemia. For which diagnostic test does the nurse provide patient teaching?
a. Bone marrow biopsy
b. Platelet count
c. Philadelphia chromosome analysis
d. Hemoglobin S
d
A client has been admitted to the hospital with suspected TB. What drugs should the nurse plan to teach the client about before discharge? Select all that apply.
A) Rifampin (Rifadin); contact lenses can become stained orange
B) Isoniazid (INH); report yellowing of the skin or darkened urine
C) Pyrazinamide (PZA); maintain a fluid restriction of 1200 mL/day
D) Ethambutol (Myambutol); report any changes in vision
E) Amoxicillin (Amoxil); take this drug with food or milk
(Chp. 31; p. 598)
A) Rifampin (Rifadin); contact lenses can become stained orange
B) Isoniazid (INH); report yellowing of the skin or darkened urine
D) Ethambutol (Myambutol); report any changes in vision
(Chp. 31; p. 598)
2. A nurse cares for a client with autosomal dominant polycystic kidney disease (ADPKD). The client asks, "Will my children develop this disease?" How should the nurse respond? a. "No genetic link is known, so your children are not at increased risk."
b. "Your sons will develop this disease because it has a sex-linked gene."
c. "Only if both you and your spouse are carriers of this disease." d. "Each of your children has a 50% risk of having ADPKD."
ANS: D
Children whose parent has the autosomal dominant form of PKD have a 50% chance of inheriting the gene that causes the disease. ADPKD is transmitted as an autosomal dominant trait and therefore is not gender specific. Both parents do not need to have this disorder.
3. Which teaching point is most important for the client with bacterial pharyngitis? a. Gargle with warm salt water.
b. Take all antibiotics as directed.
c. Use a humidifier in the bedroom.
d. Wash hands frequently.
ANS: B
Any client on antibiotics must be instructed to complete the entire course of antibiotics. Not completing them can lead to complications or drug-resistant strains of bacteria. The other instructions are appropriate, just not the most important.
For a patient who has a dysfunction of the bone marrow, which sign/symptom is the nurse most likely to observe?
a. Long bone pain
b. Fatigue
c. Loss of appetite
d. Weight gain
b
2. A nurse assesses a client after administering a prescribed beta blocker. Which assessment should the nurse expect to find?
a. Blood pressure increased from 98/42 mm Hg to 132/60 mm Hg
b. Respiratory rate decreased from 25 breaths/min to 14 breaths/min
c. Oxygen saturation increased from 88% to 96%
d. Pulse decreased from 100 beats/min to 80 beats/min
D
Beta blockers block the stimulation of beta1-adrenergic receptors. They block the sympathetic (fight-or-flight) response and decrease the heart rate (HR). The beta blocker will decrease HR and blood pressure, increasing ventricular filling time. It usually does not have effects on beta2-adrenergic receptor sites. Cardiac output will drop because of decreased HR.
Ch.61 p. 1259, Physiological Integrity
What effect on circulating levels of sodium and glucose does the nurse expect in a client who has been taking an oral cortisol preparation for 2 years because of a respiratory problem?
A. Decreased sodium; decreased glucose
B. Decreased sodium; increased glucose
C. Increased sodium; decreased glucose
D. Increased sodium; increased glucose
Answer: D
Rationale: Any of the glucocorticoids have some mineralocorticoid activity and increase the reabsorption of sodium from the kidney tubules, thus increasing the serum sodium level. Cortisol also increases liver production of glucose (gluconeogenesis) and inhibits peripheral glucose uptake by the cells. Both of these actions increase blood glucose levels.
2. Which event is most likely to trigger renin production?
a. Patient participates in strenouous exercise.
b. Patient becomes anxious and nervous.
c. Patient has urge to urinate during the night.
d. Patient sustains significant blood loss.
d
2. Which group has the highest prevalence of urinary tract infections (UTIs)?
a. Young men
b. Older women
c. Older men
d. School-aged girls
b
3. A client has a pulmonary embolism and is started on oxygen. The student nurse asks why the client's oxygen saturation has not significantly improved. What response by the nurse is best?
a. "Breathing so rapidly interferes with oxygenation."
b. "Maybe the client has respiratory distress syndrome."
c. "The blood clot interferes with perfusion in the lungs."
d. "The client needs immediate intubation and mechanical ventilation."
ANS: C
A large blood clot in the lungs will significantly impair gas exchange and oxygenation. Unless the clot is dissolved, this process will continue unabated. Hyperventilation can interfere with oxygenation by shallow breathing, but there is no evidence that the client is hyperventilating, and this is also not the most precise physiologic answer. Respiratory distress syndrome can occur, but this is not as likely. The client may need to be mechanically ventilated, but without concrete data on FiO2 and SaO2, the nurse cannot make that judgment.
The health care provider tells the nurse that a client is to be started on a platelet inhibitor. About what drug does the nurse plan to teach the client?
a.
Clopidogrel (Plavix)
b.
Enoxaparin (Lovenox)
c.
Reteplase (Retavase)
d.
Warfarin (Coumadin)
a.
Clopidogrel (Plavix)
ANS: A
Clopidogrel is a platelet inhibitor. Enoxaparin is an indirect thrombin inhibitor. Reteplase is a fibrinolytic agent. Warfarin is a vitamin K antagonist.
3. A nurse is assessing clients on a medical-surgical unit. Which client should the nurse identify as being at greatest risk for atrial fibrillation?
a. A 45-year-old who takes an aspirin daily
b. A 50-year-old who is post coronary artery bypass graft surgery
c. A 78-year-old who had a carotid endarterectomy
d. An 80-year-old with chronic obstructive pulmonary disease
ANS: B
Atrial fibrillation occurs commonly in clients with cardiac disease and is a common occurrence after coronary artery bypass graft surgery. The other conditions do not place these clients at higher risk for atrial fibrillation.
DIF: Applying/Application REF: 666
KEY: Health screening| cardiac electrical conduction
MSC: IntegratedProcess:NursingProcess:Assessment
NOT: Client Needs Category: Safe and Effective Care Environment: Management of Care
A nurse cares for a client with a deficiency of aldosterone. Which assessment finding should the nurse correlate with this deficiency?
a. Increased urine output
b. Vasoconstriction
c. Blood glucose of 98 mg/dL
d. Serum sodium of 144 mEq/L
ANS: A
Aldosterone, the major mineralocorticoid, maintains extracellular fluid volume. It promotes sodium and water reabsorption and potassium excretion in the kidney tubules. A client with an aldosterone deficiency will have increased urine output. Vasoconstriction is not related. These sodium and glucose levels are normal; in aldosterone deficiency, the client would have hyponatremia and hyperkalemia.
3. A nurse assesses a client admitted to the cardiac unit. Which statement by the client alerts the nurse to the possibility of right-sided heart failure?
a. "I sleep with four pillows at night."
b. "My shoes fit really tight lately."
c. "I wake up coughing every night."
d. "I have trouble catching my breath."
ANS: B
Signs of systemic congestion occur with right-sided heart failure. Fluid is retained, pressure builds in the venous system, and peripheral edema develops. Left-sided heart failure symptoms include respiratory symptoms. Orthopnea, coughing, and difficulty breathing all could be results of left-sided heart failure.
DIF: Understanding/Comprehension REF: 683
KEY: Heart failure| assessment/diagnostic examination
MSC: IntegratedProcess:NursingProcess:Assessment
NOT: Client Needs Category: Health Promotion and Maintenance
A client with iatrogenic Cushing's syndrome is a resident in a long-term care facility. Which nursing action included in the client's care would be best to delegate to unlicensed assistive personnel (UAP)?
A. Assist with personal hygiene and skin care.
B. Develop a plan of care to minimize risk for infection.
C. Instruct the client on the reasons to avoid overeating.
D. Monitor for signs and symptoms of fluid retention.
A.
Assisting a client with bathing and skin care is included in UAP scope of practice. It is not within their scope of practice to develop a plan of care, although they will play a very important role in following the plan of care. Client teaching requires a broad education and should not be delegated to UAP. Monitoring for signs and symptoms of fluid retention is part of client assessment, which requires a higher level of education and clinical judgment.
A nurse is assessing a dark-skinned client for pallor. What action is best?
a.
Assess the conjunctiva of the eye.
b.
Have the client open the hand widely.
c.
Look at the roof of the client's mouth.
d.
Palpate for areas of mild swelling.
a.
Assess the conjunctiva of the eye.
ANS: A
To assess pallor in dark-skinned people, assess the conjunctiva of the eye or the mucous membranes. Looking at the roof of the mouth can reveal jaundice. Opening the hand widely is not related to pallor, nor is palpating for mild swelling.
2. The community health nurse is designing programs to reduce kidney problems and kidney injury among the general public. In order to do so, the nurse targets health promotion and compliance with therapy for people with which conditions?
a. Diabetes mellitus and hypertension
b. Frequent episodes of sexually transmitted disease
c. Osteoporosis and other bone diseases
d. Gastroenteritis and poor eating habits
a
3. After teaching a client who is prescribed a long-acting beta2 agonist medication, a nurse assesses the client's understanding. Which statement indicates the client comprehends the teaching?
a. "I will carry this medication with me at all times in case I need it."
b. "I will take this medication when I start to experience an asthma attack."
c. "I will take this medication every morning to help prevent an acute attack."
d. "I will be weaned off this medication when I no longer need it."
ANS: C
Long-acting beta2 agonist medications will help prevent an acute asthma attack because they are long acting. The client will take this medication every day for best effect. The client does not have to always keep this medication with him or her because it is not used as a rescue medication. This is not the medication the client will use during an acute asthma attack because it does not have an immediate onset of action. The client will not be weaned off this medication because this is likely to be one of his or her daily medications.
DIF: Applying/Application REF: 554
KEY: Medication| patient education MSC: Integrated Process: Teaching/Learning NOT: Client Needs Category: Physiological Integrity: Pharmacological and Parenteral Therapies
2. Which description of the recessive form of PKD is correct?
a. Prognosis is better for the recessive form compared to the dominant form.
b. 100% of people with this form of PKD develop kidney failure around age 50.
c. Most people with this form of PKD die in early childhood.
d. The recessive from only manifests if other kidney problems occurs.
c
63. The client is three (3) days post-partial
laryngectomy. Which type of nutrition should
the nurse offer the client?
1. Total parenteral nutrition.
2. Soft, regular diet.
3. Partial parenteral nutrition.
4. Clear liquid diet.
2. The client should be eating normal foods
by this time. The consistency should be
soft to allow for less chewing of the food
and easier swallowing because a portion of
the throat musculature has been removed.
The client should be taught to turn the
head toward the affected side when swallowing
to help prevent aspiration.
TEST-TAKING HINT: The keys to this question
are "three (3) days" and "partial." Clients are
progressed rapidly after surgery to as normal
a life as possible.
In the bone marrow of an older adult, what would be considered a normal physiologic change related to aging?
a. Fatty tissue replaces bone
marrow.
b. Bone marrow cells become
smaller.
c. Weakened bones absorb bone
marrow.
d. Bone marrow cells fail to function.
a
3. A nurse assesses clients on a medical-surgical unit. Which client should the nurse identify as having the greatest risk for cardiovascular disease?
a. An 86-year-old man with a history of asthma
b. A 32-year-old Asian-American man with colorectal cancer
c. A 45-year-old American Indian woman with diabetes mellitus
d. A 53-year-old postmenopausal woman who is on hormone therapy
C
The incidence of coronary artery disease and hypertension is higher in American Indians than in whites or Asian Americans. Diabetes mellitus increases the risk for hypertension and coronary artery disease in people o any race or ethnicity. Asthma, colorectal cancer, and hormone therapy do not increase risk for cardiovascular disease.
2. A malfunctioning posterior pituitary gland can result in which disorders? (Select all that apply.)
a. Hypothyroidism
b. Altered sexual function
c. Diabetes insipidus (DI)
d. Growth retardation
e. Syndrome of inappropriate antidiuretic hormone (SIADH)
c, e
Diabetes insipidus
Syndrome of inappropriate antidiuretic hormone (SIADH)
2. A nurse reviews the health history of a client with an oversecretion of renin. Which disorder should the nurse correlate with this assessment finding?
a. Alzheimer's disease
b. Hypertension
c. Diabetes mellitus d. Viral hepatitis
ANS: B
Renin is secreted when special cells in the distal convoluted tubule, called the macula densa, sense changes in blood volume and pressure. When the macula densa cells sense that blood volume, blood pressure, or blood sodium levels are low, renin is secreted. Renin then converts angiotensinogen into angiotensin I. This leads to a series of reactions that cause secretion of the hormone aldosterone. This hormone increases kidney reabsorption of sodium and water, increasing blood pressure, blood volume, and blood sodium levels. Inappropriate or excessive renin secretion is a major cause of persistent hypertension. Renin has no impact on Alzheimer's disease, diabetes mellitus, or viral hepatitis.
3. A nurse assesses a client who reports waking up feeling very tired, even after 8 hours of good sleep. Which action should the nurse take first?
a. Contact the provider for a prescription for sleep medication.
b. Tell the client not to drink beverages with caffeine before bed.
c. Educate the client to sleep upright in a reclining chair.
d. Ask the client if he or she has ever been evaluated for sleep apnea.
ANS: D
Clients are usually unaware that they have sleep apnea, but it should be suspected in people who have persistent daytime sleepiness and report waking up tired. Causes of the problem should be assessed before the client is offered suggestions for treatment.
DIF: Applying/Application REF: 535
KEY: Respiratory distress/failure
MSC: IntegratedProcess:NursingProcess:Assessment
NOT: Client Needs Category: Physiological Integrity: Physiological Adaptation
p. 802, Ethical/Legal
A 27-year-old African-American man in sickle cell crisis is a patient on your unit. During report, one of the nurses from the previous shift mentions that she withheld the regularly scheduled IV opioid pain medication during the night because she had taken care of this patient a year ago and believes that he is a "drug seeker."
3. Can a patient with sickle cell disease become addicted to opioids?
It is possible for any patient with chronic pain that may have acute exacerbations to become addicted to opioids. However, this should never be assumed. Even if addiction did occur, the pain is real and must be appropriately managed, including the use of opioid analgesics.
the nurse assesses a client who had a coronary artery bypass graft yesterday. which assessment finding will the nurse report to the surgeon immediately?
A. incisional pain
B. BP of 136/76
C. decreased level of consciousness
D. apical pulse of 88
C
2. What is the name of the substance secreted by the endocrine glands?
a. Vasoactive amines
b. Chemotaxins
c. Hormones
d. Cytotoxins
c
Hormones
The nurse is teaching a group of teens about prevention of heart disease. Which point should the nurse emphasize?
Reduce abdominal fat.
Avoid stress.
Do not smoke or chew tobacco.
Avoid alcoholic beverages.
Do not smoke or chew tobacco.
Tobacco exposure, including secondhand smoke, reduces coronary blood flow; causes vasoconstriction, endothelial dysfunction, and thickening of the vessel walls; increases carbon monoxide; and decreases oxygen. Because it is highly addicting, beginning smoking in the teen years may lead to decades of exposure. Teens are not likely to experience metabolic syndrome from obesity, but are very likely to use tobacco. Avoiding stress is a less modifiable risk factor, which is less likely to cause heart disease in teens. The risk of smoking outweighs the risk of alcohol use.
A nurse plans care for a client with a growth hormone deficiency. Which action should the nurse include in this clients plan of care?
a. Avoid intramuscular medications.
b. Place the client in protective isolation.
c. Use a lift sheet to re-position the client.
d. Assist the client to dangle before rising.
C (In adults, growth hormone is necessary to maintain bone density and strength. Adults with growth hormone deficiency have thin, fragile bones. Avoiding IM medications, using protective isolation, and assisting the client as he or she moves from sitting to standing will not serve as safety measures when the client is deficient in growth hormone.)
4. A nurse teaches a client who has open vocal cord paralysis. Which technique should the nurse teach the client to prevent aspiration?
a. Tilt the head back as far as possible when swallowing.
b. Tuck the chin down when swallowing.
c. Breathe slowly and deeply while swallowing.
d. Keep the head very still and straight while swallowing.
ANS: B
The client with open vocal cord paralysis may aspirate. The nurse should teach the client to tuck in his or her chin during swallowing to prevent aspiration. Tilting the head back would increase the chance of aspiration. Breathing slowly would not decrease the risk of aspiration, but holding the breath would. Keeping the head still and straight would not decrease the risk for aspiration.
DIF: Applying/Application REF: 535
KEY: Aspiration precaution MSC: Integrated Process: Teaching/Learning NOT: Client Needs Category: Physiological Integrity: Reduction of Risk Potential
A client presents to the emergency department in sickle cell crisis. What intervention by the nurse takes priority?
a. Administer oxygen.
b. Apply an oximetry probe.
c. Give pain medication.
d. Start an IV line.
a. Administer oxygen.
3. A male client comes into the emergency department with a serum creatinine of 2.2 mg/dL and a blood urea nitrogen (BUN) of 24 mL/dL. What question should the nurse ask first when taking this client's history?
a. "Have you been taking any aspirin, ibuprofen, or naproxen recently?"
b. "Do you have anyone in your family with renal failure?" c. "Have you had a diet that is low in protein recently?"
d. "Has a relative had a kidney transplant lately?"
ANS: A
There are some medications that are nephrotoxic, such as the nonsteroidal anti-inflammatory drugs ibuprofen, aspirin, and naproxen. This would be a good question to initially ask the client since both the serum creatinine and BUN are elevated, indicating some renal problems. A family history of renal failure and kidney transplantation would not be part of the questioning and could cause anxiety in the client. A diet high in protein could be a factor in an increased BUN.
3. A nurse cares for a postmenopausal client who has had two episodes of bacterial urethritis in the last 6 months. The client asks, "I never have urinary tract infections. Why is this happening now?" How should the nurse respond?
a. "Your immune system becomes less effective as you age."
b. "Low estrogen levels can make the tissue more susceptible to infection." c. "You should be more careful with your personal hygiene in this area." d. "It is likely that you have an untreated sexually transmitted disease."
ANS: B
Low estrogen levels decrease moisture and secretions in the perineal area and cause other tissue changes, predisposing it to the development of infection. Urethritis is most common in postmenopausal women for this reason. Although immune function does decrease with aging and sexually transmitted diseases are a known cause of urethritis, the most likely reason in this client is low estrogen levels. Personal hygiene usually does not contribute to this disease process.
64. The nurse is preparing the client diagnosed
with laryngeal cancer for a laryngectomy in
the morning. Which intervention is the nurse's
priority?
1. Take the client to the intensive care unit for a
visit.
2. Explain that the client will need to ask for
pain medication.
3. Demonstrate the use of an antiembolism hose.
4. Find out if the client can read and write.
4. The client is having the vocal cords
removed and will be unable to speak.
Communication is a high priority for
this client. If the client is able to read
and write, a Magic Slate or pad of paper
should be provided. If the client is illiterate,
the nurse and the client should develop
a method of communication using
pictures.
TEST-TAKING HINT: Questions addressing
highest priority include all of the options
being possible interventions, but only one is
priority. Use Maslow's hierarchy of needs to
answer the question—safety is priority.
4. The student nurse is caring for a patient in sickle cell crisis. Which action by the student nurse warrants intervention by the supervising nurse?
a. Keeping the patient's room cool
b. Using distraction and relaxation techniques
c. Positioning painful areas of the patient with support
d. Using therapeutic touch
a
3. A client is in the hospital after suffering a myocardial infarction and has bathroom privileges. The nurse assists the client to the bathroom and notes the client's O2 saturation to be 95%, pulse 88 beats/min, and respiratory rate 16 breaths/min after returning to bed. What action by the nurse is best?
a. Administer oxygen at 2 L/min.
b. Allow continued bathroom privileges.
c. Obtain a bedside commode.
d. Suggest the client use a bedpan.
ANS: B
This client's physiologic parameters did not exceed normal during and after activity, so it is safe for the client to continue using the bathroom. There is no indication that the client needs oxygen, a commode, or a bedpan.
DIF: Applying/Application REF: 769
KEY: Coronary artery disease| activity intolerance| vital signs| nursing assessment MSC: IntegratedProcess:NursingProcess:Assessment
NOT: Client Needs Category: Physiological Integrity: Reduction of Risk Potential
Which assessment findings indicate hyperthyroidism? (Select all that apply.)
a. Weight loss with increased appetite
b. Constipation
c. Increased heart rate
d. Insomnia
e. Decreased libido
f. Heat intolerance
a, c, d, f
3. A nurse cares for a postmenopausal client who has had two episodes of bacterial urethritis in the last 6 months. The client asks, I never have urinary tract infections. Why is this happening now? How should the nurse respond?
a. Your immune system becomes less effective as you age.
b. Low estrogen levels can make the tissue more susceptible to infection.
c. You should be more careful with your personal hygiene in this area.
d. It is likely that you have an untreated sexually transmitted disease.
B
Low estrogen levels decrease moisture and secretions in the perineal area and cause other tissue changes, predisposing it to the development of infection. Urethritis is most common in postmenopausal women for this reason. Although immune function does decrease with aging and sexually transmitted diseases are a known cause of urethritis, the most likely reason in this client is low estrogen levels. Personal hygiene usually does not contribute to this disease process.
4. After teaching a client how to perform diaphragmatic breathing, the nurse assesses the client's understanding. Which action demonstrates that the client correctly understands the teaching?
a. The client lays on his or her side with his or her knees bent.
b. The client places his or her hands on his or her abdomen.
c. The client lays in a prone position with his or her legs straight.
d. The client places his or her hands above his or her head.
ANS: B
To perform diaphragmatic breathing correctly, the client should place his or her hands on his or her abdomen to create resistance. This type of breathing cannot be performed effectively while lying on the side or with hands over the head. This type of breathing would not be as effective lying prone.
DIF: Applying/Application REF: 562
KEY: Respiratory distress/failure| patient education
MSC: IntegratedProcess:Teaching/Learning
NOT: Client Needs Category: Physiological Integrity: Physiological Adaptation
A client has a serum ferritin level of 8 ng/mL and microcytic red blood cells. What action by the nurse is best?
a. Encourage high-protein foods.
b. Perform a Hemoccult test on the client's stools.
c. Offer frequent oral care.
d. Prepare to administer cobalamin (vitamin B12).
b. Perform a Hemoccult test on the client's stools.
The nurse is teaching a group of teens about prevention of heart disease. Which point should the nurse emphasize?
a. Reduce abdominal fat.
b. Avoid stress.
c. Do not smoke or chew tobacco.
d. Avoid alcoholic beverages.
c. Do not smoke or chew tobacco.
Tobacco exposure, including secondhand smoke, reduces coronary blood flow; causes vasoconstriction, endothelial dysfunction, and thickening of the vessel walls; increases carbon monoxide; and decreases oxygen. Because it is highly addicting, beginning smoking in the teen years may lead to decades of exposure. Teens are not likely to experience metabolic syndrome from obesity, but are very likely to use tobacco. Avoiding stress is a less modifiable risk factor, which is less likely to cause heart disease in teens. The risk of smoking outweighs the risk of alcohol use.
Tobacco exposure, including secondhand smoke, reduces coronary blood flow; causes vasoconstriction, endothelial dysfunction, and thickening of the vessel walls; increases carbon monoxide; and decreases oxygen. Because it is highly addicting, beginning smoking in the teen years may lead to decades of exposure. Teens are not likely to experience metabolic syndrome from obesity, but are very likely to use tobacco. Avoiding stress is a less modifiable risk factor, which is less likely to cause heart disease in teens. The risk of smoking outweighs the risk of alcohol use.
A nurse reviews laboratory results for a client with diabetes mellitus who presents with polyuria, lethargy, and a blood glucose of 560 mg/dL. Which laboratory result should the nurse correlate with the client's polyuria?
a. Serum sodium: 163 mEq/L
b. Serum creatinine: 1.6 mg/dL
c. Presence of urine ketone bodies
d. Serum osmolarity: 375 mOsm/kg
ANS: D
Hyperglycemia causes hyperosmolarity of extracellular fluid. This leads to polyuria from an osmotic diuresis. The client's serum osmolarity is high. The client's sodium would be expected to be high owing to dehydration. Serum creatinine and urine ketone bodies are not related to the polyuria.
Ch.61 p. 1264, Health Promotion and Maintenance
Which precaution or action is most important for the nurse to teach the client who is to collect a 24-hour urine specimen for endocrine testing?
A. Eat a normal diet during the collection period.
B. Wear gloves when you urinate to prevent contamination of the specimen.
C. Urinate at the end of 24 hours and add that sample to the collection container.
D. Avoid walking, running, dancing, or any vigorous exercise during the collection period.
Answer: C
Rationale: When a 24-hour urine specimen is started, the specimen should reflect all the urine produced during the specified time. The very first voiding is discarded because the urine has spent some time in the bladder and will not reflect what is happening during the actual 24 hours of the collection. The time of this discard is the beginning of the 24-hour collection period. The test requires that all urine voided after the start time be collected, including the specimen collected by emptying the bladder at end of the 24 hours, which marks the end of the test.
4. A nurse assesses a client with atrial fibrillation. Which manifestation should alert the nurse to the possibility of a serious complication from this condition?
a. Sinus tachycardia
b. Speech alterations
c. Fatigue
d. Dyspnea with activity
ANS: B
Clients with atrial fibrillation are at risk for embolic stroke. Evidence of embolic events includes changes in mentation, speech, sensory function, and motor function. Clients with atrial fibrillation often have a rapid ventricular response as a result. Fatigue is a nonspecific complaint. Clients with atrial fibrillation often have dyspnea as a result of the decreased cardiac output caused by the rhythm disturbance.
DIF: Applying/Application REF: 666
KEY: Cardiac electrical conduction| vascular perfusion
MSC: IntegratedProcess:NursingProcess:Assessment
NOT: Client Needs Category: Physiological Integrity: Reduction of Risk Potential
p. 802, Ethical/Legal
A 27-year-old African-American man in sickle cell crisis is a patient on your unit. During report, one of the nurses from the previous shift mentions that she withheld the regularly scheduled IV opioid pain medication during the night because she had taken care of this patient a year ago and believes that he is a "drug seeker."
4. What can you do to prevent an incident like this one from happening again?
Speak to the nurse manager and the patient's health care provider about the incident. It is likely that this nurse does not understand the pathophysiology of sickle cell crisis or its management. There may be other nurses who have similar views. Arrange for a sickle cell disease and pain specialist to hold mandatory inservices on this topic for the entire staff on this unit.
4. A client is on intravenous heparin to treat a pulmonary embolism. The client's most recent partial thromboplastin time (PTT) was 25 seconds. What order should the nurse anticipate? a. Decrease the heparin rate.
b. Increase the heparin rate.
c. No change to the heparin rate.
d. Stop heparin; start warfarin (Coumadin).
ANS: B
For clients on heparin, a PTT of 1.5 to 2.5 times the normal value is needed to demonstrate the heparin is working. A normal PTT is 25 to 35 seconds, so this client's PTT value is too low. The heparin rate needs to be increased. Warfarin is not indicated in this situation.
3. After teaching a client with early polycystic kidney disease (PKD) about nutritional therapy, the nurse assesses the client's understanding. Which statement made by the client indicates a correct understanding of the teaching?
a. "I will take a laxative every night before going to bed."
b. "I must increase my intake of dietary fiber and fluids." c. "I shall only use salt when I am cooking my own food." d. "I'll eat white bread to minimize gastrointestinal gas."
ANS: B
Clients with PKD often have constipation, which can be managed with increased fiber, exercise, and drinking plenty of water. Laxatives should be used cautiously. Clients with PKD should be on a restricted salt diet, which includes not cooking with salt. White bread has a low fiber count and would not be included in a high-fiber diet.
4. While assessing a client on a cardiac unit, a nurse identifies the presence of an S3 gallop. Which action should the nurse take next?
a. Assess for symptoms of left-sided heart failure.
b. Document this as a normal finding.
c. Call the health care provider immediately.
d. Transfer the client to the intensive care unit.
ANS: A
The presence of an S3 gallop is an early diastolic filling sound indicative of increasing left ventricular pressure and left ventricular failure. The other actions are not warranted.
DIF: Remembering/Knowledge REF: 683
KEY: Heart failure| assessment/diagnostic examination
MSC: IntegratedProcess:NursingProcess:Assessment
NOT: Client Needs Category: Physiological Integrity: Reduction of Risk Potential
Ch. 61 Question 2 of 16
Which negative feedback response is responsible for preventing hypoglycemia during sleep in nondiabetic clients?
a. Alpha cells of the pancreas
b. Beta cells of the pancreas
c. Glucagon release
d. Insulin release
c
Glucagon release
Glucagon is the hormone that binds to receptors on liver cells. This causes the liver cells to convert glycogen to glucose, which keeps blood sugar levels normal during sleep. Alpha cells are responsible for synthesizing and secreting the hormone glucagon. Beta cells are responsible for synthesizing and secreting the hormone insulin. Insulin is the hormone responsible for lowering blood glucose. Insulin improves glucose uptake by the cell.
3. A nurse reviews the urinalysis results of a client and notes a urine osmolality of 1200 mOsm/L. Which action should the nurse take?
a. Contact the provider and recommend a low-sodium diet.
b. Prepare to administer an intravenous diuretic.
c. Obtain a suction device and implement seizure precautions. d. Encourage the client to drink more fluids.
ANS: D
Normal urine osmolality ranges from 300 to 900 mOsm/L. This client's urine is more concentrated, indicating dehydration. The nurse should encourage the client to drink more water. Dehydration can be associated with elevated serum sodium levels. Although a low-sodium diet may be appropriate for this client, this diet change will not have a significant impact on urine osmolality. A diuretic would increase urine output and decrease urine osmolality further. Low serum sodium levels, not elevated serum levels, place the client at risk for seizure activity. These options would further contribute to the client's dehydration or elevate the osmolality.
5. The nurse has taught the patient about dietary modifications for his vitamin B12 deficiency anemia. Which statement by the patient indicates that additional teaching is needed?
a. "Dairy products are a good source of vitamin B12."
b. "Dried beans taste okay if they are prepared correctly."
c. "Leafy green vegetables interfere with my therapy."
d. "I like nuts and I will gladly eat them."
c
4. A client is in the family practice clinic reporting a severe "cold" that started 4 days ago. On examination, the nurse notes the client also has a severe headache and muscle aches. What action by the nurse is best?
a. Educate the client on oseltamivir (Tamiflu).
b. Facilitate admission to the hospital.
c. Instruct the client to have a flu vaccine.
d. Teach the client to sneeze in the upper sleeve.
ANS: D
Sneezing and coughing into one's sleeve helps prevent the spread of upper respiratory infections. The client does have manifestations of the flu (influenza), but it is too late to start antiviral medications; to be effective, they must be started within 24 to 48 hours of symptom onset. The client does not need hospital admission. The client should be instructed to have a flu vaccination, but now that he or she has the flu, vaccination will have to wait until next year.
3. In which circumstance is the regulatory role of aldosterone most important in order for the person to maintain homeostasis?
a. Person is having pain related to a kidney stone.
b. Person has been hiking in the desert for several hours.
c. Person experiences stress incontinence when coughing.
d. Person experiences a burning sensation during urination.
b
Which of these clients should the charge nurse assign to the LPN/LVN working on the medical-surgical unit?
A) Client with group A beta-hemolytic streptococcal pharyngitis who has stridor
B) Client with pulmonary tuberculosis who is receiving multiple medications
C) Client with sinusitis who has just arrived after having endoscopic sinus surgery
D) Client with tonsillitis who has a thick-sounding voice and difficulty swallowing
(Chp. 31; elsevier resources)
B) Client with pulmonary tuberculosis who is receiving multiple medications
(Chp. 31; elsevier resources)
The nurse knows that erythropoietin is a growth factor that is required for stem cells specialization. Which sign/symptom would the nurse observe if erythropoietin is lacking or not performing its role?
a. Elevated body temperature
b. Bruising and ecchymosis
c. Swelling of lymph nodes
d. Easily fatigued
d
4. A nurse assesses an older adult client who has multiple chronic diseases. The clients heart rate is 48 beats/min. Which action should the nurse take first?
a. Document the finding in the chart.
b. Initiate external pacing.
c. Assess the clients medications.
d. Administer 1 mg of atropine.
C
Pacemaker cells in the conduction system decrease in number as a person ages, resulting in bradycardia. The nurse should check the medication reconciliation for medications that might cause such a drop in heart rate, then should inform the health care provider. Documentation is important, but it is not the priority action. The heart rate is not low enough for atropine or an external pacemaker to be needed.
2. Which assessment findings indicate hyperthyroidism? (Select all that apply.)
a. Weight loss with increased appetite
b. Constipation
c. Increased heart rate
d. Insomnia
e. Decreased libido
f. Heat intolerance
a, c, d, f
2. A nurse assesses a client who is recovering from a total thyroidectomy and notes the development of stridor. Which action should the nurse take first?
a. Reassure the client that the voice change is temporary.
b. Document the finding and assess the client hourly.
c. Place the client in high-Fowler's position and apply oxygen.
d. Contact the provider and prepare for intubation.
D. Stridor on exhalation is a hallmark of respiratory distress, usually caused by obstruction resulting from edema. One emergency measure is to remove the surgical clips to relieve the pressure. This might be a physician function. The nurse should prepare to assist with emergency intubation or tracheostomy while notifying the provider or the Rapid Response Team. Stridor is an emergency situation; therefore, reassuring the client, documenting, and reassessing in an hour do not address the urgency of the situation. Oxygen should be applied, but this action will not keep the airway open.
do online practice questions
do online practice questions
A hospitalized client has a platelet count of 58,000/mm3. What action by the nurse is best?
a.
Encourage high-protein foods.
b.
Institute neutropenic precautions.
c.
Limit visitors to healthy adults.
d.
Place the client on safety precautions.
d.
Place the client on safety precautions.
ANS: D
With a platelet count between 40,000 and 80,000/mm3, clients are at risk of prolonged bleeding even after minor trauma. The nurse should place the client on safety precautions. High-protein foods, while healthy, are not the priority. Neutropenic precautions are not needed as the client's white blood cell count is not low. Limiting visitors would also be more likely related to a low white blood cell count.
The nurse is caring for a client with severe acute respiratory syndrome. What is the most important precaution the nurse should take when preparing to suction this client?
A) Keeping the head of the bed elevated 30 to 45 degrees
B) Performing oral care after suctioning the oropharynx
C) Washing hands and donning gloves prior to the procedure
D) Wearing a disposable particulate mask respirator and protective eyewear
(Chp. 31; elsevier resources)
D) Wearing a disposable particulate mask respirator and protective eyewear
(Chp. 31; elsevier resources)
3. Which patient has the highest risk for developing a complicated UTI?
a. 26-year-old woman who is sexually active, but not currently pregnant
b. 22-year-old man who has a neurogenic bladder due to spinal cord injury
c. 35-year-old woman who had three full-term pregnancies and a miscarriage
d. 53-year-old woman who is having some menstrual irregularities
b
3. What are common causes of prerenal kidney injury? (Select all that apply.)
a. Urethral cancer
b. Hypovolemic shock
c. Enlarged prostate gland
d. Sepsis
e. Severe burns
b, d, e
5. The charge nurse on a medical unit is preparing to admit several "clients" who have possible pandemic flu during a preparedness drill. What action by the nurse is best?
a. Admit the "clients" on Contact Precautions.
b. Cohort the "clients" in the same area of the unit.
c. Do not allow pregnant caregivers to care for these "clients." d. Place the "clients" on enhanced Droplet Precautions.
ANS: B
Preventing the spread of pandemic flu is equally important as caring for the clients who have it. Clients can be cohorted together in the same set of rooms on one part of the unit to use distancing to help prevent the spread of the disease. The other actions are not appropriate.
A client has Crohn's disease. What type of anemia is this client most at risk for developing?
a. Folic acid deficiency
b. Fanconi's anemia
c. Hemolytic anemia
d. Vitamin B12 anemia
a. Folic acid deficiency
To validate that a client has had a myocardial infarction (MI), the nurse assesses for positive findings on which tests?
a. Creatine kinase-MB fraction (CK-MB) and alkaline phosphatase
b. Homocysteine and C-reactive protein
c. Total cholesterol, low-density lipoprotein cholesterol, and high-density lipoprotein cholesterol
d. CK-MB and troponin
d. CK-MB and troponin
CK-MB and troponin are the cardiac markers used to determine whether MI has occurred. Alkaline phosphatase is often elevated in liver disease. Homocysteine and C-reactive protein are markers of inflammation, which may represent risk for MI, but they are not diagnostic for MI. Elevated cholesterol levels are risks for MI, but they do not validate that an MI has occurred.
A nurse cares for a client with a deficiency of aldosterone. Which assessment finding should the nurse correlate with this deficiency?
a. Increased urine output
b. Vasoconstriction
c. Blood glucose of 98 mg/dL
d. Serum sodium of 144 mEq/L
ANS: A
Aldosterone, the major mineralocorticoid, maintains extracellular fluid volume. It promotes sodium and water reabsorption and potassium excretion in the kidney tubules. A client with an aldosterone deficiency will have increased urine output. Vasoconstriction is not related. These sodium and glucose levels are normal; in aldosterone deficiency, the client would have hyponatremia and hyperkalemia.
To validate that a client has had a myocardial infarction (MI), the nurse assesses for positive findings on which tests?
Creatine kinase-MB fraction (CK-MB) and alkaline phosphatase
Homocysteine and C-reactive protein
Total cholesterol, low-density lipoprotein cholesterol, and high-density lipoprotein cholesterol
CK-MB and troponin
CK-MB and troponin
CK-MB and troponin are the cardiac markers used to determine whether MI has occurred. Alkaline phosphatase is often elevated in liver disease. Homocysteine and C-reactive protein are markers of inflammation, which may represent risk for MI, but they are not diagnostic for MI. Elevated cholesterol levels are risks for MI, but they do not validate that an MI has occurred.
p. 1264, Health Promotion and Maintenance
Which precaution or action is most important for the nurse to teach the client who is to collect a 24-hour urine specimen for endocrine testing?
A. Eat a normal diet during the collection period.
B. Wear gloves when you urinate to prevent contamination of the specimen.
C. Urinate at the end of 24 hours and add that sample to the collection container.
D. Avoid walking, running, dancing, or any vigorous exercise during the collection period.
Answer: C
Rationale: When a 24-hour urine specimen is started, the specimen should reflect all the urine produced during the specified time. The very first voiding is discarded because the urine has spent some time in the bladder and will not reflect what is happening during the actual 24 hours of the collection. The time of this discard is the beginning of the 24-hour collection period. The test requires that all urine voided after the start time be collected, including the specimen collected by emptying the bladder at end of the 24 hours, which marks the end of the test.
p. 806, Health Promotion and Maintenance
Which intervention is most important for the nurse to teach the client with polycythemia vera to prevent injury as a result of the increased bleeding tendency?
A. Use a soft-bristled toothbrush.
B. Drink at least 3 liters of liquids per day.
C. Wear gloves and socks outdoors in cool weather.
D. Exercise slowly and only on the advice of your physician.
Answer: A
Rationale: The other interventions focus on preventing venous stasis, clot formation, and myocardial infarction. Using a soft-bristled toothbrush minimizes trauma to the gums and prevents bleeding.
5. A client is hospitalized with a second episode of pulmonary embolism (PE). Recent genetic testing reveals the client has an alteration in the gene CYP2C19. What action by the nurse is best?
a. Instruct the client to eliminate all vitamin K from the diet.
b. Prepare preoperative teaching for an inferior vena cava (IVC) filter. c. Refer the client to a chronic illness support group.
d. Teach the client to use a soft-bristled toothbrush.
ANS: B
Often clients are discharged from the hospital on warfarin (Coumadin) after a PE. However, clients with a variation in the CYP2C19 gene do not metabolize warfarin well and have higher blood levels and more side effects. This client is a poor candidate for warfarin therapy, and the prescriber will most likely order an IVC filter device to be implanted. The nurse should prepare to do preoperative teaching on this procedure. It would be impossible to eliminate all vitamin K from the diet. A chronic illness support group may be needed, but this is not the best intervention as it is not as specific to the client as the IVC filter. A soft-bristled toothbrush is a safety measure for clients on anticoagulation therapy.
3. Which description of the autosomal-dominant form of PKD is correct?
a. 25% of patients with this from of PKD develop acute kidney failure by age 30.
b. The dominant from is responsive to newer gene therapy treatments.
c. 50% of people with this form of PKD develop kidney disease by age 50.
d. Most people with this form of PKD die in young adulthood.
c
4. A nursing student is caring for a client who had a myocardial infarction. The student is confused because the client states nothing is wrong and yet listens attentively while the student provides education on lifestyle changes and healthy menu choices. What response by the faculty member is best?
a. "Continue to educate the client on possible healthy changes."
b. "Emphasize complications that can occur with noncompliance."
c. "Tell the client that denial is normal and will soon go away."
d. "You need to make sure the client understands this illness."
ANS: A
Clients are often in denial after a coronary event. The client who seems to be in denial but is compliant with treatment may be using a healthy form of coping that allows time to process the event and start to use problem-focused coping. The student should not discourage this type of denial and coping, but rather continue providing education in a positive manner. Emphasizing complications may make the client defensive and more anxious. Telling the client that denial is normal is placing too much attention on the process. Forcing the client to verbalize understanding of the illness is also potentially threatening to the client.
DIF: Understanding/Comprehension REF: 769
KEY: Coronary artery disease| psychosocial response| coping| therapeutic communication MSC: Integrated Process: Communication and Documentation
NOT: Client Needs Category: Psychosocial Integrity
An RN and LPN/LVN are caring for a group of clients on the medical-surgical unit. Which client will be the best to assign to the LPN/LVN?
A. Client with Graves' disease who needs discharge teaching after a total thyroidectomy
B. Client with hyperparathyroidism who is just being admitted for a parathyroidectomy
C. Client with infiltrative ophthalmopathy who needs administration of high-dose prednisone (Deltasone)
D. Newly diagnosed client with hypothyroidism who needs education about the use of thyroid supplements
C.
Medication administration for the client with infiltrative ophthalmopathy is within the scope of practice of the LPN/LVN. Discharge teaching is a complex task that cannot be delegated to the LPN/LVN. A client being admitted for a parathyroidectomy needs preoperative teaching, which must be provided by the RN. A client who has a new diagnosis will have questions about the disease and prescribed medications; teaching is a complex task that is appropriate for the RN.
4. A client is admitted with acute kidney injury (AKI) and a urine output of 2000 mL/day. What is the major concern of the nurse regarding this client's care?
a. Edema and pain
b. Electrolyte and fluid imbalance
c. Cardiac and respiratory status d. Mental health status
ANS: B
This client may have an inflammatory cause of AKI with proteins entering the glomerulus and holding the fluid in the filtrate, causing polyuria. Electrolyte loss and fluid balance is essential. Edema and pain are not usually a problem with fluid loss. There could be changes in the client's cardiac, respiratory, and mental health status if the electrolyte imbalance is not treated.
6. Which patient is most likely to have severe manifestations of sickle cell disease even when triggering conditions are mild?
a. Both parents have hemoglobin S gene alleles
b. Mother has hemoglobin S gene alleles and father has hemoglobin A gene alleles
c. Mother has sickle cell trait and father has hemoglobin A gene alleles
d. Both parents have hemoglobin A gene alleles
a
5. A nurse cares for a client who has developed esophagitis after undergoing radiation therapy for lung cancer. Which diet selection should the nurse provide for this client?
a. Spaghetti with meat sauce, ice cream
b. Chicken soup, grilled cheese sandwich
c. Omelet, soft whole wheat bread
d. Pasta salad, custard, orange juice
ANS: C
Side effects of radiation therapy may include inflammation of the esophagus. Clients should be taught that bland, soft, high-calorie foods are best, along with liquid nutritional supplements. Tomato sauce may prove too spicy for a client with esophagitis. A grilled cheese sandwich is too difficult to swallow with this condition, and orange juice and other foods with citric acid are too caustic.
DIF: Applying/Application REF: 576 KEY: Cancer| nutrition MSC: IntegratedProcess:NursingProcess:Assessment
NOT: Client Needs Category: Physiological Integrity: Basic Care and Comfort
4. After teaching a client with bacterial cystitis who is prescribed phenazopyridine (Pyridium), the nurse assesses the client's understanding. Which statement made by the client indicates a correct understanding of the teaching?
a. "I will not take this drug with food or milk."
b. "If I think I am pregnant, I will stop the drug."
c. "An orange color in my urine should not alarm me." d. "I will drink two glasses of cranberry juice daily."
ANS: C
Phenazopyridine discolors urine, most commonly to a deep reddish orange. Many clients think they have blood in their urine when they see this. In addition, the urine can permanently stain clothing. Phenazopyridine is safe to take if the client is pregnant. There are no dietary restrictions or needs while taking this medication.
65. The client has had a total laryngectomy. Which
referral is specific for this surgery?
1. CanSurmount.
2. Dialogue.
3. Lost Chord Club.
4. SmokEnders.
3. The Lost Chord Club is an American
Cancer Society-sponsored group of
survivors of larynx cancer. These clients
are able to discuss the feelings and needs
of clients who have had laryngectomies
because they have all had this particular
surgery.
TEST-TAKING HINT: The larynx is commonly
referred to as the "vocal chords." If the test
taker were not aware of the various support
groups, option "3" has "lost" and "chords" in
it and is the best choice.
Based on knowledge of albumin's role in maintaining osmotic pressure of the blood, which sign/symptom would the nurse observe for if the patient has low albumin levels?
a. Fever
b. Edema
c. Dizziness
d. Pain
b
5. A client undergoing hemodynamic monitoring after a myocardial infarction has a right atrial pressure of 0.5 mm Hg. What action by the nurse is most appropriate?
a. Level the transducer at the phlebostatic axis.
b. Lay the client in the supine position.
c. Prepare to administer diuretics.
d. Prepare to administer a fluid bolus.
ANS: D
Normal right atrial pressures are from 1 to 8 mm Hg. Lower pressures usually indicate hypovolemia, so the nurse should prepare to administer a fluid bolus. The transducer should remain leveled at the phlebostatic axis. Positioning may or may not influence readings. Diuretics would be contraindicated.
DIF: Applying/Application REF: 770
KEY: Coronary artery disease| hemodynamic monitoring| fluid and electrolyte imbalance MSC: Integrated Process: Nursing Process: Implementation
NOT: Client Needs Category: Physiological Integrity: Physiological Adaptation
A nurse cares for a client with excessive production of thyrocalcitonin (calcitonin). For which electrolyte imbalance should the nurse assess?
a. Potassium
b. Sodium
c. Calcium
d. Magnesium
ANS: C
Parafollicular cells produce thyrocalcitonin (calcitonin), which regulates serum calcium levels. Calcitonin has no impact on potassium, sodium, or magnesium balances.
Prompt pain management with myocardial infarction is essential for which reason?
The discomfort will increase client anxiety and reduce coping.
Pain relief improves oxygen supply and decreases oxygen demand.
Relief of pain indicates that the MI is resolving.
Pain medication should not be used until a definitive diagnosis has been established.
Pain relief improves oxygen supply and decreases oxygen demand.
The focus of pain relief is on reducing myocardial oxygen demand. Chest discomfort will increase anxiety, but it may not affect coping. Relief of pain is secondary to the use of opiates or indicates that the tissue infarction is complete. Although it used to be true that pain medication was not to be used for undiagnosed abdominal pain, this does not relate to MI.
A client is referred to a home health agency after a transsphenoidal hypophysectomy. Which action does the RN case manager delegate to the home health aide who will see the client daily?
A. Document symptoms of incisional infection or meningitis.
B. Give over-the-counter laxatives if the client is constipated.
C. Set up medications as prescribed for the day.
D. Test any nasal drainage for the presence of glucose.
D.
Cerebrospinal fluid (CSF) will test positive using a glucose "dipstick." Nasal drainage that is positive for glucose after a transsphenoidal hypophysectomy would indicate a CSF leak that would require immediate notification of the health care provider. Home health aides can be taught the correct technique to perform this procedure. Assessing for symptoms of infection and documenting them in the record, medication administration, and setting up medication are not within the scope of practice of the home health aide.
A client is having a bone marrow biopsy today. What action by the nurse takes priority?
a.
Administer pain medication first.
b.
Ensure valid consent is on the chart.
c.
Have the client shower in the morning.
d.
Premedicate the client with sedatives.
b.
Ensure valid consent is on the chart.
ANS: B
A bone marrow biopsy is an invasive procedure that requires informed consent. Pain medication and sedation are important components of care for this client but do not take priority. The client may or may not need or be able to shower.
5. A nurse assesses clients on the medical-surgical unit. Which client is at greatest risk for development of obstructive sleep apnea?
a. A 26-year-old woman who is 8 months pregnant
b. A 42-year-old man with gastroesophageal reflux disease
c. A 55-year-old woman who is 50 pounds overweight
d. A 73-year-old man with type 2 diabetes mellitus
ANS: C
The client at highest risk would be the one who is extremely overweight. None of the other clients have risk factors for sleep apnea.
DIF: Applying/Application REF: 535
KEY: Respiratory distress/failure
MSC: IntegratedProcess:NursingProcess:Assessment
NOT: Client Needs Category: Health Promotion and Maintenance
4. After teaching a client with bacterial cystitis who is prescribed phenazopyridine (Pyridium), the nurse assesses the clients understanding. Which statement made by the client indicates a correct understanding of the teaching?
a. I will not take this drug with food or milk.
b. If I think I am pregnant, I will stop the drug.
c. An orange color in my urine should not alarm me.
d. I will drink two glasses of cranberry juice daily.
C
Phenazopyridine discolors urine, most commonly to a deep reddish orange. Many clients think they have blood in their urine when they see this. In addition, the urine can permanently stain clothing. Phenazopyridine is safe to take if the client is pregnant. There are no dietary restrictions or needs while taking this medication.
6. After teaching a client who is prescribed "voice rest" therapy for vocal cord polyps, a nurse assesses the client's understanding. Which statement indicates the client needs further teaching?
a. "I will stay away from smokers to minimize inhalation of secondhand smoke."
b. "When I speak, I will whisper rather than use a normal tone of voice."
c. "For the next several weeks, I will not lift more than 10 pounds."
d. "I will drink at least three quarts of water each day to stay hydrated."
ANS: B
Treatment for vocal cord polyps includes no speaking, no lifting, and no smoking. The client has to be educated not to even whisper when resting the voice. It is also appropriate for the client to stay out of rooms where people are smoking, to stay hydrated, and to use stool softeners.
DIF: Applying/Application REF: 536
KEY: Cancer| patient education MSC: Integrated Process: Teaching/Learning NOT: Client Needs Category: Physiological Integrity: Reduction of Risk Potential
The nurse is interviewing a patient who reports dizziness and lightheadedness, and bleeding gums every time she brushes her teeth. Which questions does the nurse ask the patient in order to focus in on the problem? (Select all that apply.)
a. "How often do you take aspirin or any other
nonsteroidal antiinflammatory drug?"
b. "Do you have swollen glands or a sore throat?"
c. "How much meat do you eat in a week?"
d. "Are you having trouble swallowing?"
e. "Does your heart ever seem to pound?"
a, c, e
Prompt pain management with myocardial infarction is essential for which reason?
a. The discomfort will increase client anxiety and reduce coping.
b. Pain relief improves oxygen supply and decreases oxygen demand.
c. Relief of pain indicates that the MI is resolving.
d. Pain medication should not be used until a definitive diagnosis has been established.
b. Pain relief improves oxygen supply and decreases oxygen demand.
The focus of pain relief is on reducing myocardial oxygen demand. Chest discomfort will increase anxiety, but it may not affect coping. Relief of pain is secondary to the use of opiates or indicates that the tissue infarction is complete. Although it used to be true that pain medication was not to be used for undiagnosed abdominal pain, this does not relate to MI.
3. A malfunctioning anterior pituitary gland can result in which disorder? (Select all that apply.)
a. Pituitary hypofunction
b. Pituitary hyperfunction
c. DI
d. Hypothyroidism
e. Osteoporosis
a, b, d, e
Pituitary hypofunction
Pituitary hyperfunction
Hypothyroidism
Osteoporosis
4. A nurse cares for a middle-aged female client with diabetes mellitus who is being treated for the third episode of acute pyelonephritis in the past year. The client asks, "What can I do to help prevent these infections?" How should the nurse respond?
a. "Test your urine daily for the presence of ketone bodies and proteins."
b. "Use tampons rather than sanitary napkins during your menstrual period."
c. "Drink more water and empty your bladder more frequently during the day."
d. "Keep your hemoglobin A1c under 9% by keeping your blood sugar controlled."
ANS: C
Clients with long-standing diabetes mellitus are at risk for pyelonephritis for many reasons. Chronically elevated blood glucose levels spill glucose into the urine, changing the pH and providing a favorable climate for bacterial growth. The neuropathy associated with diabetes reduces bladder tone and reduces the client's sensation of bladder fullness. Thus, even with large amounts of urine, the client voids less frequently, allowing stasis and overgrowth of microorganisms. Increasing fluid intake (specifically water) and voiding frequently prevent stasis and bacterial overgrowth. Testing urine and using tampons will not help prevent pyelonephritis. A hemoglobin A1c of 9% is too high.
4. Which hormone is released from the posterior pituitary and makes the distal convoluted tubule and the collecting duct permeable to water to maximize reabsorption and produce concentrated urine?
a. Aldosterone
b. Vasopressin
c. ZBradykinins
d. Natriuretic
b
66. The nurse and an unlicensed assistive personnel
(UAP) are caring for a group of clients on a
surgical floor. Which information provided by
the UAP requires immediate intervention by
the nurse?
1. There is a small, continuous amount of
bright-red drainage coming out from under
the dressing of the client who had a radical
neck dissection.
2. The client who has had a right upper
lobectomy is complaining that the patientcontrolled
analgesia (PCA) pump is not
providing any relief.
3. The client diagnosed with cancer of the
lung is complaining of being tired and short
of breath.
4. The client admitted with chronic obstructive
pulmonary disease is making a whistling
sound with every breath.
1. The most serious complication resulting
from a radical neck dissection is rupture
of the carotid artery. Continuous brightred
drainage indicates bleeding, and this
client should be assessed immediately.
TEST-TAKING HINT: If the test taker is not
sure of the answer and airway compromise is
not one of the answer options, then an option
dealing with bleeding is the best choice.
A nurse is caring for four clients. After reviewing today's laboratory results, which client should the nurse see first?
a.
Client with an international normalized ratio of 2.8
b.
Client with a platelet count of 128,000/mm3
c.
Client with a prothrombin time (PT) of 28 seconds
d.
Client with a red blood cell count of 5.1 million/μL
c.
Client with a prothrombin time (PT) of 28 seconds
ANS: C
A normal PT is 11 to 12.5 seconds. This client is at high risk of bleeding. The other values are within normal limits.
do study guide practice questions
do study guide practice questions
3. Which mechanism is used to transport the substance produced by the endocrine glands to their target tissue?
a. Lymph system
b. Bloodstream
c. Direct seeding
d. Gastrointestinal system
b
Bloodstream
5. A nurse evaluates prescriptions for a client with chronic atrial fibrillation. Which medication should the nurse expect to find on this client's medication administration record to prevent a common complication of this condition?
a. Sotalol (Betapace)
b. Warfarin (Coumadin)
c. Atropine (Sal-Tropine)
d. Lidocaine (Xylocaine)
ANS: B
Atrial fibrillation puts clients at risk for developing emboli. Clients at risk for emboli are treated with anticoagulants, such as heparin, enoxaparin, or warfarin. Sotalol, atropine, and lidocaine are not appropriate for this complication.
DIF: Applying/Application REF: 667
KEY: Cardiac electrical conduction| medication
MSC: Integrated Process: Nursing Process: Analysis
NOT: Client Needs Category: Physiological Integrity: Pharmacological and Parenteral Therapies
A nurse cares for a male client with hypopituitarism who is prescribed testosterone hormone replacement therapy. The client asks, How long will I need to take this medication? How should the nurse respond?
a. When your blood levels of testosterone are normal, the therapy is no longer needed.
b. When your beard thickens and your voice deepens, the dose is decreased, but treatment will continue forever.
c. When your sperm count is high enough to demonstrate fertility, you will no longer need this therapy.
d. With age, testosterone levels naturally decrease, so the medication can be stopped when you are 50 years old
B (Testosterone therapy is initiated with high-dose testosterone derivatives and is continued until virilization is achieved. The dose is then decreased, but therapy continues throughout life. Therapy will continue throughout life; therefore, it will not be discontinued when blood levels are normal, at the age of 50 years, or when sperm counts are high.)
6. The nurse is caring for a client with lung cancer who states, "I don't want any pain medication because I am afraid I'll become addicted." How should the nurse respond? a. "I will ask the provider to change your medication to a drug that is less potent."
b. "Would you like me to use music therapy to distract you from your pain?"
c. "It is unlikely you will become addicted when taking medicine for pain."
d. "Would you like me to give you acetaminophen (Tylenol) instead?"
ANS: C
Clients should be encouraged to take their pain medications; addiction usually is not an issue with a client in pain. The nurse would not request that the pain medication be changed unless it was not effective. Other methods to decrease pain can be used, in addition to pain medication.
DIF: Applying/Application REF: 576
KEY: Cancer| pain| pharmacologic pain management
MSC: Integrated Process: Nursing Process: Implementation
NOT: Client Needs Category: Physiological Integrity: Pharmacological and Parenteral Therapies
7. After teaching a client who is prescribed salmeterol (Serevent), the nurse assesses the client's understanding. Which statement by the client indicates a need for additional teaching?
a. "I will be certain to shake the inhaler well before I use it."
b. "It may take a while before I notice a change in my asthma."
c. "I will use the drug when I have an asthma attack."
d. "I will be careful not to let the drug escape out of my nose and mouth."
ANS: C
Salmeterol is designed to prevent an asthma attack; it does not relieve or reverse symptoms. Salmeterol has a slow onset of action; therefore, it should not be used as a rescue drug. The drug must be shaken well because it has a tendency to separate easily. Poor technique on the client's part allows the drug to escape through the nose and mouth.
DIF: Applying/Application REF: 554
KEY: Medication| patient education MSC: Integrated Process: Teaching/Learning NOT: Client Needs Category: Health Promotion and Maintenance
7. The nurse is caring for a patient in sickle cell crisis. What are priority interventions for this patient? (Select all that apply.)
a. Managing pain
b. Managing nutrition
c. Ensuring hydration
d. Administering platelets
e. Assessing oxygen saturation
a, c, e
4. A patient can develop intrarenal kidney injury from which causes? (Select all that apply.)
a. Vasculitis
b. Pyelonephritis
c. Strenuous exercise
d. Exposure to nephrotoxins
e. Bladder cancer
a, b, d
4. The nurse is interviewing a patient with suspected PKD. What questions does the nurse ask the patient? (Select all that apply.)
a. "Is there any family history of PKD or kidney disease?"
b. "Do you have a history of sexually transmitted disease?"
c. "Have you had any constipation or abdominal discomfort?"
d. "Have you noticed a change in urine color or frequency?"
e. "Have you had any problems with headaches?"
f. " Is there a family history of sudden death from a myocardial infarction?"
a, c, d, e
A nurse in a hematology clinic is working with four clients who have polycythemia vera. Which client should the nurse see first?
a. Client with a blood pressure of 180/98 mm Hg
b. Client who reports shortness of breath
c. Client who reports calf tenderness and swelling
d. Client with a swollen and painful left great toe
b. Client who reports shortness of breath
5. A nurse cares for a client with right-sided heart failure. The client asks, "Why do I need to weigh myself every day?" How should the nurse respond?
a. "Weight is the best indication that you are gaining or losing fluid."
b. "Daily weights will help us make sure that you're eating properly."
c. "The hospital requires that all inpatients be weighed daily."
d. "You need to lose weight to decrease the incidence of heart failure."
ANS: A
Daily weights are needed to document fluid retention or fluid loss. One liter of fluid equals 2.2 pounds. The other responses do not address the importance of monitoring fluid retention or loss.
DIF: Remembering/Knowledge REF: 683
KEY: Heart failure| patient education MSC: Integrated Process: Teaching/Learning NOT: Client Needs Category: Physiological Integrity: Physiological Adaptation
The nurse assesses a patient in the emergency department (ED) and finds the following: constipation, fatigue with increased sleeping time, impaired memory, facial puffiness, and weight gain. Which deficiency does the nurse recognize?
a. Hyperthyroidism
b. Hypothyroidism
c. Hyperparathyroidism
d. Hypoparathyroidism
b
6. A nurse is caring for four clients on intravenous heparin therapy. Which laboratory value possibly indicates that a serious side effect has occurred?
a. Hemoglobin: 14.2 g/dL
b. Platelet count: 82,000/L 3
c. Red blood cell count: 4.8/mm 3 d. White blood cell count: 8.7/mm
ANS: B
This platelet count is low and could indicate heparin-induced thrombocytopenia. The other values are normal for either gender.
5. An emergency room nurse obtains the health history of a client. Which statement by the client should alert the nurse to the occurrence of heart failure?
a. I get short of breath when I climb stairs.
b. I see halos floating around my head.
c. I have trouble remembering things.
d. I have lost weight over the past month.
A
Dyspnea on exertion is an early manifestation of heart failure and is associated with an activity such as stair climbing. The other findings are not specific to early occurrence of heart failure.
An older client with an elevated serum calcium level is receiving IV furosemide (Lasix) and an infusion of normal saline at 150 mL/hr. Which nursing action can the RN delegate to unlicensed assistive personnel (UAP)?
A. Ask the client about any numbness or tingling.
B. Check for bone deformities in the client's back.
C. Measure the client's intake and output hourly.
D. Monitor the client for shortness of breath.
C.
Measuring intake and output is a commonly delegated nursing action that is within the UAP scope of practice. Numbness and tingling is part of the client assessment that needs to be completed by a licensed nurse. Bony deformities can be due to pathologic fractures; physical assessment is a complex task that cannot be delegated. An older client receiving an IV at 150 mL/hr is at risk for congestive heart failure; careful monitoring for shortness of breath is the responsibility of the RN.
p. 809, Physiological Integrity
The blood of a client who has chronic myelogenous leukemia shows a high percentage of blast cells and promyelocytes. What is the nurse's correct interpretation of this test result?
A. The client's risk for infection is decreasing.
B. The disease has become more aggressive.
C. The drug therapy for the disease is effective.
D. The type of leukemia is now lymphocytic rather than myelogenous.
Answer: B
Rationale: The leukemia is progressing and the drug therapy is no longer effective. CML has three phases: The chronic phase is often a slowly progressing (indolent) course during which the patient may have mild symptoms and respond to standard treatments. The bone marrow usually shows less than 10% blast cells at this time. The accelerated phase features spleen enlargement and progressive manifestation, such as intermittent fevers, night sweats, and unexplained weight loss. The patient usually does not respond to standard treatment, and the bone marrow may contain 10% to 30% blast cells and promyelocytes. The blast phase indicates transformation to a very aggressive acute leukemia. The bone marrow contains more than 30% blast cells. The promyelocytes and blast cells commonly spread to other tissues and organs. The leukemia becomes more similar to acute leukemia than chronic leukemia but does not change from myelogenous to lymphocytic. With so many blast cells that are immature and do not function properly, the client is now at greatly increased risk for infection.
6. A client admitted for pneumonia has been tachypneic for several days. When the nurse starts an IV to give fluids, the client questions this action, saying "I have been drinking tons of water. How am I dehydrated?" What response by the nurse is best?
a. "Breathing so quickly can be dehydrating."
b. "Everyone with pneumonia is dehydrated."
c. "This is really just to administer your antibiotics." d. "Why do you think you are so dehydrated?"
ANS: A
Tachypnea and mouth breathing, both seen in pneumonia, increase insensible water loss and can lead to a degree of dehydration. The other options do not give the client useful information.
4. The nurse is working in a long-term care facility. Which circumstance is cause for greatest concern, because the facility has a large number of residents who are developing UTIs?
a. Residents are not drinking enough fluids with meals.
b. Unlicensed personnel are not assisting with toileting in a timely fashion.
c. A large percentage of residents have indwelling urinary catheters.
d. Many residents have severe dementia and functional incontinence.
c
A client with pneumonia caused by aspiration after alcohol intoxication has just been admitted. The client is febrile and agitated. Which health care provider order should the nurse implement first?
A) Administer levofloxacin (Levaquin) 500 mg IV.
B) Draw aerobic and anaerobic blood cultures.
C) Give lorazepam (Ativan) as needed for agitation.
D) Refer to social worker for alcohol counseling.
(Chp. 31; elsevier resources)
B) Draw aerobic and anaerobic blood cultures.
(Chp. 31; elsevier resources)
A nursing student is caring for a client with leukemia. The student asks why the client is still at risk for infection when the client's white blood cell count (WBC) is high. What response by the registered nurse is best?
a. "If the WBCs are high, there already is an infection present."
b. "The client is in a blast crisis and has too many WBCs."
c. "There must be a mistake; the WBCs should be very low."
d. "Those WBCs are abnormal and don't provide protection."
d. "Those WBCs are abnormal and don't provide protection."
6. A nurse is teaching a client with heart failure who has been prescribed enalapril (Vasotec). Which statement should the nurse include in this client's teaching?
a. "Avoid using salt substitutes."
b. "Take your medication with food."
c. "Avoid using aspirin-containing products."
d. "Check your pulse daily."
ANS: A
Angiotensin-converting enzyme (ACE) inhibitors such as enalapril inhibit the excretion of potassium. Hyperkalemia can be a life-threatening side effect, and clients should be taught to limit potassium intake. Salt substitutes are composed of potassium chloride. ACE inhibitors do not need to be taken with food and have no impact on the client's pulse rate. Aspirin is often prescribed in conjunction with ACE inhibitors and is not contraindicated.
DIF: Applying/Application REF: 685
KEY: Heart failure| angiotensin-converting enzyme (ACE) inhibitor| medication| patient education MSC: IntegratedProcess:Teaching/Learning
NOT: Client Needs Category: Physiological Integrity: Pharmacological and Parenteral Therapies
After teaching a young adult client who is newly diagnosed with type 1 diabetes mellitus, the nurse assesses the client's understanding. Which statement made by the client indicates a correct understanding of the need for eye examinations?
a. "At my age, I should continue seeing the ophthalmologist as I usually do."
b. "I will see the eye doctor when I have a vision problem and yearly after age 40."
c. "My vision will change quickly. I should see the ophthalmologist twice a year."
d. "Diabetes can cause blindness, so I should see the ophthalmologist yearly."
ANS: D
Diabetic retinopathy is a leading cause of blindness in North America. All clients with diabetes, regardless of age, should be examined by an ophthalmologist (rather than an optometrist or optician) at diagnosis and at least yearly thereafter.
4. A nurse assesses a client with renal insufficiency and a low red blood cell count. The client asks, "Is my anemia related to the renal insufficiency?" How should the nurse respond?
a. "Red blood cells produce erythropoietin, which increases blood flow to the
kidneys."
b. "Your anemia and renal insufficiency are related to inadequate vitamin D and a
loss of bone density."
c. "Erythropoietin is usually released from the kidneys and stimulates red blood cell
production in the bone marrow."
d. "Kidney insufficiency inhibits active transportation of red blood cells throughout
the blood."
ANS: C
Erythropoietin is produced in the kidney and is released in response to decreased oxygen tension in the renal blood supply. Erythropoietin stimulates red blood cell production in the bone marrow. Anemia and renal insufficiency are not manifestations of vitamin D deficiency. The kidneys do not play a role in the transportation of red blood cells or any other cells in the blood.
5. After teaching a client who has stress incontinence, the nurse assesses the client's understanding. Which statement made by the client indicates a need for additional teaching? a. "I will limit my total intake of fluids."
b. "I must avoid drinking alcoholic beverages."
c. "I must avoid drinking caffeinated beverages."
d. "I shall try to lose about 10% of my body weight."
ANS: A
Limiting fluids concentrates urine and can irritate tissues, leading to increased incontinence. Many people try to manage incontinence by limiting fluids. Alcoholic and caffeinated beverages are bladder stimulants. Obesity increases intra-abdominal pressure, causing incontinence.
The medical-surgical unit has one negative-airflow room. Which of these four clients who have just arrived on the unit should the charge nurse admit to this room?
A) Client with bacterial pneumonia and a cough productive of green sputum
B) Client with neutropenia and pneumonia caused by Candida albicans
C) Client with possible pulmonary tuberculosis who currently has hemoptysis
D) Client with right empyema who has a chest tube and a fever of 103.2° F
(Chp. 31; elsevier resources)
C) Client with possible pulmonary tuberculosis who currently has hemoptysis
(Chp. 31; elsevier resources)
p. 813, Safety; Teamwork and Collaboration
The patient is a 44-year-old chemical plant foreman who developed acute myelogenous leukemia 6 months ago. His initial therapy was successful, and he is scheduled to have a stem cell transplant (bone marrow) with his identical twin brother as the donor. His brother lives in the same city and is a professor at a local university. The patient is very grateful that his brother will donate bone marrow and states that he is certain that he has no risk for infection during the procedure because his brother is his identical twin.
1. What type of class of stem cell transplant would this procedure be considered?
When the donor is an identical sibling, all major and minor tissue antigens are a perfect match, not just the six standard major ones. Therefore, this is considered a syngeneic transplant.
6. A nurse obtains the health history of a client who is newly admitted to the medical unit. Which statement by the client should alert the nurse to the presence of edema?
a. I wake up to go to the bathroom at night.
b. My shoes fit tighter by the end of the day.
c. I seem to be feeling more anxious lately.
d. I drink at least eight glasses of water a day.
B
Weight gain can result from fluid accumulation in the interstitial spaces. This is known as edema. The nurse should note whether the client feels that his or her shoes or rings are tight, and should observe, when present, an indentation around the leg where the socks end. The other answers do not describe edema.
Ch.62 p. 1268, Safe and Effective Care Environment
For which client does the nurse question the prescription of androgen replacement therapy?
A. 35-year-old man who has had a vasectomy
B. 48-year old man who takes prednisone for severe asthma
C. 62-year-old man who has a history of prostate cancer
D. 70-year-old man who has hypertension and type 2 diabetes
Answer: C
Rationale: Prostate cancer tends to increase its growth rate in the presence of any type of androgen. Thus, the man who has a history of prostate cancer should avoid exogenous androgen because it could enhance the growth if the previously treated cancer returns. None of the other conditions are contraindicated for androgen replacement therapy.
6. A client has hemodynamic monitoring after a myocardial infarction. What safety precaution does the nurse implement for this client?
a. Document pulmonary artery wedge pressure (PAWP) readings and assess their
trends.
b. Ensure the balloon does not remain wedged.
c. Keep the client on strict NPO status.
d. Maintain the client in a semi-Fowler's position.
ANS: B
If the balloon remains inflated, it can cause pulmonary infarction or rupture. The nurse should ensure the balloon remains deflated between PAWP readings. Documenting PAWP readings and assessing trends is an important nursing action related to hemodynamic monitoring, but is not specifically related to safety. The client does not have to be NPO while undergoing hemodynamic monitoring. Positioning may or may not affect readings.
DIF: Applying/Application REF: 770
KEY: Coronary artery disease| hemodynamic monitoring| equipment safety
MSC: Integrated Process: Nursing Process: Implementation
NOT: Client Needs Category: Safe and Effective Care Environment: Safety and Infection Control
Ch.61 Question 3 of 16
The nurse is teaching a client about maintaining a proper diet to prevent an endocrine disorder. Which food does the nurse suggest after the client indicates a dislike of fish?
a. Iodized salt for cooking
b. More red meat
c. More green vegetables
d. Salt substitute for cooking
a
Iodized salt for cooking
Dietary deficiencies in iodide-containing foods may be a cause of an endocrine disorder. For clients who do not eat saltwater fish on a regular basis, teach them to use iodized salt in food preparation. The client should eat a well-balanced diet that includes less animal fat. Eating vegetables contributes to a proper diet; however, this does not prevent an endocrine disorder. Using a salt substitute does not prevent an endocrine disorder; in addition, salt substitutes may contain high levels of potassium, which may lead to electrolyte imbalances.
8. The unlicensed assistive personnel (UAP) is providing care to a patient in sickle cell crisis. Which action by the UAP requires intervention by the supervising nurse?
a. Elevating the head of the bed to 25 degrees
b. Assisting to remove any restrictive clothing
c. Obtaining the blood pressure with an external cuff
d. Offering the patient her beverage of choice
c
5. A client with acute kidney injury has a blood pressure of 76/55 mm Hg. The health care provider ordered 1000 mL of normal saline to be infused over 1 hour to maintain perfusion. The client is starting to develop shortness of breath. What is the nurse's priority action?
a. Calculate the mean arterial pressure (MAP).
b. Ask for insertion of a pulmonary artery catheter. c. Take the client's pulse.
d. Slow down the normal saline infusion.
ANS: D
The nurse should assess that the client could be developing fluid overload and respiratory distress and slow down the normal saline infusion. The calculation of the MAP also reflects perfusion. The insertion of a pulmonary artery catheter would evaluate the client's hemodynamic status, but this should not be the initial action by the nurse. Vital signs are also important after adjusting the intravenous infusion.
7. An older adult is brought to the emergency department by a family member, who reports a moderate change in mental status and mild cough. The client is afebrile. The health care provider orders a chest x-ray. The family member questions why this is needed since the manifestations seem so vague. What response by the nurse is best?
a. "Chest x-rays are always ordered when we suspect pneumonia."
b. "Older people often have vague symptoms, so an x-ray is essential." c. "The x-ray can be done and read before laboratory work is reported." d. "We are testing for any possible source of infection in the client."
ANS: B
It is essential to obtain an early chest x-ray in older adults suspected of having pneumonia because symptoms are often vague. Waiting until definitive manifestations are present to obtain the x-ray leads to a costly delay in treatment. Stating that chest x-rays are always ordered does not give the family definitive information. The x-ray can be done while laboratory values are still pending, but this also does not provide specific information about the importance of a chest x-ray in this client. The client has manifestations of pneumonia, so the staff is not testing for any possible source of infection but rather is testing for a suspected disorder.
5. A nurse evaluates a client with acute glomerulonephritis (GN). Which manifestation should the nurse recognize as a positive response to the prescribed treatment?
a. The client has lost 11 pounds in the past 10 days.
b. The client's urine specific gravity is 1.048.
c. No blood is observed in the client's urine.
d. The client's blood pressure is 152/88 mm Hg.
ANS: A
Fluid retention is a major feature of acute GN. This weight loss represents fluid loss, indicating that the glomeruli are performing the function of filtration. A urine specific gravity of 1.048 is high. Blood is not usually seen in GN, so this finding would be expected. A blood pressure of 152/88 mm Hg is too high; this may indicate kidney damage or fluid overload.
7. A client appears dyspneic, but the oxygen saturation is 97%. What action by the nurse is best?
a. Assess for other manifestations of hypoxia. b. Change the sensor on the pulse oximeter.
c. Obtain a new oximeter from central supply. d. Tell the client to take slow, deep breaths.
ANS: A
Pulse oximetry is not always the most accurate assessment tool for hypoxia as many factors can interfere, producing normal or near-normal readings in the setting of hypoxia. The nurse should conduct a more thorough assessment. The other actions are not appropriate for a hypoxic client.
8. Which drug disrupts platelet action?
a. Vitamin K
b. Ibuprofen
c. Methyldopa
d. Azathioprine
b
6. A nurse administers prescribed adenosine (Adenocard) to a client. Which response should the nurse assess for as the expected therapeutic response?
a. Decreased intraocular pressure
b. Increased heart rate
c. Short period of asystole
d. Hypertensive crisis
ANS: C
Clients usually respond to adenosine with a short period of asystole, bradycardia, hypotension, dyspnea, and chest pain. Adenosine has no conclusive impact on intraocular pressure.
DIF: Applying/Application REF: 662
KEY: Cardiac electrical conduction| medication
MSC: Integrated Process: Nursing Process: Analysis
NOT: Client Needs Category: Physiological Integrity: Pharmacological and Parenteral Therapies
3. The nurse assesses a patient in the emergency department (ED) and finds the following: constipation, fatigue with increased sleeping time, impaired memory, facial puffiness, and weight gain. Which deficiency does the nurse recognize?
a. Hyperthyroidism
b. Hypothyroidism
c. Hyperparathyroidism
d. Hypoparathyroidism
b
5. After teaching a client who has stress incontinence, the nurse assesses the clients understanding. Which statement made by the client indicates a need for additional teaching?
a. I will limit my total intake of fluids.
b. I must avoid drinking alcoholic beverages.
c. I must avoid drinking caffeinated beverages.
d. I shall try to lose about 10% of my body weight.
A
Limiting fluids concentrates urine and can irritate tissues, leading to increased incontinence. Many people try to manage incontinence by limiting fluids. Alcoholic and caffeinated beverages are bladder stimulants. Obesity increases intra-abdominal pressure, causing incontinence.
67. The charge nurse is assigning clients for the
shift. Which client should be assigned to the
new graduate nurse?
1. The client diagnosed with cancer of the lung
who has chest tubes.
2. The client diagnosed with laryngeal spasms
who has stridor.
3. The client diagnosed with laryngeal cancer
who has multiple fistulas.
4. The client who is two (2) hours post-partial
laryngectomy.
1. Chest tubes are part of the nursing education
curriculum. The new graduate should
be capable of caring for this client or at
least knowing when to get assistance.
TEST-TAKING HINT: The question is asking for
the least compromised or most stable client.
The client in option "1" already has chest
tubes in place and is presumably stable.
A client is having a bone marrow biopsy and is extremely anxious. What action by the nurse is best?
a.
Assess client fears and coping mechanisms.
b.
Reassure the client this is a common test.
c.
Sedate the client prior to the procedure.
d.
Tell the client he or she will be asleep.
a.
Assess client fears and coping mechanisms.
ANS: A
Assessing the client's specific fears and coping mechanisms helps guide the nurse in providing holistic care that best meets the client's needs. Reassurance will be helpful but is not the best option. Sedation is usually used. The client may or may not be totally asleep during the procedure.
After thrombolytic therapy, the nurse working in the cardiac catheterization laboratory would be alarmed to notice which sign?
A 1-inch backup of blood in the IV tubing
Facial drooping
Partial thromboplastin time (PTT) 68 seconds
Report of chest pressure during dye injection
Facial drooping
During and after thrombolytic administration, the nurse observes for any indications of bleeding, including changes in neurologic status, which may indicate intracranial bleeding. A 1-inch backup of blood in the IV tubing may be related to IV positioning. If heparin is used, PTT reflects a therapeutic value. Reports of chest pressure during dye injection or stent deployment are considered an expected result of the procedure.
3. A nurse assesses a client who is recovering from a subtotal thyroidectomy. On the second postoperative day the client states, "I feel numbness and tingling around my mouth." What action should the nurse take?
a. Offer mouth care.
b. Loosen the dressing.
c. Assess for Chvostek's sign.
d. Ask the client orientation questions.
c. Numbness and tingling around the mouth or in the fingers and toes are manifestations of hypocalcemia, which could progress to cause tetany and seizure activity. The nurse should assess the client further by testing for Chvostek's sign and Trousseau's sign. Then the nurse should notify the provider. Mouth care, loosening the dressing, and orientation questions do not provide important information to prevent complications of low calcium levels.
After thrombolytic therapy, the nurse working in the cardiac catheterization laboratory would be alarmed to notice which sign?
a. A 1-inch backup of blood in the IV tubing
b. Facial drooping
c. Partial thromboplastin time (PTT) 68 seconds
d. Report of chest pressure during dye injection
b. Facial drooping
During and after thrombolytic administration, the nurse observes for any indications of bleeding, including changes in neurologic status, which may indicate intracranial bleeding. A 1-inch backup of blood in the IV tubing may be related to IV positioning. If heparin is used, PTT reflects a therapeutic value. Reports of chest pressure during dye injection or stent deployment are considered an expected result of the procedure.
A nurse cares for a client with excessive production of thyrocalcitonin (calcitonin). For which electrolyte imbalance should the nurse assess?
a. Potassium
b. Sodium
c. Calcium
d. Magnesium
ANS: C
Parafollicular cells produce thyrocalcitonin (calcitonin), which regulates serum calcium levels. Calcitonin has no impact on potassium, sodium, or magnesium balances.
7. A nurse cares for a client who is experiencing epistaxis. Which action should the nurse take first?
a. Initiate Standard Precautions.
b. Apply direct pressure.
c. Sit the client upright.
d. Loosely pack the nares with gauze.
ANS: A
The nurse should implement Standard Precautions and don gloves prior to completing the other actions.
DIF: Applying/Application REF: 532 KEY: Trauma
MSC: Integrated Process: Nursing Process: Implementation
NOT: Client Needs Category: Safe and Effective Care Environment: Safety and Infection Control
The family of a neutropenic client reports the client "is not acting right." What action by the nurse is the priority?
a. Ask the client about pain.
b. Assess the client for infection.
c. Delegate taking a set of vital signs.
d. Look at today's laboratory results.
b. Assess the client for infection.
5. Based on the nurse's knowledge of the normal function of the kidney, which large particles are not found in the urine because they are too large to filter through the glomerular capillary walls? (Select all that apply.)
a. Blood cells
b. Albumin
c. Other proteins
d. E.ectrolytes
e. water
a, b, c
5. The nurse is caring for a patient with an indwelling catheter. What intervention does the nurse use to minimize catheter-related infections?
a. Assess the patient daily to determine need for catheter.
b. Irrigate the catheter daily with sterile solution to remove debris.
c. Use sterile technique when opening system to obtain urine samples.
d. Apply antiseptic solutions or antibiotic ointments to the perineal area.
a
8. A nurse cares for a client with chronic obstructive pulmonary disease (COPD). The client states that he no longer enjoys going out with his friends. How should the nurse respond?
a. "There are a variety of support groups for people who have COPD."
b. "I will ask your provider to prescribe you with an antianxiety agent."
c. "Share any thoughts and feelings that cause you to limit social activities."
d. "Friends can be a good support system for clients with chronic disorders."
ANS: C
Many clients with moderate to severe COPD become socially isolated because they are embarrassed by frequent coughing and mucus production. They also can experience fatigue, which limits their activities. The nurse needs to encourage the client to verbalize thoughts and feelings so that appropriate interventions can be selected. Joining a support group would not decrease feelings of social isolation if the client does not verbalize feelings. Antianxiety agents will not help the client with social isolation. Encouraging a client to participate in activities without verbalizing concerns also would not be an effective strategy for decreasing social isolation.
DIF: Applying/Application REF: 561 KEY: Coping| support MSC: IntegratedProcess:Caring
NOT: Client Needs Category: Psychosocial Integrity
9. A nurse is teaching a client who has cystic fibrosis (CF). Which statement should the nurse include in this client's teaching?
a. "Take an antibiotic each day."
b. "Contact your provider to obtain genetic screening."
c. "Eat a well-balanced, nutritious diet."
d. "Plan to exercise for 30 minutes every day."
ANS: C
Clients with CF often are malnourished due to vitamin deficiency and pancreatic malfunction. Maintaining nutrition is essential. Daily antibiotics and daily exercise are not essential actions. Genetic screening would not help the client manage CF better.
DIF: Applying/Application REF: 567
KEY: Nutrition| patient education MSC: Integrated Process: Teaching/Learning NOT: Client Needs Category: Health Promotion and Maintenance
p. 813, Safety; Teamwork and Collaboration
The patient is a 44-year-old chemical plant foreman who developed acute myelogenous leukemia 6 months ago. His initial therapy was successful, and he is scheduled to have a stem cell transplant (bone marrow) with his identical twin brother as the donor. His brother lives in the same city and is a professor at a local university. The patient is very grateful that his brother will donate bone marrow and states that he is certain that he has no risk for infection during the procedure because his brother is his identical twin.
2. Is the patient correct in assuming that he has no risk for infection because the donor is his twin brother? Provide a rationale for your response.
No, the patient is not correct. The patient will still have to undergo a conditioning regimen to clear his own, potentially leukemic, cells. Thus he will still be profoundly immunosuppressed and at great risk for infection unless and until his brother's stem cells properly engraft and grow in his body.
7. A client has intra-arterial blood pressure monitoring after a myocardial infarction. The nurse notes the client's heart rate has increased from 88 to 110 beats/min, and the blood pressure dropped from 120/82 to 100/60 mm Hg. What action by the nurse is most appropriate?
a. Allow the client to rest quietly.
b. Assess the client for bleeding.
c. Document the findings in the chart.
d. Medicate the client for pain.
ANS: B
A major complication related to intra-arterial blood pressure monitoring is hemorrhage from the insertion site. Since these vital signs are out of the normal range, are a change, and are consistent with blood loss, the nurse should assess the client for any bleeding associated with the arterial line. The nurse should document the findings after a full assessment. The client may or may not need pain medication and rest; the nurse first needs to rule out any emergent bleeding.
DIF: Applying/Application REF: 771
KEY: Coronary artery disease| intra-arterial blood pressure monitoring| equipment safety| vitalsigns MSC: IntegratedProcess:NursingProcess:Assessment
NOT: Client Needs Category: Physiological Integrity: Reduction of Risk Potential
Question 1 of 16
The nurse is assessing a client for endocrine dysfunction. Which comment by the client indicates a need for further assessment?
a. "I am worried about losing my job because of cutbacks."
b. "I don't have any patience with my kids. I lose my temper faster."
c. "I don't seem to have any stressors now."
d. "My weight has been stable these past few years."
b
"I don't have any patience with my kids. I lose my temper faster."
Many endocrine problems can change a client's behavior, personality, and psychological responses; the client stating that he or she has become short-tempered warrants further assessment. Worrying about losing a job is a normal concern but does not give any indication of a need for further assessment. The nurse will need to assess the client's claim that he or she has no stressors at present because the client's response does not provide enough information to make this determination; however, the client's statement about losing patience is the priority. Weight gain or loss may or may not be an indication of an endocrine disorder.
5. Postrenal kidney injury can result from which conditions? (Select all that apply.)
a. Septic shock
b. Cervical cancer
c. Nephrolithiasis or ureterolithiasis
d. Heart failure
e. Neurogenic bladder
f. Prostate cancer
b, c, e, f
8. A nurse cares for a client after radiation therapy for lung cancer. The client reports a sore throat. Which action should the nurse take first?
a. Ask the client to gargle with mouthwash containing lidocaine.
b. Administer prescribed intravenous pain medications.
c. Explain that soreness is normal and will improve in a couple days.
d. Assess the client's neck for redness and swelling.
ANS: A
Mouthwashes and throat sprays containing a local anesthetic agent such as lidocaine or diphenhydramine can provide relief from a sore throat after radiation therapy. Intravenous pain medications may be used if local anesthetics are unsuccessful. The nurse should explain to the client that this is normal and assess the client's neck, but these options do not decrease the client's discomfort.
DIF: Remembering/Knowledge REF: 539
KEY: Cancer| pain| medication
MSC: Integrated Process: Nursing Process: Implementation
NOT: Client Needs Category: Physiological Integrity: Pharmacological and Parenteral Therapies
9. a patient admitted for sickle cell crisis is being discharged home. Which statement by the patient indicates the need for further post-discharge instruction?
a. "I will stop running 2 miles every morning."
b. "I will visit my friends in Denver."
c. "I will avoid the sauna at the gym."
d. "I will not drink alcoholic beverages."
b
The nurse is caring for a client in phase 1 cardiac rehabilitation. Which activity does the nurse suggest?
The need to increase activities slowly at home
Planning and participating in a walking program
Placing a chair in the shower for independent hygiene
Consultation with social worker for disability planning
Placing a chair in the shower for independent hygiene
Phase 1 begins with the acute illness and ends with discharge from the hospital; it focuses on promoting rest and allowing clients to improve their activities of daily living based on their abilities. Phase 2 begins after discharge and continues through convalescence at home, including consultation with a social worker for long-term planning; it consists of achieving and maintaining a vital and productive life while remaining within the limits of the heart's ability to respond to increases in activity and stress. Phase 3 refers to long-term conditioning, such as a walking program.
A nurse assesses a client who is prescribed a medication that stimulates beta1 receptors. Which assessment finding should alert the nurse to urgently contact the health care provider?
a. Heart rate of 50 beats/min
b. Respiratory rate of 18 breaths/min
c. Oxygenation saturation of 92%
d. Blood pressure of 144/69 mm Hg
ANS: A
Stimulation of beta1 receptor sites in the heart has positive chronotropic and inotropic actions. The nurse expects an increase in heart rate and increased cardiac output. The client with a heart rate of 50 beats/min would be cause for concern because this would indicate that the client was not responding to the medication. The other vital signs are within normal limits and do not indicate a negative response to the medication.
These data are obtained by the RN who is assessing a client who had a transsphenoidal hypophysectomy yesterday. What information has the most immediate implications for the client's care?
A. Dry lips and oral mucosa on examination
B. Nasal drainage that tests negative for glucose
C. Client report of a headache and stiff neck
D. Urine specific gravity of 1.016
C.
Headache and stiff neck (nuchal rigidity) are symptoms of meningitis that have immediate implications for the client's care. Dry lips and mouth are not unusual after surgery. Frequent oral rinses and the use of dental floss should be encouraged because the client cannot brush the teeth. Any nasal drainage should test negative for glucose; nasal drainage that tests positive for glucose indicates the presence of a cerebrospinal fluid leak. A urine specific gravity of 1.016 is within normal limits.
The nurse is caring for a client in phase 1 cardiac rehabilitation. Which activity does the nurse suggest?
a. The need to increase activities slowly at home
b. Planning and participating in a walking program
c. Placing a chair in the shower for independent hygiene
d. Consultation with social worker for disability planning
c. Placing a chair in the shower for independent hygiene
Phase 1 begins with the acute illness and ends with discharge from the hospital; it focuses on promoting rest and allowing clients to improve their activities of daily living based on their abilities. Phase 2 begins after discharge and continues through convalescence at home, including consultation with a social worker for long-term planning; it consists of achieving and maintaining a vital and productive life while remaining within the limits of the heart's ability to respond to increases in activity and stress. Phase 3 refers to long-term conditioning, such as a walking program.
The nurse is interviewing a patient who has iron deficiency anemia. Which symptom is the patient most likely to report?
a. Fatigue
b. Nights sweats
c. Calf pain
d. Blood in urine
a
68. The nurse is writing a care plan for a client
newly diagnosed with cancer of the larynx.
Which problem is the highest priority?
1. Wound infection.
2. Hemorrhage.
3. Respiratory distress.
4. Knowledge deficit.
3. Respiratory distress is the highest priority.
Hemorrhaging and infection are serious
problems, but airway is priority.
TEST-TAKING HINT: In cardiopulmonary
resuscitation the steps are airway, breathing,
and then circulation.
This concurs with
Maslow's hierarchy of needs, which places
oxygenation at the top of the hierarchy.
5. A patient has a family hisotryof the autosomal-dominant form of PKD and has therefore been advised to monitor for and report symptoms. What is an early symptom of PKD?
a. Headache
b. Pruritus
c. Edema
d. Notcuria
d
10. While assessing a client who is 12 hours postoperative after a thoracotomy for lung cancer, a nurse notices that the lower chest tube is dislodged. Which action should the nurse take first?
a. Assess for drainage from the site.
b. Cover the insertion site with sterile gauze. \
c. Contact the provider and obtain a suture kit.
d. Reinsert the tube using sterile technique.
ANS: B
Immediately covering the insertion site helps prevent air from entering the pleural space and causing a pneumothorax. The area will not reseal quickly enough to prevent air from entering the chest. The nurse should not leave the client to obtain a suture kit. An occlusive dressing may cause a tension pneumothorax. The site should only be assessed after the insertion site is covered. The provider should be called to reinsert the chest tube or prescribe other treatment options.
DIF: Applying/Application REF: 578
KEY: Drains| surgical care
MSC: Integrated Process: Nursing Process: Implementation
NOT: Client Needs Category: Physiological Integrity: Physiological Adaptation
A client is having a radioisotopic imaging scan. What action by the nurse is most important?
a.
Assess the client for shellfish allergies.
b.
Place the client on radiation precautions.
c.
Sedate the client before the scan.
d.
Teach the client about the procedure.
d.
Teach the client about the procedure.
ANS: D
The nurse should ensure that teaching is done and the client understands the procedure. Contrast dye is not used, so shellfish/iodine allergies are not related. The client will not be radioactive and does not need radiation precautions. Sedation is not used in this procedure.
7. A telemetry nurse assesses a client with third-degree heart block who has wide QRS complexes and a heart rate of 35 beats/min on the cardiac monitor. Which assessment should the nurse complete next?
a. Pulmonary auscultation
b. Pulse strength and amplitude
c. Level of consciousness
d. Mobility and gait stability
ANS: C
A heart rate of 40 beats/min or less with widened QRS complexes could have hemodynamic consequences. The client is at risk for inadequate cerebral perfusion. The nurse should assess for level of consciousness, light-headedness, confusion, syncope, and seizure activity. Although the other assessments should be completed, the client's level of consciousness is the priority.
DIF: Applying/Application REF: 670
KEY: Cardiac electrical conduction| vascular perfusion
MSC: IntegratedProcess:NursingProcess:Assessment
NOT: Client Needs Category: Physiological Integrity: Reduction of Risk Potential
4. The assessment findings of a male patient with anterior pituitary tumor include reports of changes in secondary sex characteristics, such as episodes of impotence and decreased libido. The nurse explains to the patient that these findings are a result of overproduction of which hormone?
a. Gonadotropins inhibiting prolactin (PRL)
b. Thyroid hormone inhibiting PRL
c. PRL inhibiting secretion of gonadotropins
d. Steroids inhibiting production of sex hormones
c
PRL inhibiting secretion of gonadotropins
5. A nurse contacts the health care provider after reviewing a client's laboratory results and noting a blood urea nitrogen (BUN) of 35 mg/dL and a creatinine of 1.0 mg/dL. For which action should the nurse recommend a prescription?
a. Intravenous fluids
b. Hemodialysis
c. Fluid restriction
d. Urine culture and sensitivity
ANS: A
Normal BUN is 10 to 20 mg/dL. Normal creatinine is 0.6 to 1.2 mg/dL (males) or 0.5 to 1.1 mg/dL (females). Creatinine is more specific for kidney function than BUN, because BUN can be affected by several factors (dehydration, high-protein diet, and catabolism). This client's creatinine is normal, which suggests a non-renal cause for the elevated BUN. A common cause of increased BUN is dehydration, so the nurse should anticipate giving the client more fluids, not placing the client on fluid restrictions. Hemodialysis is not an appropriate treatment for dehydration. The lab results do not indicate an infection; therefore, a urine culture and sensitivity is not appropriate.
6. A nurse cares for a client with an increased blood urea nitrogen (BUN)/creatinine ratio. Which action should the nurse take first?
a. Assess the client's dietary habits.
b. Inquire about the use of nonsteroidal anti-inflammatory drugs (NSAIDs).
c. Hold the client's metformin (Glucophage).
d. Contact the health care provider immediately.
ANS: A
An elevated BUN/creatinine ratio is often indicative of dehydration, urinary obstruction, catabolism, or a high-protein diet. The nurse should inquire about the client's dietary habits. Kidney damage related to NSAID use most likely would manifest with elevations in both BUN and creatinine, but no change in the ratio. The nurse should obtain more assessment data before holding any medications or contacting the provider.
8. A nurse is assisting the health care provider who is intubating a client. The provider has been attempting to intubate for 40 seconds. What action by the nurse takes priority?
a. Ensure the client has adequate sedation.
b. Find another provider to intubate.
c. Interrupt the procedure to give oxygen. d. Monitor the client's oxygen saturation.
ANS: C
Each intubation attempt should not exceed 30 seconds (15 is preferable) as it causes hypoxia. The nurse should interrupt the intubation attempt and give the client oxygen. The nurse should also have adequate sedation during the procedure and monitor the client's oxygen saturation, but these do not take priority. Finding another provider is not appropriate at this time.
When assessing the patient with darker skin for pallor and cyanosis, which area would the nurse examine?
a. Chest and abdomen
b. General appearance of face
c. Fingertips and toes
d. Oral mucous membranes
d
7. After administering newly prescribed captopril (Capoten) to a client with heart failure, the nurse implements interventions to decrease complications. Which priority intervention should the nurse implement for this client?
a. Provide food to decrease nausea and aid in absorption.
b. Instruct the client to ask for assistance when rising from bed.
c. Collaborate with unlicensed assistive personnel to bathe the client.
d. Monitor potassium levels and check for symptoms of hypokalemia.
ANS: B
Administration of the first dose of angiotensin-converting enzyme (ACE) inhibitors is often associated with hypotension, usually termed first-dose effect. The nurse should instruct the client to seek assistance before arising from bed to prevent injury from postural hypotension. ACE inhibitors do not need to be taken with food. Collaboration with unlicensed assistive personnel to provide hygiene is not a priority. The client should be encouraged to complete activities of daily living as independently as possible. The nurse should monitor for hyperkalemia, not hypokalemia, especially if the client has renal insufficiency secondary to heart failure.
DIF: Applying/Application REF: 685
KEY: Heart failure| angiotensin-converting enzyme (ACE) inhibitor| medication| patient education MSC: Integrated Process: Nursing Process: Implementation
NOT: Client Needs Category: Physiological Integrity: Pharmacological and Parenteral Therapies
4. Which hormones are secreted by the posterior pituitary gland? (Select all that apply.)
a. Testosterone
b. Oxytocin
c. Growth hormone (GH)
d. Antidiuretic hormone (ADH)
e. Cortisol
b, d
Oxytocin, Antidiuretic hormone (ADH)
A nurse cares for a client after a pituitary gland stimulation test using insulin. The clients post-stimulation laboratory results indicate elevated levels of growth hormone (GH) and adrenocorticotropic hormone (ACTH). How should the nurse interpret these results?
a. Pituitary hypofunction
b. Pituitary hyperfunction
c. Pituitary-induced diabetes mellitus
d. Normal pituitary response to insulin
D (Some tests for pituitary function involve administering agents that are known to stimulate the secretion of specific pituitary hormones and then measuring the response. Such tests are termed stimulation tests. The stimulation test for GH or ACTH assessment involves injecting the client with regular insulin (0.05 to 1 unit/kg of body weight) and checking circulating levels of GH and ACTH. The presence of insulin in clients with normal pituitary function causes increased release of GH and ACTH.)
7. A nurse assesses an older adult client who is experiencing a myocardial infarction. Which clinical manifestation should the nurse expect?
a. Excruciating pain on inspiration
b. Left lateral chest wall pain
c. Disorientation and confusion
d. Numbness and tingling of the arm
C
In older adults, disorientation or confusion may be the major manifestation of myocardial infarction caused by poor cardiac output. Pain manifestations and numbness and tingling of the arm could also be related to the myocardial infarction. However, the nurse should be more concerned about the new onset of disorientation or confusion caused by decreased perfusion.
A student nurse learns that the spleen has several functions. What functions do they include? (Select all that apply.)
a.
Breaks down hemoglobin
b.
Destroys old or defective red blood cells (RBCs)
c.
Forms vitamin K for clotting
d.
Stores extra iron in ferritin
e.
Stores platelets not circulating
a.
Breaks down hemoglobin
b.
Destroys old or defective red blood cells (RBCs)
e.
Stores platelets not circulating
ANS: A, B, E
Functions of the spleen include breaking down hemoglobin released from RBCs, destroying old or defective RBCs, and storing the platelets that are not in circulation. Forming vitamin K for clotting and storing extra iron in ferritin are functions of the liver.
8. A client has been diagnosed with tuberculosis (TB). What action by the nurse takes highest priority?
a. Educating the client on adherence to the treatment regimen b. Encouraging the client to eat a well-balanced diet
c. Informing the client about follow-up sputum cultures
d. Teaching the client ways to balance rest with activity
ANS: A
The treatment regimen for TB ranges from 6 to 12 months, making adherence problematic for many people. The nurse should stress the absolute importance of following the treatment plan for the entire duration of prescribed therapy. The other options are appropriate topics to educate this client on but do not take priority.
6. A patient with a history of PKD reports dull, aching flank pain and the urinalysis is negative for infection. The health care provider tells the nurse that the pain is chronic and related to enlarging kidneys compressing abdominal contents. What nursing intervention is best for this patient?
a. Administer trimethoprim/sulfamethoxazole (Bactrim).
b. Apply cool compresses to the abdomen or flank.
c. Teach methods of relaxation such as deep-breathing.
d. Administer around-the-clock nonsteroidal antiinflammtory drugs (NSAIDs).
c
6. When shock or other problems cause an acute reduction in blood flow to the kidneys, how do the kidneys compensate? (Select all that apply.)
a. Constrict blood vessels in the kidneys.
b. Activate the renin-angiotensin-aldosterone pathway.
c. Release beta blockers.
d. Dilate blood vessels throughout the body.
e. Release antidiuretic hormones.
a, b, e
(Chp. 31; elsevier resources)
(Chp. 31; elsevier resources)
10. A patient has polycythemia vera. Which action by the UAP requires intervention by the supervising nurse?
a. Assisting the patient to floss his teeth
b. Using an electric shaver on the patient
c. Using a soft-bristled toothbrush on the patient
d. Assisting the patient to don support hose
a
8. A nurse cares for a client with an intravenous temporary pacemaker for bradycardia. The nurse observes the presence of a pacing spike but no QRS complex on the client's electrocardiogram. Which action should the nurse take next?
a. Administer intravenous diltiazem (Cardizem).
b. Assess vital signs and level of consciousness.
c. Administer sublingual nitroglycerin.
d. Assess capillary refill and temperature.
ANS: B
In temporary pacing, the wires are threaded onto the epicardial surface of the heart and exit through the chest wall. The pacemaker spike should be followed immediately by a QRS complex. Pacing spikes seen without subsequent QRS complexes imply loss of capture. If there is no capture, then there is no ventricular depolarization and contraction. The nurse should assess for cardiac output via vital signs and level of consciousness. The other interventions would not determine if the client is tolerating the loss of capture.
DIF: Applying/Application REF: 664
KEY: Cardiac electrical conduction
MSC: IntegratedProcess:NursingProcess:Assessment
NOT: Client Needs Category: Physiological Integrity: Reduction of Risk Potential
Which factor is a hallmark assessment finding that signifies hyperthyroidism?
a. Weight loss
b. Increased libido
c. Heat intolerance
d. Diarrhea
c
6. Which task related to care of patients who have indwelling catheters can be delegated to unlicensed assistive personnel (UAP)?
a. Perform daily catheter care by washing the perineum and proximal portion of the catheter with soap and water.
b. Use sterile technique when inserting the urinary catheter or when opening the system to obtain urine samples.
c. Determine whether use of condom catheters is appropriate for male patients and apply the devices accordingly.
d. Keep urine collection bag in a place that is readily visible to the patient, so that the patient is reassured of kidney function.
a
9. An intubated client's oxygen saturation has dropped to 88%. What action by the nurse takes priority?
a. Determine if the tube is kinked. b. Ensure all connections are patent. c. Listen to the client's lung sounds. d. Suction the endotracheal tube.
ANS: C
When an intubated client shows signs of hypoxia, check for DOPE: displaced tube (most common cause), obstruction (often by secretions), pneumothorax, and equipment problems. The nurse listens for equal, bilateral breath sounds first to determine if the endotracheal tube is still correctly placed. If this assessment is normal, the nurse would follow the mnemonic and assess the patency of the tube and connections and perform suction.
A nurse is caring for a client who is about to receive a bone marrow transplant. To best help the client cope with the long recovery period, what action by the nurse is best?
a. Arrange a visitation schedule among friends and family.
b. Explain that this process is difficult but must be endured.
c. Help the client find things to hope for each day of recovery.
d. Provide plenty of diversionary activities for this time.
c. Help the client find things to hope for each day of recovery.
8. A nurse assesses a client after administering isosorbide mononitrate (Imdur). The client reports a headache. Which action should the nurse take?
a. Initiate oxygen therapy.
b. Hold the next dose of Imdur.
c. Instruct the client to drink water.
d. Administer PRN acetaminophen.
ANS: D
The vasodilating effects of isosorbide mononitrate frequently cause clients to have headaches during the initial period of therapy. Clients should be told about this side effect and encouraged to take the medication with food. Some clients obtain relief with mild analgesics, such as acetaminophen. The client's headache is not related to hypoxia or dehydration; therefore, these interventions would not help. The client needs to take the medication as prescribed to prevent angina; the medication should not be held.
DIF: Applying/Application REF: 686
KEY: Heart failure| nitroglycerin/nitrates| medication| pharmacologic pain management MSC: Integrated Process: Nursing Process: Implementation
NOT: Client Needs Category: Physiological Integrity: Pharmacological and Parenteral Therapies
Which action does the postanesthesia care unit (PACU) nurse perform first when caring for a client who has just arrived after a total thyroidectomy?
A. Assess the wound dressing for bleeding.
B. Give morphine sulfate 4 to 8 mg IV for pain.
C. Monitor oxygen saturation using pulse oximetry. Correct
D. Support the head and neck with sandbags.
C.
Airway assessment and management is always the first priority with every client. This is especially important for a client who has had surgery that involves potential bleeding and edema near the trachea. Assessing the wound dressing for bleeding is a high priority, although this is not the first priority. Pain control and supporting the head and neck with sandbags are important priorities, but can be addressed after airway assessment.
8. A nurse assesses a client 2 hours after a cardiac angiography via the left femoral artery. The nurse notes that the left pedal pulse is weak. Which action should the nurse take?
a. Elevate the leg and apply a sandbag to the entrance site.
b. Increase the flow rate of intravenous fluids.
c. Assess the color and temperature of the left leg.
d. Document the finding as left pedal pulse of +1/4.
C
Loss of a pulse distal to an angiography entry site is serious, indicating a possible arterial obstruction. The pulse may be faint because of edema. The left pulse should be compared with the right, and pulses should be compared with previous assessments, especially before the procedure. Assessing color (pale, cyanosis) and temperature (cool, cold) will identify a decrease in circulation. Once all peripheral and vascular assessment data are acquired, the primary health care provider should be notified. Simply documenting the findings is inappropriate. The leg should be positioned below the level of the heart or dangling to increase blood flow to the distal portion of the leg. Increasing intravenous fluids will not address the clients problem.
A nurse assesses a client who has a 15-year history of diabetes and notes decreased tactile sensation in both feet. Which action should the nurse take first?
a. Document the finding in the client's chart.
b. Assess tactile sensation in the client's hands.
c. Examine the client's feet for signs of injury.
d. Notify the health care provider.
ANS: C
Diabetic neuropathy is common when the disease is of long duration. The client is at great risk for injury in any area with decreased sensation because he or she is less able to feel injurious events. Feet are common locations for neuropathy and injury, so the nurse should inspect them for any signs of injury. After assessment, the nurse should document findings in the client's chart. Testing sensory perception in the hands may or may not be needed. The health care provider can be notified after assessment and documentation have been completed.
5. Based on the nurse's knowledge of the normal function of the kidney, which large particles are not found in the urine because they are too large to filter through the glomerular capillary walls? (Select all that apply.)
a. Blood cells
b. Albumin
c. Other proteins
d. Electrolytes
e. water
a, b, c
p. 813, Safety; Teamwork and Collaboration
The patient is a 44-year-old chemical plant foreman who developed acute myelogenous leukemia 6 months ago. His initial therapy was successful, and he is scheduled to have a stem cell transplant (bone marrow) with his identical twin brother as the donor. His brother lives in the same city and is a professor at a local university. The patient is very grateful that his brother will donate bone marrow and states that he is certain that he has no risk for infection during the procedure because his brother is his identical twin.
3. Which, if any, complications of stem cell transplantation are reduced or eliminated by having an identical sibling donate the stem cells?
The risk for GVHD is greatly reduced, although not completely eliminated, because the transplanted stem cells are identical to his own cells. The risk for veno-occlusive disease is likewise greatly reduced.
An older adult resident in a long-term-care facility becomes confused and agitated, telling the nurse, "Get out of here! You're going to kill me!" Which action will the nurse take first?
A) Check the resident's oxygen saturation.
B) Do a complete neurologic assessment.
C) Give the prescribed PRN lorazepam (Ativan).
D) Notify the resident's primary care provider.
(Chp. 31; elsevier resources)
A) Check the resident's oxygen saturation.
(Chp. 31; elsevier resources)
6. A client has a serum potassium level of 6.5 mmol/L, a serum creatinine level of 2 mg/dL, and a urine output of 350 mL/day. What is the best action by the nurse?
a. Place the client on a cardiac monitor immediately.
b. Teach the client to limit high-potassium foods.
c. Continue to monitor the client's intake and output.
d. Ask to have the laboratory redraw the blood specimen.
ANS: A
The priority action by the nurse should be to check the cardiac status with a monitor. High potassium levels can lead to dysrhythmias. The other choices are logical nursing interventions for acute kidney injury but not the best immediate action.
9. A client has been admitted for suspected inhalation anthrax infection. What question by the nurse is most important?
a. "Are any family members also ill?"
b. "Have you traveled recently?"
c. "How long have you been ill?" d. "What is your occupation?"
ANS: D
Inhalation anthrax is rare and is an occupational hazard among people who work with animal wool, bone meal, hides, and skin, such as taxidermists and veterinarians. Inhalation anthrax seen in someone without an occupational risk is considered a bioterrorism event and must be reported to authorities immediately. The other questions are appropriate for anyone with an infection.
A nursing student is struggling to understand the process of graft-versus-host disease. What explanation by the nurse instructor is best?
a. "Because of immunosuppression, the donor cells take over."
b. "It's like a transfusion reaction because no perfect matches exist."
c. "The client's cells are fighting donor cells for dominance."
d. "The donor's cells are actually attacking the client's cells."
d. "The donor's cells are actually attacking the client's cells."
The nurse is caring for a client 36 hours after coronary artery bypass grafting, with a priority problem of intolerance for activity related to imbalance of myocardial oxygen supply and demand. Which finding causes the nurse to terminate an activity and return the client to bed?
a. Pulse 60 beats/min and regular
b. Urinary frequency
c. Incisional discomfort
d. Respiratory rate 28 breaths/min
d. Respiratory rate 28 breaths/min
Tachypnea and tachycardia reflect activity intolerance; activity should be terminated. Pulse 60 beats/min and regular is a normal finding. Urinary frequency may indicate infection or diuretic use, but not activity intolerance. Pain with activity after surgery is anticipated; pain medication should be available.
Ch.62 p. 1273, Physiological Integrity
Which urine properties indicate to the nurse that the client with syndrome of inappropriate (SIADH) antidiuretic hormone is responding to interventions?
A. Urine output volume increased; urine specific gravity increased
B. Urine output volume increased; urine specific gravity decreased
C. Urine output volume decreased; urine specific gravity increased
D. Urine output volume decreased; urine specific gravity decreased
Answer: B
Rationale: SIADH involves excessive secretion of vasopressin (ADH) when it is not needed. Water is reabsorbed, causing an increase in blood volume and a decrease in urine volume. Blood concentration is diluted, and urine concentration, as measured by specific gravity, is highly increased. When interventions to counter act SIADH are effective, the person slows water reabsorption so that urine output volume increases at the same time that urine concentration decreases, seen as a decreased urine specific gravity.
8. A client is in the preoperative holding area prior to an emergency coronary artery bypass graft (CABG). The client is yelling at family members and tells the doctor to "just get this over with" when asked to sign the consent form. What action by the nurse is best?
a. Ask the family members to wait in the waiting area.
b. Inform the client that this behavior is unacceptable.
c. Stay out of the room to decrease the client's stress levels.
d. Tell the client that anxiety is common and that you can help.
ANS: D
Preoperative fear and anxiety are common prior to cardiac surgery, especially in emergent situations. The client is exhibiting anxiety, and the nurse should reassure the client that fear is common and offer to help. The other actions will not reduce the client's anxiety.
DIF: Applying/Application REF: 776
KEY: Coronary artery disease| preoperative nursing| psychosocial response| anxiety| coping| therapeutic communication MSC: Integrated Process: Caring
NOT: Client Needs Category: Psychosocial Integrity
Ch.61 Question 4 of 16
In type 1 diabetes, insulin injections are necessary to maintain which action between insulin and glucose?
a. Glucose intolerance
b. Homeostasis
c. Insulin intolerance
d. Negative feedback
b
Homeostasis
Insulin injections maintain homeostasis, or normal balance, between insulin and glucose in the client with type 1 diabetes. Type 1 diabetes is a lack of insulin production, not glucose intolerance, and requires frequent doses of insulin. Negative feedback does not occur in type 1 diabetes because of lack of insulin.
p. 813, Safety; Teamwork and Collaboration
The patient is a 44-year-old chemical plant foreman who developed acute myelogenous leukemia 6 months ago. His initial therapy was successful, and he is scheduled to have a stem cell transplant (bone marrow) with his identical twin brother as the donor. His brother lives in the same city and is a professor at a local university. The patient is very grateful that his brother will donate bone marrow and states that he is certain that he has no risk for infection during the procedure because his brother is his identical twin.
4. Which, if any, complications (and why) are still possible even with a donor who is an identical sibling?
In addition to infection, other pancytopenia problems can occur until full engraftment. The patient will be at risk for bleeding complications and may need both red blood cell and platelet transfusions after transplantation. It is possible that engraftment may not occur for many reasons, the most common of which is insufficient stem cells transplanted. Although HSCT can help cure leukemia, it is still possible that not all cancerous bone marrow cells were killed and could come back.
11. A nurse assesses a client who is prescribed fluticasone (Flovent) and notes oral lesions. Which action should the nurse take?
a. Encourage oral rinsing after fluticasone administration.
b. Obtain an oral specimen for culture and sensitivity.
c. Start the client on a broad-spectrum antibiotic.
d. Document the finding as a known side effect.
ANS: A
The drug reduces local immunity and increases the risk for local infection, especially Candida albicans. Rinsing the mouth after using the inhaler will decrease the risk for developing this infection. Use of mouthwash and broad-spectrum antibiotics is not warranted in this situation. The nurse should document the finding, but the best action to take is to have the client start rinsing his or her mouth after using fluticasone. An oral specimen for culture and sensitivity will not provide information necessary to care for this client.
DIF: Applying/Application REF: 554
KEY: Medication| fungal infection
MSC: Integrated Process: Nursing Process: Implementation
NOT: Client Needs Category: Physiological Integrity: Pharmacological and Parenteral Therapies
12. A nurse cares for a client who is infected with Burkholderia cepacia. Which action should the nurse take first when admitting this client to a pulmonary care unit?
a. Instruct the client to wash his or her hands after contact with other people.
b. Implement Droplet Precautions and don a surgical mask.
c. Keep the client isolated from other clients with cystic fibrosis. d. Obtain blood, sputum, and urine culture specimens.
ANS: C
Burkholderia cepacia infection is spread through casual contact between cystic fibrosis clients, thus the need for these clients to be separated from one another. Strict isolation measures will not be necessary. Although the client should wash his or her hands frequently, the most important measure that can be implemented on the unit is isolation of the client from other clients with cystic fibrosis. There is no need to implement Droplet Precautions or don a surgical mask when caring for this client. Obtaining blood, sputum, and urine culture specimens will not provide information necessary to care for a client with Burkholderia cepacia infection.
DIF: Applying/Application REF: 568
KEY: Pulmonary infection| infection control
MSC: Integrated Process: Nursing Process: Implementation
NOT: Client Needs Category: Safe and Effective Care Environment: Safety and Infection Control
6. After teaching a client with nephrotic syndrome and a normal glomerular filtration, the nurse assesses the client's understanding. Which statement made by the client indicates a correct understanding of the nutritional therapy for this condition?
a. "I must decrease my intake of fat."
b. "I will increase my intake of protein."
c. "A decreased intake of carbohydrates will be required." d. "An increased intake of vitamin C is necessary."
ANS: B
In nephrotic syndrome, the renal loss of protein is significant, leading to hypoalbuminemia and edema formation. If glomerular filtration is normal or near normal, increased protein loss should be matched by increased intake of protein. The client would not need to adjust fat, carbohydrates, or vitamins based on this disorder.
6. A nurse cares for adult clients who experience urge incontinence. For which client should the nurse plan a habit training program?
a. A 78-year-old female who is confused
b. A 65-year-old male with diabetes mellitus
c. A 52-year-old female with kidney failure
d. A 47-year-old male with arthritis
A
For a bladder training program to succeed in a client with urge incontinence, the client must be alert, aware, and able to resist the urge to urinate. Habit training will work best for a confused client. This includes going to the bathroom (or being assisted to the bathroom) at set times. The other clients may benefit from another type of bladder training.
9. A nurse cares for a client who had a partial laryngectomy 10 days ago. The client states that all food tastes bland. How should the nurse respond?
a. "I will consult the speech therapist to ensure you are swallowing properly."
b. "This is normal after surgery. What types of food do you like to eat?"
c. "I will ask the dietitian to change the consistency of the food in your diet."
d. "Replacement of protein, calories, and water is very important after surgery."
ANS: B
Many clients experience changes in taste after surgery. The nurse should identify foods that the client wants to eat to ensure the client maintains necessary nutrition. Although the nurse should collaborate with the speech therapist and dietitian to ensure appropriate replacement of protein, calories, and water, the other responses do not address the client's concerns.
DIF: Applying/Application REF: 533
KEY: Surgical care| nutrition
MSC: Integrated Process: Communication and Documentation
NOT: Client Needs Category: Physiological Integrity: Basic Care and Comfort
11. A patient with a low white blood cell count is being discharged home. In which situations will the patient be instructed by the nurse to contact his or her health care provider? (Select all that apply.)
a. When temperature goes over 102F (38.9C)
b. A persistent cough develops with or without sputum
c. Pus or foul-smelling drainage develops
d. Whenever exposed to fresh fruit or vegetables or live plants
e. Urine is cloudy or foul-smelling, or if burning on urination is expeirenced
b, c, e
The nurse is caring for a client 36 hours after coronary artery bypass grafting, with a priority problem of intolerance for activity related to imbalance of myocardial oxygen supply and demand. Which finding causes the nurse to terminate an activity and return the client to bed?
Pulse 60 beats/min and regular
Urinary frequency
Incisional discomfort
Respiratory rate 28 breaths/min
Respiratory rate 28 breaths/min
Tachypnea and tachycardia reflect activity intolerance; activity should be terminated. Pulse 60 beats/min and regular is a normal finding. Urinary frequency may indicate infection or diuretic use, but not activity intolerance. Pain with activity after surgery is anticipated; pain medication should be available.
4. Which factor is a hallmark assessment finding that signifies hyperthyroidism?
a. Weight loss
b. Increased libido
c. Heat intolerance
d. Diarrhea
c
6. A nurse cares for adult clients who experience urge incontinence. For which client should the nurse plan a habit training program?
a. A 78-year-old female who is confused
b. A 65-year-old male with diabetes mellitus
c. A 52-year-old female with kidney failure d. A 47-year-old male with arthritis
ANS: A
For a bladder training program to succeed in a client with urge incontinence, the client must be alert, aware, and able to resist the urge to urinate. Habit training will work best for a confused client. This includes going to the bathroom (or being assisted to the bathroom) at set times. The other clients may benefit from another type of bladder training.
69. The male client has had a radial neck dissection
for cancer of the larynx. Which action by the
client indicates a disturbance in body image?
1. The client requests a consultation by the
speech therapist.
2. The client has a towel placed over the mirror.
3. The client is attempting to shave himself.
4. The client practices neck and shoulder
exercises.
...
Which laboratory result would indicate that the prescription for Epogen is having the desired therapeutic effect?
a. Increase in platelet count
b. Increase in white blood cell (WBC) count
c. Increase in red blood cell (RBC) count
d. Increase in iron level
c
9. A nurse prepares to defibrillate a client who is in ventricular fibrillation. Which priority intervention should the nurse perform prior to defibrillating this client?
a. Make sure the defibrillator is set to the synchronous mode.
b. Administer 1 mg of intravenous epinephrine.
c. Test the equipment by delivering a smaller shock at 100 joules.
d. Ensure that everyone is clear of contact with the client and the bed.
ANS: D
To avoid injury, the rescuer commands that all personnel clear contact with the client or the bed and ensures their compliance before delivery of the shock. A precordial thump can be delivered when no defibrillator is available. Defibrillation is done in asynchronous mode. Equipment should not be tested before a client is defibrillated because this is an emergency procedure; equipment should be checked on a routine basis. Epinephrine should be administered after defibrillation.
DIF: Applying/Application REF: 668
KEY: Cardiac electrical conduction| safety
MSC: Integrated Process: Nursing Process: Implementation
NOT: Client Needs Category: Safe and Effective Care Environment: Safety and Infection Control
4. A nurse assesses a client on the medical-surgical unit. Which statement made by the client should alert the nurse to the possibility of hypothyroidism?
a. "My sister has thyroid problems."
b. "I seem to feel the heat more than other people."
c. "Food just doesn't taste good without a lot of salt."
d. "I am always tired, even with 12 hours of sleep."
d. Clients with hypothyroidism usually feel tired or weak despite getting many hours of sleep. Thyroid problems are not inherited. Heat intolerance is indicative of hyperthyroidism. Loss of taste is not a manifestation of hypothyroidism.
A nurse assesses a client who is prescribed a medication that stimulates beta1 receptors. Which assessment finding should alert the nurse to urgently contact the health care provider?
a. Heart rate of 50 beats/min
b. Respiratory rate of 18 breaths/min
c. Oxygenation saturation of 92%
d. Blood pressure of 144/69 mm Hg
ANS: A
Stimulation of beta1 receptor sites in the heart has positive chronotropic and inotropic actions. The nurse expects an increase in heart rate and increased cardiac output. The client with a heart rate of 50 beats/min would be cause for concern because this would indicate that the client was not responding to the medication. The other vital signs are within normal limits and do not indicate a negative response to the medication.
9. A nurse teaches a client who is prescribed digoxin (Lanoxin) therapy. Which statement should the nurse include in this client's teaching?
a. "Avoid taking aspirin or aspirin-containing products."
b. "Increase your intake of foods that are high in potassium."
c. "Hold this medication if your pulse rate is below 80 beats/min."
d. "Do not take this medication within 1 hour of taking an antacid."
ANS: D
Gastrointestinal absorption of digoxin is erratic. Many medications, especially antacids, interfere with its absorption. Clients are taught to hold their digoxin for bradycardia; a heart rate of 80 beats/min is too high for this cutoff. Potassium and aspirin have no impact on digoxin absorption, nor do these statements decrease complications of digoxin therapy.
DIF: Applying/Application REF: 686
KEY: Heart failure| digoxin| medication| patient education
MSC: Integrated Process: Nursing Process: Implementation
NOT: Client Needs Category: Physiological Integrity: Pharmacological and Parenteral Therapies
An older client asks the nurse why "people my age" have weaker immune systems than younger people. What responses by the nurse are best? (Select all that apply.)
a.
"Bone marrow produces fewer blood cells."
b.
"You may have decreased levels of circulating platelets."
c.
"You have lower levels of plasma proteins in the blood."
d.
"Lymphocytes become more reactive to antigens."
e.
"Spleen function declines after age 60."
a.
"Bone marrow produces fewer blood cells."
c.
"You have lower levels of plasma proteins in the blood."
ANS: A, C
The aging adult has bone marrow that produces fewer cells and decreased blood volume with fewer plasma proteins. Platelet numbers remain unchanged, lymphocytes become less reactive, and spleen function stays the same.
10. A nurse cares for a client who is scheduled for a total laryngectomy. Which action should the nurse take prior to surgery?
a. Assess airway patency, breathing, and circulation.
b. Administer prescribed intravenous pain medication.
c. Assist the client to choose a communication method.
d. Ambulate the client in the hallway to assess gait.
ANS: C
The client will not be able to speak after surgery. The nurse should assist the client to choose a communication method that he or she would like to use after surgery. Assessing the client's airway and administering IV pain medication are done after the procedure. Although ambulation promotes health and decreases the complications of any surgery, this client's gait should not be impacted by a total laryngectomy and therefore is not a priority.
DIF: Applying/Application REF: 540 KEY: Surgical care MSC: IntegratedProcess:Caring
NOT: Client Needs Category: Psychosocial Integrity
7. A client has just had a central line catheter placed that is specific for hemodialysis. What is the most appropriate action by the nurse?
a. Use the catheter for the next laboratory blood draw.
b. Monitor the central venous pressure through this line.
c. Access the line for the next intravenous medication.
d. Place a heparin or heparin/saline dwell after hemodialysis.
ANS: D
The central line should have a heparin or heparin/saline dwell after hemodialysis treatment. The central line catheter used for dialysis should not be used for blood sampling, monitoring central venous pressures, or giving drugs or fluids.
70. The HCP has recommended a total
laryngectomy for a male client diagnosed with
cancer of the larynx but the client refuses.
Which intervention by the nurse illustrates the
ethical principle of nonmalfeasance?
1. The nurse listens to the client explain why he
is refusing surgery.
2. The nurse and significant other insist that the
client have the surgery.
3. The nurse refers the client to a counselor for
help with the decision.
4. The nurse asks a cancer survivor to come and
discuss the surgery with the client.
2. Placing a towel over the mirror indicates
the client is having difficulty looking at
his reflection, a body-image problem.
TEST-TAKING HINT: The test taker must try to
match the problem with the answer choices.
This would eliminate options "1" and "4."
Shaving himself is a positive action, and the
question asks for an action indicating a disturbance
in body image.
Based on knowledge of physiologic triggers for RBC production, the nurse would anticipate which chronic health condition to be associated with an increase in RBC production?
a. Diabetes mellitus
b. Osteoarthritis
c. Chronic obstructive pulmonary disease
d. Chronic kidney disease
c
9. A nurse assesses a client who is recovering after a left-sided cardiac catheterization. Which assessment finding requires immediate intervention?
a. Urinary output less than intake
b. Bruising at the insertion site
c. Slurred speech and confusion
d. Discomfort in the left leg
C
A left-sided cardiac catheterization specifically increases the risk for a cerebral vascular accident. A change in neurologic status needs to be acted on immediately. Discomfort and bruising are expected at the site. If intake decreases, a client can become dehydrated because of dye excretion. The second intervention would be to increase the clients fluid status. Neurologic changes would take priority.
Question 2 of 16
Which negative feedback response is responsible for preventing hypoglycemia during sleep in nondiabetic clients?
a. Alpha cells of the pancreas
b. Beta cells of the pancreas
c. Glucagon release
d. Insulin release
c
Glucagon release
Glucagon is the hormone that binds to receptors on liver cells. This causes the liver cells to convert glycogen to glucose, which keeps blood sugar levels normal during sleep. Alpha cells are responsible for synthesizing and secreting the hormone glucagon. Beta cells are responsible for synthesizing and secreting the hormone insulin. Insulin is the hormone responsible for lowering blood glucose. Insulin improves glucose uptake by the cell.
7. After delegating care to an unlicensed assistive personnel (UAP) for a client who is prescribed habit training to manage incontinence, a nurse evaluates the UAPs understanding. Which action indicates the UAP needs additional teaching?
a. Toileting the client after breakfast
b. Changing the clients incontinence brief when wet
c. Encouraging the client to drink fluids
d. Recording the clients incontinence episodes
B
Habit training is undermined by the use of absorbent incontinence briefs or pads. The nurse should re-educate the UAP on the technique of habit training. The UAP should continue to toilet the client after meals, encourage the client to drink fluids, and record incontinent episodes.
The nurse in the coronary care unit is caring for a group of clients who have had myocardial infarction. Which client does the nurse see first?
a. Client with dyspnea on exertion when ambulating to the bathroom
b. Client with third-degree heart block on the monitor
c. Client with normal sinus rhythm and PR interval of 0.28 second
d. Client who refuses to take heparin or nitroglycerin
b. Client with third-degree heart block on the monitor
Third-degree heart block is a serious complication that indicates that a large portion of the left ventricle and conduction system are involved, so the client with the third-degree heart block should be seen first. Third-degree heart block usually requires pacemaker insertion. A normal rhythm with prolonged PR interval indicates first-degree heart block, which usually does not require treatment. The client with dyspnea on exertion when ambulating to the bathroom is not at immediate risk. The client's uncooperative behavior when refusing to take heparin or nitroglycerin may indicate fear or denial; he should be seen after emergency situations have been handled.
A client with syndrome of inappropriate antidiuretic hormone is admitted with a serum sodium level of 105 mEq/L. Which request by the health care provider does the nurse address first?
A. Administer infusion of 150 mL of 3% NaCl over 3 hours.
B. Draw blood for hemoglobin and hematocrit.
C. Insert retention catheter and monitor urine output.
D. Weigh the client on admission and daily thereafter.
A.
The client with a sodium level of 105 mEq/L is at high risk for seizures and coma. The priority intervention is to increase the sodium level to a more normal range. Ideally, 3% NaCl should be infused through a central line or with a small needle through a large vein to prevent irritation. Monitoring laboratory values for fluid balance and monitoring urine output are important, but are not the top priority. Monitoring client weight will help in the assessment of fluid balance; however, this is also not the top priority.
A nursing student learns that many drugs can impair the immune system. Which drugs does this include? (Select all that apply.)
a.
Acetaminophen (Tylenol)
b.
Amphotericin B (Fungizone)
c.
Ibuprofen (Motrin)
d.
Metformin (Glucophage)
e.
Nitrofurantoin (Macrobid)
b.
Amphotericin B (Fungizone)
c.
Ibuprofen (Motrin)
e.
Nitrofurantoin (Macrobid)
ANS: B, C, E
Amphotericin B, ibuprofen, and nitrofurantoin all can disrupt the hematologic (immune) system. Acetaminophen and metformin do not.
13. A nurse cares for a client who had a chest tube placed 6 hours ago and refuses to take deep breaths because of the pain. Which action should the nurse take?
a. Ambulate the client in the hallway to promote deep breathing.
b. Auscultate the client's anterior and posterior lung fields.
c. Encourage the client to take shallow breaths to help with the pain.
d. Administer pain medication and encourage the client to take deep breaths.
ANS: D
A chest tube is placed in the pleural space and may be uncomfortable for a client. The nurse should provide pain medication to minimize discomfort and encourage the client to take deep breaths. The other responses do not address the client's discomfort and need to take deep breaths to prevent complications.
DIF: Applying/Application REF: 580
KEY: Pain| pharmacologic pain management| drain| surgical care
MSC: Integrated Process: Nursing Process: Implementation
NOT: Client Needs Category: Physiological Integrity: Reduction of Risk Potential
7. A nurse assesses a client who is recovering from a radical nephrectomy for renal cell carcinoma. The nurse notes that the client's blood pressure has decreased from 134/90 to 100/56 mm Hg and urine output is 20 mL for this past hour. Which action should the nurse take?
a. Position the client to lay on the surgical incision. b. Measure the specific gravity of the client's urine. c. Administer intravenous pain medications.
d. Assess the rate and quality of the client's pulse.
ANS: D
The nurse should first fully assess the client for signs of volume depletion and shock, and then notify the provider. The radical nature of the surgery and the proximity of the surgery to the adrenal gland put the client at risk for hemorrhage and adrenal insufficiency. Hypotension is a clinical manifestation associated with both hemorrhage and adrenal insufficiency. Hypotension is particularly dangerous for the remaining kidney, which must receive adequate perfusion to function effectively. Re-positioning the client, measuring specific gravity, and administering pain medication would not provide data necessary to make an appropriate clinical decision, nor are they appropriate interventions at this time.
10. A client is on a ventilator and is sedated. What care may the nurse delegate to the unlicensed assistive personnel (UAP)?
a. Assess the client for sedation needs.
b. Get family permission for restraints.
c. Provide frequent oral care per protocol. d. Use nonverbal pain assessment tools.
ANS: C
The client on mechanical ventilation needs frequent oral care, which can be delegated to the UAP. The other actions fall within the scope of practice of the nurse.
p. 814, Safe and Effective Care Environment
The client is 3 weeks post-transplant from an allogeneic stem cell transplantation for acute lymphocytic leukemia. There is now some peeling of the client's skin on the palms of the hands and the soles of the feet. Which additional assessment data supports the nurse's suspicion of possible graft-versus-host disease (GVHD)?
A. The client's temperature is slightly below normal.
B. Today's platelet count is 5,000/mm3 and the WBCs are low.
C. The client has had 6 to 10 watery stools daily for 3 days.
D. The client's urine output is less than 800 mL in 24 hours.
Answer: C
Rationale: GVHD occurs when the immunocompetent cells of the donated marrow recognize the patient's (recipient) cells, tissues, and organs as foreign and start an immunologic attack against them. The tissues most susceptible are the skin, intestinal tract, and liver. The earliest manifestation of gastrointestinal involvement for GVHD is large-volume watery diarrhea. The temperature is unaffected. The fact that the urine output is low is related to dehydration from diarrhea, not kidney damage by GVHD.
The nurse in the coronary care unit is caring for a group of clients who have had myocardial infarction. Which client does the nurse see first?
Client with dyspnea on exertion when ambulating to the bathroom
Client with third-degree heart block on the monitor
Client with normal sinus rhythm and PR interval of 0.28 second
Client who refuses to take heparin or nitroglycerin
Client with third-degree heart block on the monitor
Third-degree heart block is a serious complication that indicates that a large portion of the left ventricle and conduction system are involved, so the client with the third-degree heart block should be seen first. Third-degree heart block usually requires pacemaker insertion. A normal rhythm with prolonged PR interval indicates first-degree heart block, which usually does not require treatment. The client with dyspnea on exertion when ambulating to the bathroom is not at immediate risk. The client's uncooperative behavior when refusing to take heparin or nitroglycerin may indicate fear or denial; he should be seen after emergency situations have been handled.
5. A patient with a PRL-secreting tumor is likely to be treated with which medications?
a. Dopamine agonists
b. Vasopressin
c. Steroids
d. Growth hormone (GH)
a
Dopamine agonists
7. After delegating care to an unlicensed assistive personnel (UAP) for a client who is prescribed habit training to manage incontinence, a nurse evaluates the UAP's understanding. Which action indicates the UAP needs additional teaching?
a. Toileting the client after breakfast
b. Changing the client's incontinence brief when wet c. Encouraging the client to drink fluids
d. Recording the client's incontinence episodes
ANS: B
Habit training is undermined by the use of absorbent incontinence briefs or pads. The nurse should re-educate the UAP on the technique of habit training. The UAP should continue to toilet the client after meals, encourage the client to drink fluids, and record incontinent episodes.
7. The nurse is caring for a patient who sustained major injuries in an automobile accident. Which blood pressure will result in compromised kidney function, in particular the glomerular filtration rate (GFR)?
a. 150/70 mm Hg
b. 70/40 mm Hg
c. 80/60 mm Hg
d. 140/80 mm Hg
b
10. A nurse is caring for several older clients in the hospital that the nurse identifies as being at high risk for healthcare-associated pneumonia. To reduce this risk, what activity should the nurse delegate to the unlicensed assistive personnel (UAP)?
a. Encourage between-meal snacks.
b. Monitor temperature every 4 hours. c. Provide oral care every 4 hours.
d. Report any new onset of cough.
ANS: C
Oral colonization by gram-negative bacteria is a risk factor for healthcare-associated pneumonia. Good, frequent oral care can help prevent this from developing and is a task that can be delegated to the UAP. Encouraging good nutrition is important, but this will not prevent pneumonia. Monitoring temperature and reporting new cough in clients is important to detect the onset of possible pneumonia but do not prevent it.
12. Which food should a patient with a low white blood cell count be encouraged to eat?
a. Fresh strawberries
b. Raw carrots
c. Green leaf lettuce
d. Well-done poultry
d
9. A client is in the clinic a month after having a myocardial infarction. The client reports sleeping well since moving into the guest bedroom. What response by the nurse is best?
a. "Do you have any concerns about sexuality?"
b. "I'm glad to hear you are sleeping well now."
c. "Sleep near your spouse in case of emergency."
d. "Why would you move into the guest room?"
ANS: A
Concerns about resuming sexual activity are common after cardiac events. The nurse should gently inquire if this is the issue. While it is good that the client is sleeping well, the nurse should investigate the reason for the move. The other two responses are likely to cause the client to be defensive.
DIF: Applying/Application REF: 781
KEY: Coronary artery disease| sexuality| anxiety| therapeutic communication MSC: IntegratedProcess:Caring
NOT: Client Needs Category: Psychosocial Integrity
5. Which hormones are secreted by the thyroid gland? (Select all that apply.)
a. Calcitonin
b. Somatostatin
c. Glucagon
d. Thyroxine (T4)
e. Aldosterone
f. Triiodothyronine (T3)
a, d, f
Calcitonin, Thyroxine (T4), Triiodothyronine (T3)
10. A client in the cardiac stepdown unit reports severe, crushing chest pain accompanied by nausea and vomiting. What action by the nurse takes priority?
a. Administer an aspirin.
b. Call for an electrocardiogram (ECG).
c. Maintain airway patency.
d. Notify the provider.
ANS: C
Airway always is the priority. The other actions are important in this situation as well, but the nurse should stay with the client and ensure the airway remains patent (especially if vomiting occurs) while another person calls the provider (or Rapid Response Team) and facilitates getting an ECG done. Aspirin will probably be administered, depending on the provider's prescription and the client's current medications.
DIF: Applying/Application REF: 769
KEY: Coronary artery disease| critical rescue| medical emergencies
MSC: Integrated Process: Nursing Process: Implementation
NOT: Client Needs Category: Safe and Effective Care Environment: Management of Care
After teaching a client with acromegaly who is scheduled for a hypophysectomy, the nurse assesses the clients understanding. Which statement made by the client indicates a need for additional teaching?
a. I will no longer need to limit my fluid intake after surgery.
b. I am glad no visible incision will result from this surgery.
c. I hope I can go back to wearing size 8 shoes instead of size 12.
d. I will wear slip-on shoes after surgery to limit bending over.
C (Although removal of the tissue that is oversecreting hormones can relieve many symptoms of hyperpituitarism, skeletal changes and organ enlargement are not reversible. It will be appropriate for the client to drink as needed postoperatively and avoid bending over. The client can be reassured that the incision will not be visible.)
The nurse is caring for a client with leukemia who has the priority problem of fatigue. What action by the client best indicates that an important goal for this problem has been met?
a. Doing activities of daily living (ADLs) using rest periods
b. Helping plan a daily activity schedule
c. Requesting a sleeping pill at night
d. Telling visitors to leave when fatigued
a. Doing activities of daily living (ADLs) using rest periods
A client is admitted to the emergency department (ED) with a possible diagnosis of avian influenza ("bird flu"). Which of these actions included in the hospital protocol for avian influenza will the nurse take first?
A) Ensure that ED staff members receive oseltamivir (Tamiflu).
B) Obtain specimens for the H5 polymerase chain reaction test.
C) Place the client in a negative air pressure room.
D) Start an IV line and administer rehydration therapy.
(Chp. 31; elsevier resources)
C) Place the client in a negative air pressure room.
(Chp. 31; elsevier resources)
7. The nurse reads in the patient's chart that he has acute-on-chronic kidney disease. How does the nurse interpret this information?
a. Kidney disease has progressed to the need for dialysis or transplant.
b. Patient has chronic kidney disease and has sustained an acute kidney injury.
c. Acute kidney injury requires aggressive management to prevent chronic disease.
d. The condition could by acute or chronic; further diagnostic testing is needed.
b
7. Why may a patient with PKD experience constipation?
a. Polycystic kidneys enlarge and put pressure on the large intestine.
b. Patient becomes dehydrated because the kdineys are dysfunctional.
c. Constipation is a side effect from the medications given to treat PKD.
d. Patients with PKD have special dietary restrictions that cause constipation.
a
11. A nurse is caring for a client on mechanical ventilation. When double-checking the ventilator settings with the respiratory therapist, what should the nurse ensure as a priority? a. The client is able to initiate spontaneous breaths.
b. The inspired oxygen has adequate humidification.
c. The upper peak airway pressure limit alarm is off. d. The upper peak airway pressure limit alarm is on.
ANS: D
The upper peak airway pressure limit alarm will sound when the airway pressure reaches a preset maximum. This is critical to prevent damage to the lungs. Alarms should never be turned off. Initiating spontaneous breathing is important for some modes of ventilation but not others. Adequate humidification is important but does not take priority over preventing injury.
A nurse is caring for a young male client with lymphoma who is to begin treatment. What teaching topic is a priority?
a. Genetic testing
b. Infection prevention
c. Sperm banking
d. Treatment options
c. Sperm banking
The client in the cardiac care unit has had a large myocardial infarction. How does the nurse recognize onset of left ventricular failure?
Urine output of 1500 mL on the preceding day
Crackles in the lung fields
Pedal edema
Expectoration of yellow sputum
Crackles in the lung fields
Manifestations of left ventricular failure and pulmonary edema are noted by listening for crackles and identifying their locations in the lung fields. A urine output of 1500 mL is normal. Edema is a sign of right ventricular heart failure. Yellow sputum indicates the presence of white blood cells and possible infection.
Which factor is a main assessment finding that signifies hypothyroidism?
a. Irritability
b. Cold intolerance
c. Diarrhea
d. Fatigue
b
7. A nurse cares for a client with a urine specific gravity of 1.040. Which action should the nurse take?
a. Obtain a urine culture and sensitivity. b. Place the client on restricted fluids. c. Assess the client's creatinine level.
d. Increase the client's fluid intake.
ANS: D
Normal specific gravity for urine is 1.005 to 1.030. A high specific gravity can occur with dehydration, decreased kidney blood flow (often because of dehydration), and the presence of antidiuretic hormone. Increasing the client's fluid intake would be a beneficial intervention. Assessing the creatinine or obtaining a urine culture would not provide data necessary for the nurse to make a clinical decision.
11. The emergency department (ED) manager is reviewing client charts to determine how well the staff performs when treating clients with community-acquired pneumonia. What outcome demonstrates that goals for this client type have been met?
a. Antibiotics started before admission
b. Blood cultures obtained within 20 minutes c. Chest x-ray obtained within 30 minutes
d. Pulse oximetry obtained on all clients
ANS: A
Goals for treatment of community-acquired pneumonia include initiating antibiotics prior to inpatient admission or within 6 hours of presentation to the ED. Timely collection of blood cultures, chest x-ray, and pulse oximetry are important as well but do not coincide with established goals.
p. 818, Prioritization, Delegation, and Supervision
The patient is a 52-year-old woman who has undergone an autologous stem cell transplantation for non-Hodgkin's lymphoma. She is recovering, and her white blood cell count is improving but still very low. She remains on neutropenic precautions. The LPN reports that the patient's heart rate, respiratory rate, temperature, and blood pressure are all elevated.
1. Which vital sign finding would you report to the health care provider immediately and why?
The temperature elevation, no matter how slight, in a patient with neutropenia indicates infection until it has been ruled out. This elevation should be reported immediately, and you need to take the standard neutropenic interventions of full assessment, obtaining appropriate specimens for culture, obtaining chest radiography, and starting antibiotic therapy.
The client in the cardiac care unit has had a large myocardial infarction. How does the nurse recognize onset of left ventricular failure?
a. Urine output of 1500 mL on the preceding day
b. Crackles in the lung fields
c. Pedal edema
d. Expectoration of yellow sputum
b. Crackles in the lung fields
Manifestations of left ventricular failure and pulmonary edema are noted by listening for crackles and identifying their locations in the lung fields. A urine output of 1500 mL is normal. Edema is a sign of right ventricular heart failure. Yellow sputum indicates the presence of white blood cells and possible infection.
A nurse prepares to palpate a client's thyroid gland. Which action should the nurse take when performing this assessment?
a. Stand in front of the client instead of behind the client.
b. Ask the client to swallow after palpating the thyroid.
c. Palpate the right lobe with the nurse's left hand.
d. Place the client in a sitting position with the chin tucked down.
ANS: D
The client should be in a sitting position with the chin tucked down as the examiner stands behind the client. The nurse feels for the thyroid isthmus while the client swallows and turns the head to the right, and the nurse palpates the right lobe with the right hand. The technique is repeated in the opposite fashion for the left lobe.
Ch.62 p. 1276, Patient-Centered Care; Quality Improvement; Safety
The patient is a 32-year-old woman admitted to your unit after surgery for fractures of the left arm and leg resulting from a car crash. She is awake and able to verify her medical history of rheumatoid arthritis and her usual daily medications. These are 10 mg of prednisone, naproxen 800 mg twice daily, oral contraceptives, calcium 600 mg, and one multiple vitamin tablet. All of these are prescribed for her to receive during her hospitalization. She is concerned about pain management and how long the recovery will be for the fractures. She is friendly, somewhat anxious, asks many questions, and wants to do "her part" to ensure good recovery. Over the next 4 days, she has become quieter, mumbles that her head and stomach hurt, and now does not recognize the assistant who has been providing her daily care. When she receives her medications, she has difficulty picking them up. The nursing assistant remarks that taking her pulse is difficult because it is so slow and irregular. When you assess her, she is so weak that she is unable to lift her arm for a blood pressure check. Her blood pressure is 92/50, which is down from the 128/84 reading on admission. You also verify that her heart beat is slow and irregular.
1. What other assessment data should you obtain immediately and why?
Listen to her apical pulse to assess the true heart rate. With some dysrhythmias, especially if she is having premature contraction, the radial pulse can be very different from the apical pulse. Assess her oxygen saturation to determine whether cardiac function is adequate for the moment or whether the rapid response team is needed now. (Cardiac arrest is possible because of hyperkalemia.) Perform a finger stick blood glucose analysis immediately to determine whether she is hypoglycemic.
10. After teaching a client who has an implantable cardioverter-defibrillator (ICD), a nurse assesses the client's understanding. Which statement by the client indicates a correct understanding of the teaching?
a. "I should wear a snug-fitting shirt over the ICD."
b. "I will avoid sources of strong electromagnetic fields."
c. "I should participate in a strenuous exercise program."
d. "Now I can discontinue my antidysrhythmic medication."
ANS: B
The client being discharged with an ICD is instructed to avoid strong sources of electromagnetic fields. Clients should avoid tight clothing, which could cause irritation over the ICD generator. The client should be encouraged to exercise but should not engage in strenuous activities that cause the heart rate to meet or exceed the ICD cutoff point because the ICD can discharge inappropriately. The client should continue all prescribed medications.
DIF: Applying/Application REF: 674
KEY: Cardiac electrical conduction MSC: Integrated Process: Teaching/Learning NOT: Client Needs Category: Health Promotion and Maintenance
13. The nurse is caring for a patient with acute leukemia. Which signs/symptoms is the nurse most likely to observe during the assessment? (Select all that apply.)
a. Hematuria
b. Orthostatic hypotension
c. Bone pain
d. Joint swelling
e. Fatigue
f. Weight gain
a, b, c, d, e
The community health nurse is planning tuberculosis treatment for a client who is homeless and heroin-addicted. Which action will be most effective in ensuring that the client completes treatment?
A) Arrange for a health care worker to watch the client take the medication.
B) Give the client written instructions about how to take prescribed medications.
C) Have the client repeat medication names and side effects.
D) Instruct the client about the possible consequences of nonadherence.
(Chp. 31; elsevier resources)
A) Arrange for a health care worker to watch the client take the medication.
(Chp. 31; elsevier resources)
7. In which patient circumstance would the nurse question the order for the insertion of an indwelling catheter?
a. Patient is critically ill and at risk for hypovolemic shock.
b. Patient has urinary retention with beginnings of hydronephrosis.
c. Patient was in a card accident and has a possible spinal cord injury.
d. Patient has functional incontinence related to Alzheimer's disease.
d
8. A client in the intensive care unit is started on continuous venovenous hemofiltration (CVVH). Which finding is the cause of immediate action by the nurse?
a. Blood pressure of 76/58 mm Hg
b. Sodium level of 138 mEq/L
c. Potassium level of 5.5 mEq/L d. Pulse rate of 90 beats/min
ANS: A
Hypotension can be a problem with CVVH if replacement fluid does not provide enough volume to maintain blood pressure. The specially trained nurse needs to monitor for ongoing fluid and electrolyte replacement. The sodium level is normal and the potassium level is slightly elevated, which could be normal findings for someone with acute kidney injury. A pulse rate of 90 beats/min is normal.
14. A nurse cares for a client who has a chest tube. When would this client be at highest risk for developing a pneumothorax?
a. When the insertion site becomes red and warm to the touch
b. When the tube drainage decreases and becomes sanguineous
c. When the client experiences pain at the insertion site
d. When the tube becomes disconnected from the drainage system
ANS: D
Intrathoracic pressures are less than atmospheric pressures; therefore, if the chest tube becomes disconnected from the drainage system, air can be sucked into the pleural space and cause a pneumothorax. A red, warm, and painful insertion site does not increase the client's risk for a pneumothorax. Tube drainage should decrease and become serous as the client heals. Sanguineous drainage is a sign of bleeding but does not increase the client's risk for a pneumothorax.
DIF: Applying/Application REF: 578
KEY: Drain| respiratory distress/failure
MSC: Integrated Process: Nursing Process: Implementation
NOT: Client Needs Category: Physiological Integrity: Reduction of Risk Potential
The patient reports a history of splenectomy. Based on this information, what is the nurse most likely to assess for?
a. Signs of bleeding
b. Signs of infection
c. Digestive problems
d. Jaundice of the skin
b
10. A nurse teaches a client who has a history of heart failure. Which statement should the nurse include in this client's discharge teaching?
a. "Avoid drinking more than 3 quarts of liquids each day."
b. "Eat six small meals daily instead of three larger meals."
c. "When you feel short of breath, take an additional diuretic."
d. "Weigh yourself daily while wearing the same amount of clothing."
ANS: D
Clients with heart failure are instructed to weigh themselves daily to detect worsening heart failure early, and thus avoid complications. Other signs of worsening heart failure include increasing dyspnea, exercise intolerance, cold symptoms, and nocturia. Fluid overload increases symptoms of heart failure. The client should be taught to eat a heart-healthy diet, balance intake and output to prevent dehydration and overload, and take medications as prescribed. The most important discharge teaching is daily weights as this provides the best data related to fluid retention.
DIF: Applying/Application REF: 687
KEY: Heart failure| patient education MSC: Integrated Process: Teaching/Learning
NOT: Client Needs Category: Health Promotion and Maintenance
A nurse cares for a client who has a family history of diabetes mellitus. The client states, "My father has type 1 diabetes mellitus. Will I develop this disease as well?" How should the nurse respond?
a. "Your risk of diabetes is higher than the general population, but it may not occur."
b. "No genetic risk is associated with the development of type 1 diabetes mellitus."
c. "The risk for becoming a diabetic is 50% because of how it is inherited."
d. "Female children do not inherit diabetes mellitus, but male children will."
ANS: A
Risk for type 1 diabetes is determined by inheritance of genes coding for HLA-DR and HLA-DQ tissue types. Clients who have one parent with type 1 diabetes are at increased risk for its development. Diabetes (type 1) seems to require interaction between inherited risk and environmental factors, so not everyone with these genes develops diabetes. The other statements are not accurate.
8. A nurse plans care for a client with overflow incontinence. Which intervention should the nurse include in this clients plan of care to assist with elimination?
a. Stroke the medial aspect of the thigh.
b. Use intermittent catheterization.
c. Provide digital anal stimulation.
d. Use the Valsalva maneuver.
D
In clients with overflow incontinence, the voiding reflex arc is not intact. Mechanical pressure, such as that achieved through the Valsalva maneuver (holding the breath and bearing down as if to defecate), can initiate voiding. Stroking the medial aspect of the thigh or providing digital anal stimulation requires the reflex arc to be intact to initiate elimination. Due to the high risk for infection, intermittent catheterization should only be implemented when other interventions are not successful.
71. The client diagnosed with cancer of the larynx
has had four (4) weeks of radiation therapy to the
neck. The client is complaining of severe pain
when swallowing. Which scientific rationale
explains the pain?
1. The cancer has grown to obstruct the
esophagus.
2. The treatments are working on the cancer
and the throat is edematous.
3. Cancers are painful and this is expected.
4. The treatments are also affecting the
esophagus, causing ulcerations.
1. This is an example of nonmalfeasance,
where the nurse "does no harm." In
attempting
to discuss the client's refusal,
the nurse is not trying to influence the
client; the nurse is merely attempting to
listen therapeutically.
TEST-TAKING HINT: If the test taker were not aware of the terms of ethical principles, then
dissecting the word "nonmalfeasance" into
its portions might help. Non- means "nothing"
or "none," and mal- means "bad," so "no
bad action"
could be inferred. This would
eliminate option "2."
A nurse works in a gerontology clinic. What age-related changes cause the nurse to alter standard assessment techniques from those used for younger adults? (Select all that apply.)
a.
Dentition deteriorates with more cavities.
b.
Nail beds may be thickened or discolored.
c.
Progressive loss of hair occurs with age.
d.
Sclerae begin to turn yellow or pale.
e.
Skin becomes dry as the client ages.
b.
Nail beds may be thickened or discolored.
c.
Progressive loss of hair occurs with age.
e.
Skin becomes dry as the client ages.
ANS: B, C, E
Common findings in older adults include thickened or discolored nail beds, dry skin, and thinning hair. The nurse adapts to these changes by altering assessment techniques. Having more dental caries and changes in the sclerae are not normal age-related changes.
10. A nurse assesses a client who is scheduled for a cardiac catheterization. Which assessment should the nurse complete prior to this procedure?
a. Clients level of anxiety
b. Ability to turn self in bed
c. Cardiac rhythm and heart rate
d. Allergies to iodine-based agents
D
Before the procedure, the nurse should ascertain whether the client has an allergy to iodine-containing preparations, such as seafood or local anesthetics. The contrast medium used during the procedure is iodine based. This allergy can cause a life-threatening reaction, so it is a high priority. Second, it is important for the nurse to assess anxiety, mobility, and baseline cardiac status.
Ch.61 Question 5 of 16
To best determine how well a client with diabetes mellitus is controlling blood glucose, which test does the nurse monitor?
a. Fasting blood glucose
b. Glycosylated hemoglobin (HbA1c)
c. Oral glucose tolerance test
d. Urine glucose level
b
Glycosylated hemoglobin (HbA1c)
Glycosylated hemoglobin indicates the average blood glucose over several months and is the best indicator of overall blood glucose control. Fasting blood glucose can be used to monitor glucose control, but it is not the best method (although this may be the method that clients are most familiar with). Oral glucose testing and urine glucose levels look at one period of time and are not the best methods to look at overall effectiveness of treatment.
5. Which factor is a main assessment finding that signifies hypothyroidism?
a. Irritability
b. Cold intolerance
c. Diarrhea
d. Fatigue
b
11. A nurse cares for a client who is prescribed magnetic resonance imaging (MRI) of the heart. The clients health history includes a previous myocardial infarction and pacemaker implantation. Which action should the nurse take?
a. Schedule an electrocardiogram just before the MRI.
b. Notify the health care provider before scheduling the MRI.
c. Call the physician and request a laboratory draw for cardiac enzymes.
d. Instruct the client to increase fluid intake the day before the MRI.
B
The magnetic fields of the MRI can deactivate the pacemaker. The nurse should call the health care provider and report that the client has a pacemaker so the provider can order other diagnostic tests. The client does not need an electrocardiogram, cardiac enzymes, or increased fluids.
11. While assessing a client who has facial trauma, the nurse auscultates stridor. The client is anxious and restless. Which action should the nurse take first?
a. Contact the provider and prepare for intubation.
b. Administer prescribed albuterol nebulizer therapy.
c. Place the client in high-Fowler's position.
d. Ask the client to perform deep-breathing exercises.
ANS: A
Facial and neck tissue edema can occur in clients with facial trauma. Airway patency is the highest priority. Clients who experience stridor and hypoxia, manifested by anxiety and restlessness, should be immediately intubated to ensure airway patency. Albuterol decreases bronchi and bronchiole inflammation, not facial and neck edema. Although putting the client in high-Fowler's position and asking the client to perform breathing exercises may temporarily improve the client's comfort, these actions will not decrease the underlying problem or improve airway patency.
DIF: Applying/Application REF: 534 KEY: Trauma
MSC: Integrated Process: Nursing Process: Implementation
NOT: Client Needs Category: Physiological Integrity: Physiological Adaptation
The patient is admitted for a chronic liver disorder and will be receiving vitamin K to address one of the problems associated with the disorder. Which clinical manifestation is the nurse most likely to observe before the vitamin K therapy is initiated?
a. Sore throat and a smooth tongue
b. Bruising and bleeding at venipuncture sites
c. Fever and increase in WBC count
d. Calf swelling due to deep vein thrombosis
b
8. An emergency department nurse assesses a client with kidney trauma and notes that the client's abdomen is tender and distended and blood is visible at the urinary meatus. Which prescription should the nurse consult the provider about before implementation?
a. Assessing vital signs every 15 minutes
b. Inserting an indwelling urinary catheter
c. Administering intravenous fluids at 125 mL/hr d. Typing and crossmatching for blood products
ANS: B
Clients with blood at the urinary meatus should not have a urinary catheter inserted via the urethra before additional diagnostic studies are done. The urethra could be torn. The nurse should question the provider about the need for a catheter; if one is needed, the provider can insert a suprapubic catheter. The nurse should monitor the client's vital signs closely, send blood for type and crossmatch in case the client needs blood products, and administer intravenous fluids.
8. A nurse plans care for a client with overflow incontinence. Which intervention should the nurse include in this client's plan of care to assist with elimination?
a. Stroke the medial aspect of the thigh.
b. Use intermittent catheterization.
c. Provide digital anal stimulation. d. Use the Valsalva maneuver.
ANS: D
In clients with overflow incontinence, the voiding reflex arc is not intact. Mechanical pressure, such as that achieved through the Valsalva maneuver (holding the breath and bearing down as if to defecate), can initiate voiding. Stroking the medial aspect of the thigh or providing digital anal stimulation requires the reflex arc to be intact to initiate elimination. Due to the high risk for infection, intermittent catheterization should only be implemented when other interventions are not successful.
12. A nurse is caring for a client who has sleep apnea and is prescribed modafinil (Provigil). The client asks, "How will this medication help me?" How should the nurse respond?
a. "This medication will treat your sleep apnea."
b. "This sedative will help you to sleep at night."
c. "This medication will promote daytime wakefulness."
d. "This analgesic will increase comfort while you sleep."
ANS: C
Modafinil is helpful for clients who have narcolepsy (uncontrollable daytime sleep) related to sleep apnea. This medication promotes daytime wakefulness.
DIF: Remembering/Knowledge REF: 535 KEY: Medication MSC: IntegratedProcess:Teaching/Learning
NOT: Client Needs Category: Physiological Integrity: Pharmacological and Parenteral Therapies
14. In caring for a patient with acute leukemia, what is the priority collaborative problem?
a. Protecting the patient from infection
b. Minimizing the side effects of chemotherapy
c. Controlling the patient's pain
d. Assisting the patient to cope with fatigue
a
11. A nurse admits a client who is experiencing an exacerbation of heart failure. Which action should the nurse take first?
a. Assess the client's respiratory status.
b. Draw blood to assess the client's serum electrolytes.
c. Administer intravenous furosemide (Lasix).
d. Ask the client about current medications.
ANS: A
Assessment of respiratory and oxygenation status is the priority nursing intervention for the prevention of complications. Monitoring electrolytes, administering diuretics, and asking about current medications are important but do not take priority over assessing respiratory status.
DIF: Applying/Application REF: 687
KEY: Heart failure| respiratory distress/failure| assessment/diagnostic examination
MSC: Integrated Process: Nursing Process: Implementation
NOT: Client Needs Category: Safe and Effective Care Environment: Management of Care
5. A nurse cares for a client who presents with bradycardia secondary to hypothyroidism. Which medication should the nurse anticipate being prescribed to the client?
a. Atropine sulfate
b. Levothyroxine sodium (Synthroid)
c. Propranolol (Inderal)
d. Epinephrine (Adrenalin)
b. The treatment for bradycardia from hypothyroidism is to treat the hypothyroidism using levothyroxine sodium. If the heart rate were so slow that it became an emergency, then atropine or epinephrine might be an option for short-term management. Propranolol is a beta blocker and would be contraindicated for a client with bradycardia.
8. Damage to which renal structure or tissues can change the actual production of urine?
a. Kidney parenchyma
b. Convoluted tubules
c. Calyces
d. Ureters
b
72. The client who has undergone a radical neck
dissection and tracheostomy for cancer of the
larynx is being discharged. Which discharge
instructions should the nurse teach? Select all
that apply.
1. The client will be able to speak again after the
surgery area has healed.
2. The client should wear a protective covering
over the stoma when showering.
3. The client should clean the stoma and then
apply a petroleum-based ointment.
4. The client should use a humidifier in the
room.
5. The client can get a special telephone for
communication.
4. The esophagus is extremely radiosensitive,
and esophageal ulcerations are common.
The pain can become so severe the
client cannot swallow saliva. This is a
situation in which the client will be admitted
to the hospital for IV narcotic pain
medication and possibly total parenteral
nutrition.
TEST-TAKING HINT: The test taker must remember
not to jump to conclusions and to
realize what a word is actually saying. Swallowing
is an action involving the esophagus,
so the best choice would be either option
"1" or option "4," both of which contain the
word "esophagus."
p. 818, Prioritization, Delegation, and Supervision
The patient is a 52-year-old woman who has undergone an autologous stem cell transplantation for non-Hodgkin's lymphoma. She is recovering, and her white blood cell count is improving but still very low. She remains on neutropenic precautions. The LPN reports that the patient's heart rate, respiratory rate, temperature, and blood pressure are all elevated.
2. You must assign an unlicensed assistive personnel (UAP) to help care for this patient. Of the
four UAPs available, one is newly pregnant and has worked on this unit for 3 years, one has had cold symptoms for 3 days, one has not yet cared for a patient on neutropenic precautions, and one has a fear of people with cancer. Which UAP should you avoid assigning to this patient? Provide a rationale for your choice.
Assign the UAP who is newly pregnant and very experienced to assist with this patient. The patient's possible infection is not harmful to the pregnant UAP because the infection is most likely from overgrowth of the patient's normal flora. Infection in a patient with neutropenia can lead to sepsis and death very quickly. Therefore, an experienced and healthy caregiver is critical. The UAP who fears people with cancer may let his or her fear overshadow the ability to provide appropriate care. If this person fears cancer, he or she may also fear infection. In fact, it may be appropriate to request that this UAP be transferred to a unit that does not include people with cancer in its usual population.
11. A nurse cares for a client with atrial fibrillation who reports fatigue when completing activities of daily living. What interventions should the nurse implement to address this client's concerns?
a. Administer oxygen therapy at 2 liters per nasal cannula.
b. Provide the client with a sleeping pill to stimulate rest.
c. Schedule periods of exercise and rest during the day.
d. Ask unlicensed assistive personnel to help bathe the client.
ANS: C
Clients who have atrial fibrillation are at risk for decreased cardiac output and fatigue when completing activities of daily living. The nurse should schedule periods of exercise and rest during the day to decrease fatigue. The other interventions will not assist the client with self-care activities.
DIF: Applying/Application REF: 658
KEY: Cardiac electrical conduction MSC: Integrated Process: Teaching/Learning NOT: Client Needs Category: Physiological Integrity: Basic Care and Comfort
A nurse prepares to palpate a client's thyroid gland. Which action should the nurse take when performing this assessment?
a. Stand in front of the client instead of behind the client.
b. Ask the client to swallow after palpating the thyroid.
c. Palpate the right lobe with the nurse's left hand.
d. Place the client in a sitting position with the chin tucked down.
ANS: D
The client should be in a sitting position with the chin tucked down as the examiner stands behind the client. The nurse feels for the thyroid isthmus while the client swallows and turns the head to the right, and the nurse palpates the right lobe with the right hand. The technique is repeated in the opposite fashion for the left lobe.
A client undergoing coronary artery bypass grafting asks why the surgeon has chosen to use the internal mammary artery for the surgery. Which response by the nurse is correct?
a. "This way you will not need to have a leg incision."
b. "The surgeon prefers this approach because it is easier."
c. "These arteries remain open longer."
d. "The surgeon has chosen this approach because of your age."
c. "These arteries remain open longer."
Mammary arteries remain patent much longer than other grafts. Although no leg incision will be made with this approach, veins from the legs do not remain patent as long as the mammary artery graft does. Long-term patency, not ease of the procedure, is the primary concern. Age is not a determining factor in selection of these grafts.
Question 3 of 16
The nurse is teaching a client about maintaining a proper diet to prevent an endocrine disorder. Which food does the nurse suggest after the client indicates a dislike of fish?
a. Iodized salt for cooking
b. More red meat
c. More green vegetables
d. Salt substitute for cooking
a
Iodized salt for cooking
Dietary deficiencies in iodide-containing foods may be a cause of an endocrine disorder. For clients who do not eat saltwater fish on a regular basis, teach them to use iodized salt in food preparation. The client should eat a well-balanced diet that includes less animal fat. Eating vegetables contributes to a proper diet; however, this does not prevent an endocrine disorder. Using a salt substitute does not prevent an endocrine disorder; in addition, salt substitutes may contain high levels of potassium, which may lead to electrolyte imbalances.
A client has been admitted after sustaining a humerus fracture that occurred when picking up the family cat. What test result would the nurse correlate to this condition?
a. Bence-Jones protein in urine
b. Epstein-Barr virus: positive
c. Hemoglobin: 18 mg/dL
d. Red blood cell count: 8.2/mm3
a. Bence-Jones protein in urine
15. A nurse cares for a client with a 40-year smoking history who is experiencing distended neck veins and dependent edema. Which physiologic process should the nurse correlate with this client's history and clinical manifestations?
a. Increased pulmonary pressure creating a higher workload on the right side of the
heart
b. Exposure to irritants resulting in increased inflammation of the bronchi and
bronchioles
c. Increased number and size of mucus glands producing large amounts of thick
mucus
d. Left ventricular hypertrophy creating a decrease in cardiac output
ANS: A
Smoking increases pulmonary hypertension, resulting in cor pulmonale, or right-sided heart failure. Increased pressures in the lungs make it more difficult for blood to flow through the lungs. Blood backs up into the right side of the heart and then into the peripheral venous system, creating distended neck veins and dependent edema. Inflammation in bronchi and bronchioles creates an airway obstruction which manifests as wheezes. Thick mucus in the lungs has no impact on distended neck veins and edema. Left ventricular hypertrophy is associated with left heart failure and is not caused by a 40-year smoking history.
DIF: Remembering/Knowledge REF: 58
KEY: Heart failure| cor pulmonale
MSC: Integrated Process: Nursing Process: Implementation
NOT: Client Needs Category: Physiological Integrity: Physiological Adaptation
8. The nurse is talking to a group of healthy young college students about maintaining good kidney health and preventing AKI. Which health promotion point is the nurse most likely to emphasize with this group?
a. "Have your blood pressure checked regularly."
b. "Find out if you have a family history of diabetes."
c. "Avoid dehydration by drinking at least 2 to 3 L of water daily."
d. "Have annual testing for microalbuminuria and urine protein."
c
8. The nurse is developing a teaching plan for a patient with PKD. Which topics does the nurse include? (Select all that apply.)
a. Teach how to measure and record blood pressure.
b. Assist to develop a schedule for self-monitoring drugs.
c. Instruct to take and record weight twice a month.
d. Explain the potential side effects of the drugs.
e. Review high-protein, low-fat diet plan.
a, b, d
12. A nurse is caring for a client on mechanical ventilation and finds the client agitated and thrashing about. What action by the nurse is most appropriate?
a. Assess the cause of the agitation.
b. Reassure the client that he or she is safe.
c. Restrain the client's hands.
d. Sedate the client immediately.
ANS: A
The nurse needs to determine the cause of the agitation. The inability to communicate often makes clients anxious, even to the point of panic. Pain and confusion can also cause agitation. Once the nurse determines the cause of the agitation, he or she can implement measures to relieve the underlying cause. Reassurance is also important but may not address the etiology of the agitation. Restraints and more sedation may be necessary, but not as a first step.
A client with multiple myeloma demonstrates worsening bone density on diagnostic scans. About what drug does the nurse plan to teach this client?
a. Bortezomib (Velcade)
b. Dexamethasone (Decadron)
c. Thalidomide (Thalomid)
d. Zoledronic acid (Zometa)
d. Zoledronic acid (Zometa)
A client undergoing coronary artery bypass grafting asks why the surgeon has chosen to use the internal mammary artery for the surgery. Which response by the nurse is correct?
"This way you will not need to have a leg incision."
"The surgeon prefers this approach because it is easier."
"These arteries remain open longer."
"The surgeon has chosen this approach because of your age."
"These arteries remain open longer."
Mammary arteries remain patent much longer than other grafts. Although no leg incision will be made with this approach, veins from the legs do not remain patent as long as the mammary artery graft does. Long-term patency, not ease of the procedure, is the primary concern. Age is not a determining factor in selection of these grafts.
The RN has just received change-of-shift report on the medical-surgical unit. Which client will need to be assessed first?
A. Client with Hashimoto's thyroiditis and a large goiter
B. Client with hypothyroidism and an apical pulse of 51 beats/min
C. Client with parathyroid adenoma and flank pain due to a kidney stone
D. Client who had a parathyroidectomy yesterday and has muscle twitching
D.
A client who is 1 day postoperative for parathyroidectomy and has muscle twitching is showing signs of hypocalcemia and is at risk for seizures. Rapid assessment and intervention are needed. Clients with Hashimoto's thyroiditis are usually stable; this client does not need to be assessed first. Although an apical pulse of 51 is considered bradycardia, a low heart rate is a symptom of hypothyroidism. A client with a kidney stone will be uncomfortable and should be asked about pain medication as soon as possible, but this client does not need to be assessed first.
8. A nurse reviews laboratory results for a client who was admitted for a myocardial infarction and cardiogenic shock 2 days ago. Which laboratory test result should the nurse expect to find?
a. Blood urea nitrogen (BUN) of 52 mg/dL
b. Creatinine of 2.3 mg/dL
c. BUN of 10 mg/dL
d. BUN/creatinine ratio of 8:1
ANS: A
Shock leads to decreased renal perfusion. An elevated BUN accompanies this condition. The creatinine should be normal because no kidney damage occurred. A low BUN signifies overhydration, malnutrition, or liver damage. A low BUN/creatinine ratio indicates fluid volume excess or acute renal tubular acidosis.
12. A nurse has educated a client on isoniazid (INH). What statement by the client indicates teaching has been effective?
a. "I need to take extra vitamin C while on INH."
b. "I should take this medicine with milk or juice."
c. "I will take this medication on an empty stomach." d. "My contact lenses will be permanently stained."
ANS: C
INH needs to be taken on an empty stomach, either 1 hour before or 2 hours after meals. Extra vitamin B needs to be taken while on the drug. Staining of contact lenses commonly occurs while taking rifampin (Rifadin).
p. 818, Prioritization, Delegation, and Supervision
The patient is a 52-year-old woman who has undergone an autologous stem cell transplantation for non-Hodgkin's lymphoma. She is recovering, and her white blood cell count is improving but still very low. She remains on neutropenic precautions. The LPN reports that the patient's heart rate, respiratory rate, temperature, and blood pressure are all elevated.
3. A nursing student tearfully reports to you, "I took some flowers into the patient's room to
cheer him up, and he told me that he didn't think he was supposed to have flowers. I took them out of the room right away, and then I realized I had made a mistake." How should you respond to this student?
Acknowledge the student for taking responsibility for the error. Helping the student to feel comfortable in reporting errors rather than hiding mistakes is essential for patient safety. Then go with the student to explain the situation to the nursing instructor so that the student can be counseled and procedures reviewed. Suggest to the instructor that the incident be used as a "teachable moment" and carefully shared with the other students. Suggest that the students discuss some of the ways patients with neutropenia can be protected from infections during hospitalization.
11. An older adult is on cardiac monitoring after a myocardial infarction. The client shows frequent dysrhythmias. What action by the nurse is most appropriate?
a. Assess for any hemodynamic effects of the rhythm.
b. Prepare to administer antidysrhythmic medication.
c. Notify the provider or call the Rapid Response Team.
d. Turn the alarms off on the cardiac monitor.
ANS: A
Older clients may have dysrhythmias due to age-related changes in the cardiac conduction system. They may have no significant hemodynamic effects from these changes. The nurse should first assess for the effects of the dysrhythmia before proceeding further. The alarms on a cardiac monitor should never be shut off. The other two actions may or may not be needed.
DIF: Applying/Application REF: 769
KEY: Coronary artery disease| older adult| pathophysiology| nursing assessment MSC: IntegratedProcess:NursingProcess:Assessment
NOT: Client Needs Category: Health Promotion and Maintenance
A nurse collaborates with an unlicensed assistive personnel (UAP) to provide care for a client who is prescribed a 24-hour urine specimen collection. Which statement should the nurse include when delegating this activity to the UAP?
a. "Note the time of the client's first void and collect urine for 24 hours."
b. "Add the preservative to the container at the end of the test."
c. "Start the collection by saving the first urine of the morning."
d. "It is okay if one urine sample during the 24 hours is not collected."
ANS: A
The collection of a 24-hour urine specimen is often delegated to a UAP. The nurse must ensure that the UAP understands the proper process for collecting the urine. The 24-hour urine collection specimen is started after the client's first urination. The first urine specimen is discarded because there is no way to know how long it has been in the bladder, but the time of the client's first void is noted. The client adds all urine voided after that first discarded specimen during the next 24 hours. When the 24-hour mark is reached, the client voids one last time and adds this specimen to the collection. The preservative, if used, must be added to the container at the beginning of the collection. All urine samples need to be collected for the test results to be accurate.
A client has just returned from coronary artery bypass graft surgery. For which finding does the nurse contact the surgeon?
Temperature 98.2° F
Chest tube drainage 175 mL last hour
Serum potassium 3.9 mEq/L
Incisional pain 6 on a scale of 0 to 10
Chest tube drainage 175 mL last hour
Some bleeding is expected after surgery; however, the nurse should report chest drainage over 150 mL/hr to the surgeon. Although hypothermia is a common problem after surgery, a temperature of 98.2° F is a normal finding. Serum potassium of 3.9 mEq/L is a normal finding. Incisional pain of 6 on a scale of 0 to 10 is expected immediately after major surgery; the nurse should administer prescribed analgesics.
After receiving change-of-shift report about these four clients, which client does the nurse attend to first?
A. Client with acute adrenal insufficiency who has a blood glucose of 36 mg/dL
B. Client with diabetes insipidus who has a dose of desmopressin (DDAVP) due
C. Client with hyperaldosteronism who has a serum potassium of 3.4 mEq/L
D. Client with pituitary adenoma who is reporting a severe headache
A.
A glucose level of 36 mg/dL is considered an emergency; this client must be assessed and treated immediately. Although it is important to maintain medications on schedule, the client requiring a dose of desmopressin is not the first client who needs to be seen. A serum potassium of 3.4 mEq/L in the client with hyperaldosteronism may be considered normal (or slightly hypokalemic), based on specific hospital levels. The client reporting a severe headache needs to be evaluated as soon as possible after the client with acute adrenal insufficiency. As an initial measure, the RN could delegate obtaining vital signs to unlicensed assistive personnel.
8. For a patient who needs an indwelling catheter for at least 2 weeks, which intervention would help reduce the bacterial colonization along the catheter?
a. Secure the catheter to the female patient's thigh.
b. Consider the use of a coated catheter.
c. Wash the urine bag and outflow tube every day.
d. Apply antiseptic ointment to the catheter tubing.
b
15. The nurse is helping a patient prepare for induction therapy for acute leukemia. What information will the nurse give to the patient?
a. A donor is needed for hematopoietic stem cell transplantation.
b. Prolonged hospitalization is common to protect against infection.
c. The therapy may last from months to years to maintain remission.
d. Success of the therapy results in remission and the intent is to cure.
b
An older client presents to the emergency department with a 2-day history of cough, pain on inspiration, shortness of breath, and dyspnea. The client never had a pneumococcal vaccine. The client's chest x-ray shows density in both bases. The client has wheezing upon auscultation of both lungs. Would a bronchodilator be beneficial for this client?
A) It would not be beneficial for this client.
B) It would help decrease the bronchospasm.
C) It would clear up the density in the bases of the client's lungs.
D) It would decrease the client's pain on inspiration.
(Chp. 31; elsevier resources)
B) It would help decrease the bronchospasm.
(Chp. 31; elsevier resources)
9. The nurse is caring for four clients with chronic kidney disease. Which client should the nurse assess first upon initial rounding?
a. Woman with a blood pressure of 158/90 mm Hg
b. Client with Kussmaul respirations
c. Man with skin itching from head to toe d. Client with halitosis and stomatitis
ANS: B
Kussmaul respirations indicate a worsening of chronic kidney disease (CKD). The client is increasing the rate and depth of breathing to excrete carbon dioxide through the lungs. Hypertension is common in most clients with CKD, and skin itching increases with calcium-phosphate imbalances, another common finding in CKD. Uremia from CKD causes ammonia to be formed, resulting in the common findings of halitosis and stomatitis.
13. A nurse cares for a client who has packing inserted for posterior nasal bleeding. Which action should the nurse take first?
a. Assess the client's pain level.
b. Keep the client's head elevated.
c. Teach the client about the causes of nasal bleeding.
d. Make sure the string is taped to the client's cheek.
ANS: D
The string should be attached to the client's cheek to hold the packing in place. The nurse needs to make sure that this does not move because it can occlude the client's airway. The other options are good interventions, but ensuring that the airway is patent is the priority objective.
DIF: Applying/Application REF: 533 KEY: Surgical care MSC: Integrated Process: Nursing Process: Implementation
NOT: Client Needs Category: Safe and Effective Care Environment: Management of Care
Venous stasis is considered an intrinsic factor that could result in activating which physiologic process?
a. Increased RBC production
b. Adjustment of osmotic fluid pressure
c. Initiation of anticlotting forces
d. Initiation of blood clotting cascade
d
A client has just returned from coronary artery bypass graft surgery. For which finding does the nurse contact the surgeon?
a. Temperature 98.2° F
b. Chest tube drainage 175 mL last hour
c. Serum potassium 3.9 mEq/L
d. Incisional pain 6 on a scale of 0 to 10
b. Chest tube drainage 175 mL last hour
Some bleeding is expected after surgery; however, the nurse should report chest drainage over 150 mL/hr to the surgeon. Although hypothermia is a common problem after surgery, a temperature of 98.2° F is a normal finding. Serum potassium of 3.9 mEq/L is a normal finding. Incisional pain of 6 on a scale of 0 to 10 is expected immediately after major surgery; the nurse should administer prescribed analgesics.
6. A patient is prescribed bromocriptine mesylate (Parlodel). Which information does the nurse teach the patient? (Select all that apply.)
a. Get up slowly from a lying position.
b. Take medication on an empty stomach.
c. Take daily for purposes of raising GH levels to reduce symptom of acromegaly.
d. Begin therapy with a maintenance level dose.
e. Report watery nasal discharge to the health care provider immediately.
a, e
Get up slowly from a lying position.
Report watery nasal discharge to the health care provider immediately.
9. A confused client with pneumonia is admitted with an indwelling catheter in place. During interdisciplinary rounds the following day, which question should the nurse ask the primary health care provider?
a. "Do you want daily weights on this client?"
b. "Will the client be able to return home?"
c. "Can we discontinue the indwelling catheter?" d. "Should we get another chest x-ray today?"
ANS: C
An indwelling catheter dramatically increases the risks of urinary tract infection and urosepsis. Nursing staff should ensure that catheters are left in place only as long as they are medically needed. The nurse should inquire about removing the catheter. All other questions might be appropriate, but because of client safety, this question takes priority.
16. A nurse cares for a client with chronic obstructive pulmonary disease (COPD) who appears thin and disheveled. Which question should the nurse ask first?
a. "Do you have a strong support system?"
b. "What do you understand about your disease?"
c. "Do you experience shortness of breath with basic activities?"
d. "What medications are you prescribed to take each day?"
ANS: C
Clients with severe COPD may not be able to perform daily activities, including bathing and eating, because of excessive shortness of breath. The nurse should ask the client if shortness of breath is interfering with basic activities. Although the nurse should know about the client's support systems, current knowledge, and medications, these questions do not address the client's appearance.
DIF: Applying/Application REF: 561 KEY: Functional ability MSC: IntegratedProcess:NursingProcess:Assessment
NOT: Client Needs Category: Physiological Integrity
A client with autoimmune idiopathic thrombocytopenic purpura (ITP) has had a splenectomy and returned to the surgical unit 2 hours ago. The nurse assesses the client and finds the abdominal dressing saturated with blood. What action is most important?
a. Preparing to administer a blood transfusion
b. Reinforcing the dressing and documenting findings
c. Removing the dressing and assessing the surgical site
d. Taking a set of vital signs and notifying the surgeon
d. Taking a set of vital signs and notifying the surgeon
12. A nurse assists with the cardioversion of a client experiencing acute atrial fibrillation. Which action should the nurse take prior to the initiation of cardioversion?
a. Administer intravenous adenosine.
b. Turn off oxygen therapy.
c. Ensure a tongue blade is available.
d. Position the client on the left side.
ANS: B
For safety during cardioversion, the nurse should turn off any oxygen therapy to prevent fire. The other interventions are not appropriate for a cardioversion. The client should be placed in a supine position.
DIF: Remembering/Knowledge REF: 668
KEY: Assessment/diagnostic examination| safety
MSC: Integrated Process: Nursing Process: Implementation
NOT: Client Needs Category: Safe and Effective Care Environment: Safety and Infection Control
6. A patient has a low serum cortisol level. Which hormone would the nurse expect to be secreted to correct this?
a. Thyroid-stimulating hormone (TSH)
b. Adrenocorticotropic hormone
c. Parathyroid hormone
d. Antidiuretic hormone
b
Adrenocorticotropic hormone
12. A nurse assesses a client with mitral valve stenosis. What clinical manifestation should alert the nurse to the possibility that the client's stenosis has progressed?
a. Oxygen saturation of 92%
b. Dyspnea on exertion
c. Muted systolic murmur
d. Upper extremity weakness
ANS: B
Dyspnea on exertion develops as the mitral valvular orifice narrows and pressure in the lungs increases. The other manifestations do not relate to the progression of mitral valve stenosis.
DIF: Applying/Application REF: 688
KEY: Valve disorder| respiratory distress/failure
MSC: IntegratedProcess:NursingProcess:Assessment
NOT: Client Needs Category: Physiological Integrity: Reduction of Risk Potential
A nurse assesses a client who is recovering from a transsphenoidal hypophysectomy. The nurse notes nuchal rigidity. Which action should the nurse take first?
a. Encourage range-of-motion exercises.
b. Document the finding and monitor the client.
c. Take vital signs, including temperature.
d. Assess pain and administer pain medication.
C (Nuchal rigidity is a major manifestation of meningitis, a potential postoperative complication associated with this surgery. Meningitis is an infection; usually the client will also have a fever and tachycardia. Range-of-motion exercises are inappropriate because meningitis is a possibility. Documentation should be done after all assessments are completed and should not be the only action. Although pain medication may be a palliative measure, it is not the most appropriate initial action.)
9. A confused client with pneumonia is admitted with an indwelling catheter in place. During interdisciplinary rounds the following day, which question should the nurse ask the primary health care provider?
a. Do you want daily weights on this client?
b. Will the client be able to return home?
c. Can we discontinue the indwelling catheter?
d. Should we get another chest x-ray today?
C
An indwelling catheter dramatically increases the risks of urinary tract infection and urosepsis. Nursing staff should ensure that catheters are left in place only as long as they are medically needed. The nurse should inquire about removing the catheter. All other questions might be appropriate, but because of client safety, this question takes priority.
p. 818, Prioritization, Delegation, and Supervision
The patient is a 52-year-old woman who has undergone an autologous stem cell transplantation for non-Hodgkin's lymphoma. She is recovering, and her white blood cell count is improving but still very low. She remains on neutropenic precautions. The LPN reports that the patient's heart rate, respiratory rate, temperature, and blood pressure are all elevated.
4. The student asks you whether a book still wrapped in shrink wrap just now brought in by a
friend of the patient can be taken to the patient's room. How will you help the student know what to do in this situation?
A book in shrink wrap could have potential organisms on it. However, the shrink wrap is waterproof. Tell the student to cleanse the shrink wrap with an alcohol-based hand rub and then remove the shrink wrap before giving the book to the patient. Although the book and its jacket are not sterile, they most likely have been handled only by machines in the printing process and are unlikely to convey pathogenic microorganisms (or any other organisms) to the patient.
13. A nurse is preparing to admit a client on mechanical ventilation from the emergency department. What action by the nurse takes priority?
a. Assessing that the ventilator settings are correct
b. Ensuring there is a bag-valve-mask in the room
c. Obtaining personal protective equipment
d. Planning to suction the client upon arrival to the room
ANS: B
Having a bag-valve-mask device is critical in case the client needs manual breathing. The respiratory therapist is usually primarily responsible for setting up the ventilator, although the nurse should know and check the settings. Personal protective equipment is important, but ensuring client safety takes priority. The client may or may not need suctioning on arrival.
9. After teaching a client with hypertension secondary to renal disease, the nurse assesses the client's understanding. Which statement made by the client indicates a need for additional teaching?
a. "I can prevent more damage to my kidneys by managing my blood pressure."
b. "If I have increased urination at night, I need to drink less fluid during the day." c. "I need to see the registered dietitian to discuss limiting my protein intake."
d. "It is important that I take my antihypertensive medications as directed."
ANS: B
The client should not restrict fluids during the day due to increased urination at night. Clients with renal disease may be prescribed fluid restrictions. These clients should be assessed thoroughly for potential dehydration. Increased nocturnal voiding can be decreased by consuming fluids earlier in the day. Blood pressure control is needed to slow the progression of renal dysfunction. When dietary protein is restricted, refer the client to the registered dietitian as needed.
13. A client has been taking isoniazid (INH) for tuberculosis for 3 weeks. What laboratory results need to be reported to the health care provider immediately?
a. Albumin: 5.1 g/dL
b. Alanine aminotransferase (ALT): 180 U/L
c. Red blood cell (RBC) count: 5.2/mm3
d. White blood cell (WBC) count: 12,500/mm3
ANS: B
INH can cause liver damage, especially if the client drinks alcohol. The ALT (one of the liver enzymes) is extremely high and needs to be reported immediately. The albumin and RBCs are normal. The WBCs are slightly high, but that would be an expected finding in a client with an infection.
16. Which factors are associated with an increased risk for lymphoma? (Select all that apply.)
a. Immunosuppressive disorders
b. Chronic infection from Helicobacter pylori
c. Epstein-Barr viral infection
d. Chronic alcoholism
e. Pesticides and insecticides
a, b, c, e
12. A nurse assesses a client who is recovering from a myocardial infarction. The clients pulmonary artery pressure reading is 25/12 mm Hg. Which action should the nurse take first?
a. Compare the results with previous pulmonary artery pressure readings.
b. Increase the intravenous fluid rate because these readings are low.
c. Immediately notify the health care provider of the elevated pressures.
d. Document the finding in the clients chart as the only action.
A
Normal pulmonary artery pressures range from 15 to 26 mm Hg for systolic and from 5 to 15 mm Hg for diastolic. Although this clients readings are within normal limits, the nurse needs to assess any trends that may indicate a need for medical treatment to prevent complications. There is no need to increase intravenous fluids or notify the provider.
Which sign/symptom is one of the first indicators of hyperthyroidism that is often noticed by the patient?
a. Eyelid or globe lag
b. Vision changes or tiring of the eyes
c. Protruding eyes
d. Photophobia
b
A client has been diagnosed with hypothyroidism. What medication is usually prescribed to treat this disorder?
A. Atenolol (Tenormin)
B. Levothyroxine sodium (Synthroid)
C. Methimazole (Tapazole)
D. Propylthiouracil
B.
Levothyroxine is a synthetic form of thyroxine (T4) that is used to treat hypothyroidism. Atenolol is a beta blocker that is used to treat cardiovascular disease. Methimazole and propylthiouracil are used to treat hyperthyroidism.
12. The nurse is preparing to change a client's sternal dressing. What action by the nurse is most important?
a. Assess vital signs.
b. Don a mask and gown.
c. Gather needed supplies.
d. Perform hand hygiene.
ANS: D
To prevent a sternal wound infection, the nurse washes hands or performs hand hygiene as a priority. Vital signs do not necessarily need to be assessed beforehand. A mask and gown are not needed. The nurse should gather needed supplies, but this is not the priority.
DIF: Applying/Application REF: 776
KEY: Coronary artery disease| infection control| hand hygiene
MSC: Integrated Process: Nursing Process: Implementation
NOT: Client Needs Category: Safe and Effective Care Environment: Safety and Infection Control
A deficiency in any of the anticlotting factors, such as protein C, protein S, and antithrombin III increases the patient's risk for which disorder(s)?
(Select all that apply.)
a. Pulmonary embolism
b. Myocardial infarction
c. Sepsis
d. Pernicious anemia
e. Stroke
a, b, e
A client who has been homeless and has spent the past 6 months living in shelters has been diagnosed with confirmed tuberculosis (TB). Which medications does the nurse expect to be ordered for the client?
A) Isoniazid (INH), rifampin (Rifadin), pyrazinamide (Zinamide), ethambutol (Myambutol)
B) Metronidazole (Flagyl), acyclovir (Zovirax), flunisolide (AeroBid), rifampin (Rifadin)
C) Prednisone (Prednisone), guaifenesin (Organidin), ketorolac (Toradol), pyrazinamide (Zinamide)
D) Salmeterol (Serevent), cromolyn sodium (Intal), dexamethasone (Decadron), isoniazid (INH)
(Chp. 31; elsevier resources)
A) Isoniazid (INH), rifampin (Rifadin), pyrazinamide (Zinamide), ethambutol (Myambutol)
(Chp. 31; elsevier resources)
9. The nurse hears in report that the patient is being treated for a fungal UTI. In addition to performing routine care and assessments, the nurse is extra-vigilant for signs/symptoms of which systemic disorder that may underlie the fungal UTI?
a. Chronic cardiac disease
b. Immune system compromise
c. Respiratory system dysfunction
d. Connective tissue disorder
b
9. Which hematologic disorder is most likely to occur if the hormonal function of the kidneys is not working properly?
a. Leukemia
b. Thrombocytopenia
c. Neutropenia
d. Anemia
d
A client has a platelet count of 9000/mm3. The nurse finds the client confused and mumbling. What action takes priority?
a. Calling the Rapid Response Team
b. Delegating taking a set of vital signs
c. Instituting bleeding precautions
d. Placing the client on bedrest
a. Calling the Rapid Response Team
13. A client has an intra-arterial blood pressure monitoring line. The nurse notes bright red blood on the client's sheets. What action should the nurse perform first?
a. Assess the insertion site.
b. Change the client's sheets.
c. Put on a pair of gloves.
d. Assess blood pressure.
ANS: C
For the nurse's safety, he or she should put on a pair of gloves to prevent blood exposure. The other actions are appropriate as well, but first the nurse must don a pair of gloves.
DIF: Applying/Application REF: 771
KEY: Standard Precautions| infection control| intra-arterial blood pressure monitoring| staff safety MSC: Integrated Process: Nursing Process: Implementation
NOT: Client Needs Category: Safe and Effective Care Environment: Safety and Infection Control
A nurse teaches a client who is diagnosed with diabetes mellitus. Which statement should the nurse include in this client's plan of care to delay the onset of microvascular and macrovascular complications?
a. "Maintain tight glycemic control and prevent hyperglycemia."
b. "Restrict your fluid intake to no more than 2 liters a day."
c. "Prevent hypoglycemia by eating a bedtime snack."
d. "Limit your intake of protein to prevent ketoacidosis."
ANS: A
Hyperglycemia is a critical factor in the pathogenesis of long-term diabetic complications. Maintaining tight glycemic control will help delay the onset of complications. Restricting fluid intake is not part of the treatment plan for clients with diabetes. Preventing hypoglycemia and ketosis, although important, are not as important as maintaining daily glycemic control.
14. A client seen in the emergency department reports fever, fatigue, and dry cough but no other upper respiratory symptoms. A chest x-ray reveals mediastinal widening. What action by the nurse is best?
a. Collect a sputum sample for culture by deep suctioning.
b. Inform the client that antibiotics will be needed for 60 days. c. Place the client on Airborne Precautions immediately.
d. Tell the client that directly observed therapy is needed.
ANS: B
This client has manifestations of early inhalation anthrax. For treatment, after IV antibiotics are finished, oral antibiotics are continued for at least 60 days. Sputum cultures are not needed. Anthrax is not transmissible from person to person, so Standard Precautions are adequate. Directly observed therapy is often used for tuberculosis.
The visiting nurse is seeing a client postoperative for coronary artery bypass graft. Which nursing action should be performed first?
Assess coping skills.
Assess for postoperative pain at the client's incision site.
Monitor for dysrhythmias.
Monitor mental status.
Monitor for dysrhythmias.
Dysrhythmias are the leading cause of prehospital death; the nurse should monitor the client's heart rhythm. Assessing mental status, coping skills, or postoperative pain is not the priority for this client.
Ch.62 p. 1276, Patient-Centered Care; Quality Improvement; Safety
The patient is a 32-year-old woman admitted to your unit after surgery for fractures of the left arm and leg resulting from a car crash. She is awake and able to verify her medical history of rheumatoid arthritis and her usual daily medications. These are 10 mg of prednisone, naproxen 800 mg twice daily, oral contraceptives, calcium 600 mg, and one multiple vitamin tablet. All of these are prescribed for her to receive during her hospitalization. She is concerned about pain management and how long the recovery will be for the fractures. She is friendly, somewhat anxious, asks many questions, and wants to do "her part" to ensure good recovery. Over the next 4 days, she has become quieter, mumbles that her head and stomach hurt, and now does not recognize the assistant who has been providing her daily care. When she receives her medications, she has difficulty picking them up. The nursing assistant remarks that taking her pulse is difficult because it is so slow and irregular. When you assess her, she is so weak that she is unable to lift her arm for a blood pressure check. Her blood pressure is 92/50, which is down from the 128/84 reading on admission. You also verify that her heart beat is slow and irregular.
2. What is the most likely cause of the changes in this patient's physical and mental status?
The most likely cause is acute adrenal insufficiency as a result of increased cortisol needs related to the stress of surgery and injury. Because she has been on prednisone long term, she has some degree of adrenal suppression and cannot increase the extra cortisol needed during the additional stress. Although she is receiving 10 mg of prednisone daily, it is not enough for her current needs.
p. 826, Safe and Effective Care Environment
The nurse who just came on duty observes that the client, whose blood type is AB negative, is receiving a transfusion with type O negative packed red blood cells. What is the nurse's best first action?
A. Call the blood bank.
B. Take and record the client's vital signs.
C. Stop the transfusion and keep the IV open.
D. Document the observation as the only action.
Answer: B
Rationale: Clients with AB negative blood types can receive O negative blood because they do not have antibodies against this type of blood. Therefore, the transfusion does not need to be stopped nor does the blood bank need to be notified. The transfusion can proceed. Because the nurse is seeing the client for the first time since the transfusion was initiated, the client's vital signs need to be assessed rather than just documenting the observation.
A client is taking isoniazid, rifampin, pyrazinamide, and ethambutol for tuberculosis. The client calls to report visual changes, including blurred vision and reduced visual fields. Which medication may be causing these changes?
A) Ethambutol
B) Isoniazid
C) Pyrazinamide
D) Rifampin
(Chp. 31; elsevier resources)
A) Ethambutol
(Chp. 31; elsevier resources)
9. The nurse is caring for a patient who had hypovolemic shock secondary to trauma in the emergency department (ED) 2 days ago. Based on the pathophysiology of hypovolemia and prerenal azotemia, what does the nurse assess at least every hour?
a. Urinary output
b. Presence of edema
c. Urine color
d. Presence of pain
a
14. A nurse teaches a client to use a room humidifier after a laryngectomy. Which statement should the nurse include in this client's teaching?
a. "Add peppermint oil to the humidifier to relax the airway."
b. "Make sure you clean the humidifier to prevent infection."
c. "Keep the humidifier filled with water at all times."
d. "Use the humidifier when you sleep, even during daytime naps."
ANS: B
Priority teaching related to the use of a room humidifier focuses on infection control. Clients should be taught to meticulously clean the humidifier to prevent the spread of mold or other sources of infection. Peppermint oil should not be added to a humidifier. The humidifier should be refilled with water as needed and should be used while awake and asleep.
DIF: Understanding/Comprehension REF: 544 KEY: Surgical care
MSC: IntegratedProcess:Teaching/Learning
NOT: Client Needs Category: Safe and Effective Care Environment: Safety and Infection Control
MULTIPLE RESPONSE
17. Which disorder creates the highest risk for the patient to develop infection?
a. Sickle cell crisis
b. Vitamin B12 deficiency anemia
c. Polycythemia vera
d. Thrombocytopenia
a
The visiting nurse is seeing a client postoperative for coronary artery bypass graft. Which nursing action should be performed first?
a. Assess coping skills.
b. Assess for postoperative pain at the client's incision site.
c. Monitor for dysrhythmias.
d. Monitor mental status.
c. Monitor for dysrhythmias.
Dysrhythmias are the leading cause of prehospital death; the nurse should monitor the client's heart rhythm. Assessing mental status, coping skills, or postoperative pain is not the priority for this client.
Ch.61 Question 6 of 16
A client is hospitalized for pituitary function testing. Which nursing action included in the client's plan of care will be most appropriate for the RN to delegate to the LPN/LVN?
a. Assess the client for clinical manifestations of hypopituitarism.
b. Inject regular insulin for the growth hormone stimulation test.
c. Palpate the thyroid gland for size and firmness.
d. Teach the client about the adrenocorticotropic hormone stimulation test.
b
Inject regular insulin for the growth hormone stimulation test.
Injection of insulin is within the LPN/LVN scope of practice. Client assessment for clinical manifestations of hypopituitarism, palpating the thyroid gland, and client education are complex skills requiring training and expertise, and are best performed by an RN.
9. A patient with PKD reports sharp flank pain followed by blood in the urine. How does the nurse interpret these signs/symptoms?
a. Infection
b. Ruptured cyst
c. Increased kidney size
d. Ruptured renal artery aneurysm
b
9. A nurse cares for a client with a urine specific gravity of 1.018. Which action should the nurse take?
a. Evaluate the client's intake and output for the past 24 hours. b. Document the finding in the chart and continue to monitor. c. Obtain a specimen for a urine culture and sensitivity.
d. Encourage the client to drink more fluids, especially water.
ANS: B
This specific gravity is within the normal range for urine. There is no need to evaluate the client's intake and output, obtain a urine specimen, or increase fluid intake.
14. A client is on mechanical ventilation and the client's spouse wonders why ranitidine (Zantac) is needed since the client "only has lung problems." What response by the nurse is best?
a. "It will increase the motility of the gastrointestinal tract."
b. "It will keep the gastrointestinal tract functioning normally."
c. "It will prepare the gastrointestinal tract for enteral feedings."
d. "It will prevent ulcers from the stress of mechanical ventilation."
ANS: D
Stress ulcers occur in many clients who are receiving mechanical ventilation, and often prophylactic medications are used to prevent them. Frequently used medications include antacids, histamine blockers, and proton pump inhibitors. Zantac is a histamine blocking agent.
Severe anemia could cause enlargement of which organ?
a. Gallbladder
b. Kidneys
c. Colon
d. Liver
d
13. A nurse prepares to discharge a client with cardiac dysrhythmia who is prescribed home health care services. Which priority information should be communicated to the home health nurse upon discharge?
a. Medication reconciliation
b. Immunization history
c. Religious beliefs
d. Nutrition preferences
ANS: A
The home health nurse needs to know current medications the client is taking to ensure assessment, evaluation, and further education related to these medications. The other information will not assist the nurse to develop a plan of care for the client.
DIF: Applying/Application REF: 673
KEY: Hand-off communication
MSC: Integrated Process: Communication and Documentation
NOT: Client Needs Category: Safe and Effective Care Environment: Management of Care
6. Which sign/symptom is one of the first indicators of hyperthyroidism that is often noticed by the patient?
a. Eyelid or globe lag
b. Vision changes or tiring of the eyes
c. Protruding eyes
d. Photophobia
b
13. A nurse cares for a client recovering from prosthetic valve replacement surgery. The client asks, "Why will I need to take anticoagulants for the rest of my life?" How should the nurse respond?
a. "The prosthetic valve places you at greater risk for a heart attack."
b. "Blood clots form more easily in artificial replacement valves."
c. "The vein taken from your leg reduces circulation in the leg."
d. "The surgery left a lot of small clots in your heart and lungs."
ANS: B
Synthetic valve prostheses and scar tissue provide surfaces on which platelets can aggregate easily and initiate the formation of blood clots. The other responses are inaccurate.
DIF: Applying/Application REF: 696
KEY: Valve disorder| patient education| anticoagulants
MSC: IntegratedProcess:Teaching/Learning
NOT: Client Needs Category: Physiological Integrity: Reduction of Risk Potential
6. A nurse plans care for a client with hypothyroidism. Which priority problem should the nurse plan to address first for this client?
a. Heat intolerance
b. Body image problems
c. Depression and withdrawal
d. Obesity and water retention
c. Hypothyroidism causes many problems in psychosocial functioning. Depression is the most common reason for seeking medical attention. Memory and attention span may be impaired. The client's family may have great difficulty accepting and dealing with these changes. The client is often unmotivated to participate in self-care. Lapses in memory and attention require the nurse to ensure that the client's environment is safe. Heat intolerance is seen in hyperthyroidism. Body image problems and weight issues do not take priority over mental status and safety.
The nurse sees that a 45-year-old woman has a low hemoglobin level. The nurse would perform a dietary assessment to identify a possible deficiency in which nutrient?
a. Calcium
b. Vitamin K
c. Iron
d. Vitamin D
c
10. The charge nurse of the medical-surgical unit is making staff assignments. Which staff member should be assigned to a client with chronic kidney disease who is exhibiting a low-grade fever and a pericardial friction rub?
a. Registered nurse who just floated from the surgical unit
b. Registered nurse who just floated from the dialysis unit
c. Registered nurse who was assigned the same client yesterday d. Licensed practical nurse with 5 years' experience on this floor
ANS: C
The client is exhibiting symptoms of pericarditis, which can occur with chronic kidney disease. Continuity of care is important to assess subtle differences in clients. Therefore, the registered nurse (RN) who was assigned to this client previously should again give care to this client. The float nurses would not be as knowledgeable about the unit and its clients. The licensed practical nurse may not have the education level of the RN to assess for pericarditis.
17. The nurse is caring for a client who is prescribed a long-acting beta2 agonist. The client states, "The medication is too expensive to use every day. I only use my inhaler when I have an attack." How should the nurse respond?
a. "You are using the inhaler incorrectly. This medication should be taken daily."
b. "If you decrease environmental stimuli, it will be okay for you to use the inhaler
only for asthma attacks."
c. "Tell me more about your fears related to feelings of breathlessness."
d. "It is important to use this type of inhaler every day. Let's identify potential
community services to help you."
ANS: D
Long-acting beta2 agonists should be used every day to prevent asthma attacks. This medication should not be taken when an attack starts. Asthma medications can be expensive. Telling the client that he or she is using the inhaler incorrectly does not address the client's financial situation, which is the main issue here. Clients with limited incomes should be provided with community resources. Asking the client about fears related to breathlessness does not address the client's immediate concerns.
DIF: Applying/Application REF: 554
KEY: Case management| medication
MSC: Integrated Process: Communication and Documentation NOT: Client Needs Category: Psychosocial Integrity
10. After teaching a client with a history of renal calculi, the nurse assesses the clients understanding. Which statement made by the client indicates a correct understanding of the teaching?
a. I should drink at least 3 liters of fluid every day.
b. I will eliminate all dairy or sources of calcium from my diet.
c. Aspirin and aspirin-containing products can lead to stones.
d. The doctor can give me antibiotics at the first sign of a stone.
A
Dehydration contributes to the precipitation of minerals to form a stone. Although increased intake of calcium causes hypercalcemia and leads to excessive calcium filtered into the urine, if the client is well hydrated the calcium will be excreted without issues. Dehydration increases the risk for supersaturation of calcium in the urine, which contributes to stone formation. The nurse should encourage the client to drink more fluids, not decrease calcium intake. Ingestion of aspirin or aspirin-containing products does not cause a stone. Antibiotics neither prevent nor treat a stone.
1. A nurse assesses a client who is 6 hours post-surgery for a nasal fracture and has nasal packing in place. Which actions should the nurse take? (Select all that apply.)
a. Observe for clear drainage.
b. Assess for signs of bleeding.
c. Watch the client for frequent swallowing.
d. Ask the client to open his or her mouth.
e. Administer a nasal steroid to decrease edema.
f. Change the nasal packing.
ANS: A, B, C, D
The nurse should observe for clear drainage because of the risk for cerebrospinal fluid leakage. The nurse should assess for signs of bleeding by asking the client to open his or her mouth and observing the back of the throat for bleeding. The nurse should also note whether the client is swallowing frequently because this could indicate postnasal bleeding. A nasal steroid would increase the risk for infection. It is too soon to change the packing, which should be changed by the surgeon the first time.
DIF: Applying/Application REF: 531 KEY: Surgical care MSC: Integrated Process: Nursing Process: Implementation
NOT: Client Needs Category: Physiological Integrity: Reduction of Risk Potential
Question 1 of 41
Which client statement indicates that stem cell transplantation that is scheduled to take place in his home is not a viable option?
a. "I don't feel strong enough, but my wife said she would help."
b. "I was a nurse, so I can take care of myself."
c. "I will have lots of medicine to take."
d. "We live 5 miles from the hospital."
b
"I was a nurse, so I can take care of myself."
Stem cell transplantation in the home setting requires support, assistance, and coordination from others. The client cannot manage this type of care on his own. The client must be emotionally stable to be a candidate for this type of care. It is acceptable for the client's spouse to support the client undergoing this procedure. It is not unexpected for the client to be taking several prescriptions. Five miles is an acceptable distance from the hospital, in case of emergency.
13. A nurse cares for a client who has an 80% blockage of the right coronary artery (RCA) and is scheduled for bypass surgery. Which intervention should the nurse be prepared to implement while this client waits for surgery?
a. Administration of IV furosemide (Lasix)
b. Initiation of an external pacemaker
c. Assistance with endotracheal intubation
d. Placement of central venous access
B
The RCA supplies the right atrium, the right ventricle, the inferior portion of the left ventricle, and the atrioventricular (AV) node. It also supplies the sinoatrial node in 50% of people. If the client totally occludes the RCA, the AV node would not function and the client would go into heart block, so emergency pacing should be available for the client. Furosemide, intubation, and central venous access will not address the primary complication of RCA occlusion, which is AV node malfunction.
A nurse collaborates with an unlicensed assistive personnel (UAP) to provide care for a client who is prescribed a 24-hour urine specimen collection. Which statement should the nurse include when delegating this activity to the UAP?
a. "Note the time of the client's first void and collect urine for 24 hours."
b. "Add the preservative to the container at the end of the test."
c. "Start the collection by saving the first urine of the morning."
d. "It is okay if one urine sample during the 24 hours is not collected."
ANS: A
The collection of a 24-hour urine specimen is often delegated to a UAP. The nurse must ensure that the UAP understands the proper process for collecting the urine. The 24-hour urine collection specimen is started after the client's first urination. The first urine specimen is discarded because there is no way to know how long it has been in the bladder, but the time of the client's first void is noted. The client adds all urine voided after that first discarded specimen during the next 24 hours. When the 24-hour mark is reached, the client voids one last time and adds this specimen to the collection. The preservative, if used, must be added to the container at the beginning of the collection. All urine samples need to be collected for the test results to be accurate.
10. A nurse cares for a client who is recovering after a nephrostomy tube was placed 6 hours ago. The nurse notes drainage in the tube has decreased from 40 mL/hr to 12 mL over the last hour. Which action should the nurse take?
a. Document the finding in the client's record.
b. Evaluate the tube as working in the hand-off report. c. Clamp the tube in preparation for removing it.
d. Assess the client's abdomen and vital signs.
ANS: D
The nephrostomy tube should continue to have a consistent amount of drainage. If the drainage slows or stops, it may be obstructed. The nurse must notify the provider, but first should carefully assess the client's abdomen for pain and distention and check vital signs so that this information can be reported as well. The other interventions are not appropriate.
18. A pulmonary nurse cares for clients who have chronic obstructive pulmonary disease (COPD). Which client should the nurse assess first?
a. A 46-year-old with a 30-pack-year history of smoking
b. A 52-year-old in a tripod position using accessory muscles to breathe
c. A 68-year-old who has dependent edema and clubbed fingers
d. A 74-year-old with a chronic cough and thick, tenacious secretions
ANS: B
The client who is in a tripod position and using accessory muscles is working to breathe. This client must be assessed first to establish how well the client is breathing and provide interventions to minimize respiratory failure. The other clients are not in acute distress.
DIF: Applying/Application REF: 559 KEY: Health screening MSC: IntegratedProcess:NursingProcess:Assessment
NOT: Client Needs Category: Safe and Effective Care Environment: Management of Care
18. Which medication increases the risk for the patient to develop infection?
a. Steroids
b. Aspirin
c. Iron solutions
d. Heparin
a
14. After teaching a client who is being discharged home after mitral valve replacement surgery, the nurse assesses the client's understanding. Which client statement indicates a need for additional teaching?
a. "I'll be able to carry heavy loads after 6 months of rest."
b. "I will have my teeth cleaned by my dentist in 2 weeks."
c. "I must avoid eating foods high in vitamin K, like spinach."
d. "I must use an electric razor instead of a straight razor to shave."
ANS: B
Clients who have defective or repaired valves are at high risk for endocarditis. The client who has had valve surgery should avoid dental procedures for 6 months because of the risk for endocarditis. When undergoing a mitral valve replacement surgery, the client needs to be placed on anticoagulant therapy to prevent vegetation forming on the new valve. Clients on anticoagulant therapy should be instructed on bleeding precautions, including using an electric razor. If the client is prescribed warfarin, the client should avoid foods high in vitamin K. Clients recovering from open heart valve replacements should not carry anything heavy for 6 months while the chest incision and muscle heal.
DIF: Applying/Application REF: 694
KEY: Valve disorder| patient education| hygiene
MSC: IntegratedProcess:Teaching/Learning
NOT: Client Needs Category: Physiological Integrity: Reduction of Risk Potential
Question 4 of 16
In type 1 diabetes, insulin injections are necessary to maintain which action between insulin and glucose?
a. Glucose intolerance
b. Homeostasis
c. Insulin intolerance
d. Negative feedback
b
Homeostasis
Insulin injections maintain homeostasis, or normal balance, between insulin and glucose in the client with type 1 diabetes. Type 1 diabetes is a lack of insulin production, not glucose intolerance, and requires frequent doses of insulin. Negative feedback does not occur in type 1 diabetes because of lack of insulin.
14. A nurse teaches a client with diabetes mellitus and a body mass index of 42 who is at high risk for coronary artery disease. Which statement related to nutrition should the nurse include in this clients teaching?
a. The best way to lose weight is a high-protein, low-carbohydrate diet.
b. You should balance weight loss with consuming necessary nutrients.
c. A nutritionist will provide you with information about your new diet.
d. If you exercise more frequently, you wont need to change your diet.
B
Clients at risk for cardiovascular diseases should follow the American Heart Association guidelines to combat obesity and improve cardiac health. The nurse should encourage the client to eat vegetables, fruits, unrefined whole-grain products, and fat-free dairy products while losing weight. High-protein food items are often high in fat and calories. Although the nutritionist can assist with client education, the nurse should include nutrition education and assist the client to make healthy decisions. Exercising and eating nutrient-rich foods are both important components in reducing cardiovascular risk.
A client diagnosed with hyperpituitarism resulting from a prolactin-secreting tumor has been prescribed bromocriptine mesylate (Parlodel). As a dopamine agonist, what effect does this drug have by stimulating dopamine receptors in the brain?
A. Decreases the risk for cerebrovascular disease
B. Increases the risk for depression
C. Inhibits the release of some pituitary hormones
D. Stimulates the release of some pituitary hormones
C.
Bromocriptine mesylate inhibits the release of both prolactin and growth hormone. It does not decrease the risk for cerebrovascular disease leading to stroke. Increased risk for depression is not associated with the use of bromocriptine mesylate; however, hallucinations have been reported as a side effect. Bromocriptine mesylate does not stimulate the release of any hormones.
15. A client has been brought to the emergency department with a life-threatening chest injury. What action by the nurse takes priority?
a. Apply oxygen at 100%.
b. Assess the respiratory rate.
c. Ensure a patent airway.
d. Start two large-bore IV lines.
ANS: C
The priority for any chest trauma client is airway, breathing, circulation. The nurse first ensures the client has a patent airway. Assessing respiratory rate and applying oxygen are next, followed by inserting IVs.
A nurse is preparing to administer a blood transfusion. What action is most important?
a. Correctly identifying client using two identifiers
b. Ensuring informed consent is obtained if required
c. Hanging the blood product with Ringer's lactate
d. Staying with the client for the entire transfusion
b. Ensuring informed consent is obtained if required
10. After teaching a client with a history of renal calculi, the nurse assesses the client's understanding. Which statement made by the client indicates a correct understanding of the teaching?
a. "I should drink at least 3 liters of fluid every day."
b. "I will eliminate all dairy or sources of calcium from my diet." c. "Aspirin and aspirin-containing products can lead to stones." d. "The doctor can give me antibiotics at the first sign of a stone."
ANS: A
Dehydration contributes to the precipitation of minerals to form a stone. Although increased intake of calcium causes hypercalcemia and leads to excessive calcium filtered into the urine, if the client is well hydrated the calcium will be excreted without issues. Dehydration increases the risk for supersaturation of calcium in the urine, which contributes to stone formation. The nurse should encourage the client to drink more fluids, not decrease calcium intake. Ingestion of aspirin or aspirin-containing products does not cause a stone. Antibiotics neither prevent nor treat a stone.
10. Which patient is most likely to exceed the renal threshold if there is noncompliance with the prescribed therapeutic regimen?
a. Has recurrent kidney stone formation
b. Has type 2 diabetes mellitus
c. Has functional urinary incontinence
d. Has biliary obstruction
b
15. A client has been hospitalized with tuberculosis (TB). The client's spouse is fearful of entering the room where the client is in isolation and refuses to visit. What action by the nurse is best?
a. Ask the spouse to explain the fear of visiting in further detail.
b. Inform the spouse the precautions are meant to keep other clients safe.
c. Show the spouse how to follow the isolation precautions to avoid illness. d. Tell the spouse that he or she has already been exposed, so it's safe to visit.
ANS: A
The nurse needs to obtain further information about the spouse's specific fears so they can be addressed. This will decrease stress and permit visitation, which will be beneficial for both client and spouse. Precautions for TB prevent transmission to all who come into contact with the client. Explaining isolation precautions and what to do when entering the room will be helpful, but this is too narrow in scope to be the best answer. Telling the spouse it's safe to visit is demeaning of the spouse's feelings.
A nurse is preparing to hang a blood transfusion. Which action is most important?
a. Documenting the transfusion
b. Placing the client on NPO status
c. Placing the client in isolation
d. Putting on a pair of gloves
d. Putting on a pair of gloves
14. A nurse is in charge of the coronary intensive care unit. Which client should the nurse see first?
a. Client on a nitroglycerin infusion at 5 mcg/min, not titrated in the last 4 hours
b. Client who is 1 day post coronary artery bypass graft, blood pressure 180/100 mm
Hg
c. Client who is 1 day post percutaneous coronary intervention, going home this
morning
d. Client who is 2 days post coronary artery bypass graft, became dizzy this a.m.
while walking
ANS: B
Hypertension after coronary artery bypass graft surgery can be dangerous because it puts too much pressure on the suture lines and can cause bleeding. The charge nurse should see this client first. The client who became dizzy earlier should be seen next. The client on the nitroglycerin drip is stable. The client going home can wait until the other clients are cared for.
DIF: Analyzing/Analysis REF: 777
KEY: Coronary artery disease| coronary artery bypass graft| collaboration
MSC: IntegratedProcess:NursingProcess:Assessment
NOT: Client Needs Category: Safe and Effective Care Environment: Management of Care
7. Patients diagnosed with an anterior pituitary tumor can have symptoms of acremegaly or gigantism. These symptoms are a result of overproduction of which hormone?
a. ACTH
b. PRL
c. Gonadotropins
d. GH
d
GH
10. A nurse cares for a client who has elevated levels of antidiuretic hormone (ADH). Which disorder should the nurse identify as a trigger for the release of this hormone?
a. Pneumonia
b. Dehydration
c. Renal failure d. Edema
ANS: B
ADH increases tubular permeability to water, leading to absorption of more water into the capillaries. ADH is triggered by a rising extracellular fluid osmolarity, as occurs in dehydration. Pneumonia, renal failure, and edema would not trigger the release of ADH.
A client is being admitted for pneumonia. The sputum culture is positive for streptococcus, and the client asks about the length of the treatment. On what does the nurse base the answer?
A) The client will be treated for 5 to 7 days.
B) The client will require IV antibiotics for 7 to 10 days.
C) The client will complete 6 days of therapy.
D) The client must be afebrile for 24 hours.
(Chp. 31; elsevier resources)
A) The client will be treated for 5 to 7 days.
(Chp. 31; elsevier resources)
Question 2 of 41
A 32-year-old client recovering from a sickle cell crisis is to be discharged. The nurse says, "You and all clients with sickle cell disease are at risk for infection because of your decreased spleen function. For this reason, you will most likely be prescribed an antibiotic before discharge." Which drug does the nurse anticipate the health care provider will request?
a. Cefaclor (Ceclor)
b. Gentamicin (Garamycin)
c. Penicillin V (Pen-V K)
d. Vancomycin (Vancocin)
c
Penicillin V (Pen-V K)
Prophylactic therapy with twice-daily oral penicillin reduces the incidence of pneumonia and other streptococcal infections and is the correct drug to use. It is a standard protocol for long-term prophylactic use in clients with sickle cell disease. Cefaclor (Ceclor) and vancomycin (Vancocin) are antibiotics more specific for short-term use and would be inappropriate for this client. Gentamicin (Garamycin) is a drug that can cause liver and kidney damage with long-term use.
14. A nurse assesses a client with tachycardia. Which clinical manifestation requires immediate intervention by the nurse?
a. Mid-sternal chest pain
b. Increased urine output
c. Mild orthostatic hypotension
d. P wave touching the T wave
ANS: A
Chest pain, possibly angina, indicates that tachycardia may be increasing the client's myocardial workload and oxygen demand to such an extent that normal oxygen delivery cannot keep pace. This results in myocardial hypoxia and pain. Increased urinary output and mild orthostatic hypotension are not life-threatening conditions and therefore do not require immediate intervention. The P wave touching the T wave indicates significant tachycardia and should be assessed to determine the underlying rhythm and cause; this is an important assessment but is not as critical as chest pain, which indicates cardiac cell death.
DIF: Applying/Application REF: 663
KEY: Cardiac electrical conduction
MSC: IntegratedProcess:NursingProcess:Assessment
NOT: Client Needs Category: Safe and Effective Care Environment: Management of Care
7. The target tissue for ADH is which organ?
a. Hypothalamus
b. Thyroid
c. Ovary
d. Kidney
d
Kidney
After teaching a client who is recovering from an endoscopic trans-nasal hypophysectomy, the nurse assesses the clients understanding. Which statement made by the client indicates a correct understanding of the teaching?
a. I will wear dark glasses to prevent sun exposure.
b. Ill keep food on upper shelves so I do not have to bend over.
c. I must wash the incision with peroxide and redress it daily.
d. I shall cough and deep breathe every 2 hours while I am awake.
B (After this surgery, the client must take care to avoid activities that can increase intracranial pressure. The client should avoid bending from the waist and should not bear down, cough, or lie flat. With this approach, there is no incision to clean and dress. Protection from sun exposure is not necessary after this procedure.)
During discharge planning after admission for a myocardial infarction, the client says, "I won't be able to increase my activity level. I live in an apartment, and there is no place to walk." What is the nurse's best response?
a. "You are right. Work on your diet then."
b. "You must find someplace to walk."
c. "Walk around the edge of your apartment complex."
d. "Where might you be able to walk?"
d. "Where might you be able to walk?"
Asking the client where he or she might be able to walk calls for cooperation and participation from the client; increased activity is imperative for this client. Telling the client to work on diet is an inappropriate response. Telling the client to find someplace to walk is too demanding to be therapeutic. Telling the client to walk around the apartment complex is domineering and will not likely achieve cooperation from the client.
19. The nurse is caring for a patient with thrombocytopenia. Which order does the nurse question?
a. Test all urine and stool for occult blood.
b. Avoid IM injections.
c. Administer enemas.
d. Apply ice to areas of trauma.
c
2. A nurse assesses a client who has developed epistaxis. Which conditions in the client's history should the nurse identify as potential contributors to this problem? (Select all that apply.)
a. Diabetes mellitus
b. Hypertension
c. Leukemia
d. Cocaine use
e. Migraine
f. Elevated platelets
ANS: B, C, D
Frequent causes of nosebleeds include trauma, hypertension, leukemia and other blood dyscrasias, inflammation, tumor, dry air, blowing or picking the nose, cocaine use, and intranasal procedures. Diabetes, migraines, and elevated platelets and cholesterol levels do not cause epistaxis.
DIF: Understanding/Comprehension REF: 532
KEY: Respiratory distress/failure
MSC: IntegratedProcess:NursingProcess:Assessment
NOT: Client Needs Category: Physiological Integrity: Reduction of Risk Potential
10. The nurse is caring for a patient who has an indwelling catheter and subsequently developed a UTI. The patient has been receiving antibiotics for several days, but develops hypotension, a rapid pulse, and confusion. The nurse suspects urosepsis and alerts the health care provider. Which diagnostics test is the provider most likely to order to confirm urosepsis?
a. Culture of the drainage bag
b. Culture of the catheter tip
c. Blood culture
d. Repeat urinalysis
c
19. The nurse is teaching a client with chronic obstructive pulmonary disease who has been prescribed continuous oxygen therapy at home. Which statement indicates the client correctly understands the teaching?
a. "I plan to wear my oxygen when I exercise and feel short of breath."
b. "I will use my portable oxygen when grilling burgers in the backyard."
c. "I plan to use cotton balls to cushion the oxygen tubing on my ears."
d. "I will only smoke while I am wearing my oxygen via nasal cannula."
ANS: C
Cotton balls can decrease pressure ulcers from the oxygen tubing. Continuous oxygen orders mean the client should wear the oxygen at all times. Oxygen fuels a fire. Wearing oxygen while grilling and smoking increases the risk for fire.
DIF: Applying/Application REF: 563
KEY: Safety| patient education| oxygen therapy
MSC: IntegratedProcess:NursingProcess:Assessment
NOT: Client Needs Category: Safe and Effective Care Environment: Safety and Infection Control
In assessing the patient's hematologic status, which questions would the nurse include?
(Select all that apply.)
a. "Have you had unusual or increased fatigue?"
b. "Have you ever had any radiation therapy?"
c. "Have you ever had a job that exposed you to
chemicals?"
d. "Do you have a personal or family history of blood
disorders?"
e. "What drugs have you used in the past 3 days?"
a, b, c, d
During discharge planning after admission for a myocardial infarction, the client says, "I won't be able to increase my activity level. I live in an apartment, and there is no place to walk." What is the nurse's best response?
"You are right. Work on your diet then."
"You must find someplace to walk."
"Walk around the edge of your apartment complex."
"Where might you be able to walk?"
"Where might you be able to walk?"
Asking the client where he or she might be able to walk calls for cooperation and participation from the client; increased activity is imperative for this client. Telling the client to work on diet is an inappropriate response. Telling the client to find someplace to walk is too demanding to be therapeutic. Telling the client to walk around the apartment complex is domineering and will not likely achieve cooperation from the client.
Which laboratory result is consistent with a diagnosis of hyperthyroidism?
a. Decreased serum triiodothyronine (T3) and thyroxine
(T4) levels
b. Elevated serum thyrotropin-releasing hormone
(TRH) level
c. Decreased radioactive iodine uptake
d. Increased serum T3 and T4
d
10. A patient with PKD reports a severe headache and is at risk for a berry aneurysm. What is the nurse's priority action?
a. Assess the pain and give a prn pain medication.
b. Reassure the patient that this is an expected aspect of the disease.
c. Assess for neurlologic changes and check vital signs.
d. Monitor for hematuria and decreased urinary output.
c
10. The nurse is talking to an older adult male patient who is reasonably healthy for his age, but has benign prostatic hyperplasia (BPH). Which conditioin does the BPH potentially place him at risk for?
a. Prerenal acute kidney injury
b. Postrenal acute kidney injury
c. Polycystic kidney disease
d. Acute glomerulonephritis
b
16. A client is being discharged soon on warfarin (Coumadin). What menu selection for dinner indicates the client needs more education regarding this medication?
a. Hamburger and French fries
b. Large chef's salad and muffin
c. No selection; spouse brings pizza d. Tuna salad sandwich and chips
ANS: B
Warfarin works by inhibiting the synthesis of vitamin K-dependent clotting factors. Foods high in vitamin K thus interfere with its action and need to be eaten in moderate, consistent amounts. The chef's salad most likely has too many leafy green vegetables, which contain high amounts of vitamin K. The other selections, while not particularly healthy, will not interfere with the medication's mechanism of action.
A client receiving a blood transfusion develops anxiety and low back pain. After stopping the transfusion, what action by the nurse is most important?
a. Documenting the events in the client's medical record
b. Double-checking the client and blood product identification
c. Placing the client on strict bedrest until the pain subsides
d. Reviewing the client's medical record for known allergies
b. Double-checking the client and blood product identification
15. A client with coronary artery disease (CAD) asks the nurse about taking fish oil supplements. What response by the nurse is best?
a. "Fish oil is contraindicated with most drugs for CAD."
b. "The best source is fish, but pills have benefits too."
c. "There is no evidence to support fish oil use with CAD."
d. "You can reverse CAD totally with diet and supplements."
ANS: B
Omega-3 fatty acids have shown benefit in reducing lipid levels, in reducing the incidence of sudden cardiac death, and for stabilizing atherosclerotic plaque. The best source is fish three times a week or some fish oil supplements. The other options are not accurate.
DIF: Understanding/Comprehension REF: 761
KEY: Coronary artery disease| lipid-reducing agents| supplements| patient education MSC: IntegratedProcess:Teaching/Learning
NOT: Client Needs Category: Health Promotion and Maintenance
A nurse assesses a female client who presents with hirsutism. Which question should the nurse ask when assessing this client?
a. "How do you plan to pay for your treatments?"
b. "How do you feel about yourself?"
c. "What medications are you prescribed?"
d. "What are you doing to prevent this from happening?"
ANS: B
Hirsutism, or excessive hair growth on the face and body, can result from endocrine disorders. This may cause a disruption in body image, especially for female clients. The nurse should inquire into the client's body image and self-perception. Asking about the client's financial status or current medications does not address the client's immediate problem. The client is not doing anything to herself to cause the problem, nor can the client prevent it from happening.
Ch.62 p. 1276, Patient-Centered Care; Quality Improvement; Safety
The patient is a 32-year-old woman admitted to your unit after surgery for fractures of the left arm and leg resulting from a car crash. She is awake and able to verify her medical history of rheumatoid arthritis and her usual daily medications. These are 10 mg of prednisone, naproxen 800 mg twice daily, oral contraceptives, calcium 600 mg, and one multiple vitamin tablet. All of these are prescribed for her to receive during her hospitalization. She is concerned about pain management and how long the recovery will be for the fractures. She is friendly, somewhat anxious, asks many questions, and wants to do "her part" to ensure good recovery. Over the next 4 days, she has become quieter, mumbles that her head and stomach hurt, and now does not recognize the assistant who has been providing her daily care. When she receives her medications, she has difficulty picking them up. The nursing assistant remarks that taking her pulse is difficult because it is so slow and irregular. When you assess her, she is so weak that she is unable to lift her arm for a blood pressure check. Her blood pressure is 92/50, which is down from the 128/84 reading on admission. You also verify that her heart beat is slow and irregular.
3. How could this problem been avoided?
Daily assessment of her salivary or serum cortisol levels could have indicated a need for a higher dose. Also, because she was receiving prednisone, daily blood glucose levels should have been performed. Examining these parameters would provide data to determine the adequacy of her therapy, as well as its potential side effects.
An older adult client, 4 hours after coronary artery bypass graft (CABG), has a blood pressure of 80/50 mm Hg. What action does the nurse take?
No action is required; low blood pressure is normal for older adults.
No action is required for postsurgical CABG clients.
Assess pulmonary artery wedge pressure (PAWP).
Give ordered loop diuretics.
Assess pulmonary artery wedge pressure (PAWP).
Decreased preload as exhibited by decreased PAWP could indicate hypovolemia secondary to hemorrhage or vasodilation; hypotension could cause the graft to collapse. Low blood pressure is not normal in older adults or postoperative clients. The cause of hypotension must be found and treated; further action is needed to determine additional interventions. Hypotension could be caused by hypovolemia; giving loop diuretics increases hypovolemia.
Question 3 of 41
The nurse is infusing platelets to a client who is scheduled for a hematopoietic stem cell transplant. What procedure does the nurse follow?
a. Administer intravenous corticosteroids before starting the transfusion.
b. Allow the platelets to stabilize at the client's bedside for 30 minutes.
c. Infuse the transfusion over a 15- to 30-minute period.
d. Set up the infusion with the standard transfusion Y tubing.
c
Infuse the transfusion over a 15- to 30-minute period
The volume of platelets—200 or 300 mL (standard amount)—needs to be infused rapidly over a 15- to 30-minute period. Administering steroids is not standard practice in administering platelets. Platelets must be administered immediately after they are received; they are considered to be quite fragile. A special transfusion set with a smaller filter and shorter tubing is used to get the platelets into the client quickly and efficiently.
16. A client is being discharged on long-term therapy for tuberculosis (TB). What referral by the nurse is most appropriate?
a. Community social worker for Meals on Wheels
b. Occupational therapy for job retraining
c. Physical therapy for homebound therapy services
d. Visiting Nurses for directly observed therapy
ANS: D
Directly observed therapy is often utilized for managing clients with TB in the community. Meals on Wheels, job retraining, and home therapy may or may not be appropriate.
11. A nurse reviews a female client's laboratory results. Which results from the client's urinalysis should the nurse recognize as abnormal?
a. pH 5.6
b. Ketone bodies present
c. Specific gravity of 1.020 d. Clear and yellow color
ANS: B
Ketone bodies are by-products of incomplete metabolism of fatty acids. Normally no ketones are present in urine. Ketone bodies are produced when fat sources are used instead of glucose to provide cellular energy. A pH between 4.6 and 8, specific gravity between 1.005 and 1.030, and clear yellow urine are normal findings for a female client's urinalysis.
11. A patient with PKD reports nocturia. What is the nocturia caused by?
a. Increased fluid intake in the evening.
b. Increased hypertension.
c. Decreased urine-concentrating ability
d. Detrusor irritability
c
A local hunter is admitted to the intensive care unit with a diagnosis of inhalation anthrax. Which medications does the RN anticipate the health care provider will order?
A) Amoxicillin (Amoxil, Triamox) 500 mg orally every 8 hours
B) Ceftriaxone (Rocephin) 2 g IV every 8 hours
C) Ciprofloxacin (Cipro) 400 mg IV every 12 hours
D) Pyrazinamide (Zinamide) 1000 to 2000 mg orally every day
(Chp. 31; elsevier resources)
C) Ciprofloxacin (Cipro) 400 mg IV every 12 hours
(Chp. 31; elsevier resources)
20. A patient undergoing hematopoietic stem cell transplantation reports severe fatigue. To assist the patient with energy management, what does the nurse encourage the patient to do? (Select all that apply.)
a. Verbalize feelings about limitations.
b. Monitor nutritional intake to ensure adequate energy resources.
c. Avoid napping throughout the day.
d. Limit the number of visitors as appropriate.
e. Plan activities for periods when the patient has the most energy.
f. Monitor overall response to self-care activities.
a, b, d, e, f
An older adult client, 4 hours after coronary artery bypass graft (CABG), has a blood pressure of 80/50 mm Hg. What action does the nurse take?
a. No action is required; low blood pressure is normal for older adults.
b. No action is required for postsurgical CABG clients.
c. Assess pulmonary artery wedge pressure (PAWP).
d. Give ordered loop diuretics.
c. Assess pulmonary artery wedge pressure (PAWP).
Decreased preload as exhibited by decreased PAWP could indicate hypovolemia secondary to hemorrhage or vasodilation; hypotension could cause the graft to collapse. Low blood pressure is not normal in older adults or postoperative clients. The cause of hypotension must be found and treated; further action is needed to determine additional interventions. Hypotension could be caused by hypovolemia; giving loop diuretics increases hypovolemia.
A nurse assesses clients who are at risk for diabetes mellitus. Which client is at greatest risk?
a. A 29-year-old Caucasian
b. A 32-year-old African-American
c. A 44-year-old Asian
d. A 48-year-old American Indian
Diabetes is a particular problem among African Americans, Hispanics, and American Indians. The incidence of diabetes increases in all races and ethnic groups with age. Being both an American Indian and middle-aged places this client at highest risk.
11. Which combination of drugs is the most nephrotoxic?
a. Angiotensin-converting enzyme (ACE) inhibitors and aspirin
b. Antiotensin II receptor blockers and antiacids
c. Aminoglycoside antibiotics and nonsteroidal antiinflammatory drugs (NSAIDs). n
d. Calcium channel blockers and antihistamines
c
17. A client is in the family practice clinic reporting a severe cough that has lasted for 5 weeks. The client is so exhausted after coughing that work has become impossible. What action by the nurse is most appropriate?
a. Arrange for immediate hospitalization.
b. Facilitate polymerase chain reaction testing. c. Have the client produce a sputum sample. d. Obtain two sets of blood cultures.
ANS: B
Polymerase chain reaction testing is used to diagnose pertussis, which this client is showing manifestations of. Hospitalization may or may not be needed but is not the most important action. The client may or may not be able to produce sputum, but sputum cultures for this disease must be obtained via deep suctioning. Blood cultures will be negative.
The nurse is reviewing the patient's medication list and sees that the patient is receiving parenteral enoxaparin (Lovenox). Which outcome statement is the target of the enoxaparin therapy?
a. Patient will not develop signs/symptoms of a blood
clot.
b. Patient will report a decrease in fatigue and
dizziness.
c. Patient will not develop signs/symptoms of infection.
d. Patient will demonstrate no shortness of breath on
exertion.
a
15. A nurse teaches a client who experiences occasional premature atrial contractions (PACs) accompanied by palpitations that resolve spontaneously without treatment. Which statement should the nurse include in this client's teaching?
a. "Minimize or abstain from caffeine."
b. "Lie on your side until the attack subsides."
c. "Use your oxygen when you experience PACs."
d. "Take amiodarone (Cordarone) daily to prevent PACs."
ANS: A
PACs usually have no hemodynamic consequences. For a client experiencing infrequent PACs, the nurse should explore possible lifestyle causes, such as excessive caffeine intake and stress. Lying on the side will not prevent or resolve PACs. Oxygen is not necessary. Although medications may be needed to control symptomatic dysrhythmias, for infrequent PACs, the client first should try lifestyle changes to control them.
DIF: Applying/Application REF: 663
KEY: Patient education| cardiac electrical conduction
MSC: IntegratedProcess:Teaching/Learning
NOT: Client Needs Category: Health Promotion and Maintenance
Ch.61 Question 7 of 16
The nurse is instructing a client who will undergo a suppression test. Which statement by the client indicates that teaching was effective?
a. "I am being tested to see whether my hormone glands are hyperactive."
b. "I am being tested to see whether my hormone glands are hypoactive."
c. "I am being tested to see whether my kidneys work at all."
d. "I will be given more hormones as a trigger."
a
"I am being tested to see whether my hormone glands are hyperactive."
Suppression tests are used when hormone levels are high or in the upper range of normal. Failure of suppression of hormone production during testing indicates hyperfunction. A stimulation test assesses whether hormone glands are hypoactive. The adrenal glands are endocrine glands that are located on the kidneys; a suppression test does not measure kidney function. Hormones are given as a trigger in a stimulation test.
15. A nurse cares for a client with infective endocarditis. Which infection control precautions should the nurse use?
a. Standard Precautions
b. Bleeding precautions
c. Reverse isolation
d. Contact isolation
ANS: A
The client with infective endocarditis does not pose any specific threat of transmitting the causative organism. Standard Precautions should be used. Bleeding precautions or reverse or contact isolation is not necessary.
DIF: Applying/Application REF: 697
KEY: Infection| Standard Precautions
MSC: Integrated Process: Nursing Process: Implementation
NOT: Client Needs Category: Safe and Effective Care Environment: Safety and Infection Control
7. Which laboratory result is consistent with a diagnosis of hyperthyroidism?
a. Decreased serum triiodothyronine (T3) and thyroxine (T4) levels
b. Elevated serum thyrotropin-releasing hormone (TRH) level
c. Decreased radioactive idoine uptake
d. Increased serum T3 and T4
d
11. The nurse is teaching a woman how to prevent UTIs. What information does the nurse include?
a. Clean the perineal area from front to back.
b. Always use a condom if spermicides are used for contraception.
c. Obtain prescription for estrogen for vaginal dryness.
d. Avoid urinary stasis by urinating every 6 to 8 hours.
a
A client has thrombocytopenia. What client statement indicates the client understands self-management of this condition?
a. "I brush and use dental floss every day."
b. "I chew hard candy for my dry mouth."
c. "I usually put ice on bumps or bruises."
d. "Nonslip socks are best when I walk."
c. "I usually put ice on bumps or bruises."
The nurse has taught a client about influenza infection control. Which client statement indicates the need for further teaching?
A) "Handwashing is the best way to prevent transmission."
B) "I should avoid kissing and shaking hands."
C) "It is best to cough and sneeze into my upper sleeve."
D) "The intranasal vaccine can be given to everybody in the family."
(Chp. 31; elsevier resources)
D) "The intranasal vaccine can be given to everybody in the family."
(Chp. 31; elsevier resources)
11. A male client with chronic kidney disease (CKD) is refusing to take his medication and has missed two hemodialysis appointments. What is the best initial action for the nurse?
a. Discuss what the treatment regimen means to him.
b. Refer the client to a mental health nurse practitioner.
c. Reschedule the appointments to another date and time. d. Discuss the option of peritoneal dialysis.
ANS: A
The initial action for the nurse is to assess anxiety, coping styles, and the client's acceptance of the required treatment for CKD. The client may be in denial of the diagnosis. While rescheduling hemodialysis appointments may help, and referral to a mental health practitioner and the possibility of peritoneal dialysis are all viable options, assessment of the client's acceptance of the treatment should come first.
3. A nurse assesses a client who has facial trauma. Which assessment findings require immediate intervention? (Select all that apply.)
a. Stridor
b. Nasal stuffiness
c. Edema of the cheek
d. Ecchymosis behind the ear
e. Eye pain
f. Swollen chin
ANS: A, D
Stridor is a sign of airway obstruction and requires immediate intervention. Ecchymosis, or bruising, behind the ear is called "battle sign" and indicates basilar skull fracture.
Nasal stuffiness, edema of the cheek or chin, and eye pain do not interfere with respirations or neurologic function, and therefore are not priorities for immediate intervention.
DIF: Applying/Application REF: 534
KEY: Trauma| medical emergencies
MSC: IntegratedProcess:NursingProcess:Assessment
NOT: Client Needs Category: Safe and Effective Care Environment: Management of Care
Question 4 of 41
A 32-year-old client is recovering from a sickle cell crisis. His discomfort is controlled with pain medications and he is to be discharged. What medication does the nurse expect to be prescribed for him before his discharge?
a. Heparin (Heparin)
b. Hydroxyurea (Droxia)
c. Tissue plasminogen activator (t-PA)
d. Warfarin (Coumadin)
b
Hydroxyurea (Droxia)
Hydroxyurea (Droxia) has been used successfully to reduce sickling of cells and pain episodes associated with sickle cell disease (SCD). Clients with SCD are not prescribed anticoagulants such as heparin or warfarin (Coumadin). t-PA is used as a "clot buster" in clients who have had ischemic strokes.
15. A nurse cares for a client who has advanced cardiac disease and states, I am having trouble sleeping at night. How should the nurse respond?
a. I will consult the provider to prescribe a sleep study to determine the problem.
b. You become hypoxic while sleeping; oxygen therapy via nasal cannula will help.
c. A continuous positive airway pressure, or CPAP, breathing mask will help you breathe at night.
d. Use pillows to elevate your head and chest while you are sleeping.
D
The client is experiencing orthopnea (shortness of breath while lying flat). The nurse should teach the client to elevate the head and chest with pillows or sleep in a recliner. A sleep study is not necessary to diagnose this client. Oxygen and CPAP will not help a client with orthopnea.
7. A nurse assesses a client who is prescribed levothyroxine (Synthroid) for hypothyroidism. Which assessment finding should alert the nurse that the medication therapy is effective?
a. Thirst is recognized and fluid intake is appropriate.
b. Weight has been the same for 3 weeks.
c. Total white blood cell count is 6000 cells/mm3.
d. Heart rate is 70 beats/min and regular.
D. Hypothyroidism decreases body functioning and can result in effects such as bradycardia, confusion, and constipation. If a client's heart rate is bradycardic while on thyroid hormone replacement, this is an indicator that the replacement may not be adequate. Conversely, a heart rate above 100 beats/min may indicate that the client is receiving too much of the thyroid hormone. Thirst, fluid intake, weight, and white blood cell count do not represent a therapeutic response to this medication.
11. A nurse teaches a client who is recovering from a nephrectomy secondary to kidney trauma. Which statement should the nurse include in this client's teaching?
a. "Since you only have one kidney, a salt and fluid restriction is required."
b. "Your therapy will include hemodialysis while you recover."
c. "Medication will be prescribed to control your high blood pressure." d. "You need to avoid participating in contact sports like football."
ANS: D
Clients with one kidney need to avoid contact sports because the kidneys are easily injured. The client will not be required to restrict salt and fluids, end up on dialysis, or have new hypertension because of the nephrectomy.
17. A nurse is teaching a client about warfarin (Coumadin). What assessment finding by the nurse indicates a possible barrier to self-management?
a. Poor visual acuity
b. Strict vegetarian
c. Refusal to stop smoking d. Wants weight loss surgery
ANS: B
Warfarin works by inhibiting the synthesis of vitamin K-dependent clotting factors. Foods high in vitamin K thus interfere with its action and need to be eaten in moderate, consistent amounts. A vegetarian may have trouble maintaining this diet. The nurse should explore this possibility with the client. The other options are not related.
21. The home care nurse is visiting a patient who had a stem cell transplant. Which observation by the nurse requires immediate action?
a. The patient's grandson is visiting after receiving a MMR vaccine.
b. The patient bumps his toe on a chair and applies pressure to the toe for 10 minutes.
c. The patient with a platelet count of 48,000/mm3 follows platelet precautions.
d. The patient avoids going out to grocery shop in the winter months.
a
16. The nurse asks a client who has experienced ventricular dysrhythmias about substance abuse. The client asks, "Why do you want to know if I use cocaine?" How should the nurse respond?
a. "Substance abuse puts clients at risk for many health issues."
b. "The hospital requires that I ask you about cocaine use."
c. "Clients who use cocaine are at risk for fatal dysrhythmias."
d. "We can provide services for cessation of substance abuse."
ANS: C
Clients who use cocaine or illicit inhalants are particularly at risk for potentially fatal dysrhythmias. The other responses do not adequately address the client's question.
DIF: Remembering/Knowledge REF: 657 KEY: Cardiac electrical conduction| substance abuse MSC: IntegratedProcess:NursingProcess:Assessment NOT: Client Needs Category: Psychosocial Integrity
A nurse assesses a female client who presents with hirsutism. Which question should the nurse ask when assessing this client?
a. "How do you plan to pay for your treatments?"
b. "How do you feel about yourself?"
c. "What medications are you prescribed?"
d. "What are you doing to prevent this from happening?"
ANS: B
Hirsutism, or excessive hair growth on the face and body, can result from endocrine disorders. This may cause a disruption in body image, especially for female clients. The nurse should inquire into the client's body image and self-perception. Asking about the client's financial status or current medications does not address the client's immediate problem. The client is not doing anything to herself to cause the problem, nor can the client prevent it from happening.
16. A nurse assesses a client with pericarditis. Which assessment finding should the nurse expect to find?
a. Heart rate that speeds up and slows down
b. Friction rub at the left lower sternal border
c. Presence of a regular gallop rhythm
d. Coarse crackles in bilateral lung bases
ANS: B
The client with pericarditis may present with a pericardial friction rub at the left lower sternal border. This sound is the result of friction from inflamed pericardial layers when they rub together. The other assessments are not related.
DIF: Remembering/Knowledge REF: 699
KEY: Inflammatory response| assessment/diagnostic examination
MSC: IntegratedProcess:NursingProcess:Assessment
NOT: Client Needs Category: Physiological Integrity: Physiological Adaptation
Question 5 of 16
To best determine how well a client with diabetes mellitus is controlling blood glucose, which test does the nurse monitor?
a. Fasting blood glucose
b. Glycosylated hemoglobin (HbA1c)
c. Oral glucose tolerance test
d. Urine glucose level
b
Glycosylated hemoglobin (HbA1c)
Glycosylated hemoglobin indicates the average blood glucose over several months and is the best indicator of overall blood glucose control. Fasting blood glucose can be used to monitor glucose control, but it is not the best method (although this may be the method that clients are most familiar with). Oral glucose testing and urine glucose levels look at one period of time and are not the best methods to look at overall effectiveness of treatment.
11. A nurse cares for a client who has kidney stones from secondary hyperoxaluria. Which medication should the nurse anticipate administering?
a. Phenazopyridine (Pyridium)
b. Propantheline (Pro-Banthine)
c. Tolterodine (Detrol LA)
d. Allopurinol (Zyloprim)
D
Stones caused by secondary hyperoxaluria respond to allopurinol (Zyloprim). Phenazopyridine is given to clients with urinary tract infections. Propantheline is an anticholinergic. Tolterodine is an anticholinergic with smooth muscle relaxant properties.
To assist the health care provider in determining whether a patient is a candidate for fibrinolytic therapy, the nurse is interviewing the patient diagnosed with a myocardial infarction. Why is determining the time of symptom onset essential in decision-making?
a. Fibrinolytic drugs will not dissolve clots that are
older than 6 hours.
b. Clots that are older than 6 hours are too large and
tightly meshed.
c. Tissue that is anoxic for more than 6 hours is unlikely
to benefit.
d. After 6 hours, the patient is more likely to have
excessive bleeding.
c
20. A nurse cares for a client who has a pleural chest tube. Which action should the nurse take to ensure safe use of this equipment?
a. Strip the tubing to minimize clot formation and ensure patency.
b. Secure tubing junctions with clamps to prevent accidental disconnections.
c. Connect the chest tube to wall suction at the level prescribed by the provider.
d. Keep padded clamps at the bedside for use if the drainage system is interrupted.
ANS: D
Padded clamps should be kept at the bedside for use if the drainage system becomes dislodged or is interrupted. The nurse should never strip the tubing. Tubing junctions should be taped, not clamped. Wall suction should be set at the level indicated by the device's manufacturer, not the provider.
DIF: Remembering/Knowledge REF: 578
KEY: Drains| postsurgical care
MSC: Integrated Process: Nursing Process: Implementation
NOT: Client Needs Category: Safe and Effective Care Environment: Safety and Infection Control
11. A nurse cares for a client who has kidney stones from secondary hyperoxaluria. Which medication should the nurse anticipate administering?
a. Phenazopyridine (Pyridium)
b. Propantheline (Pro-Banthine)
c. Tolterodine (Detrol LA) d. Allopurinol (Zyloprim)
ANS: D
Stones caused by secondary hyperoxaluria respond to allopurinol (Zyloprim). Phenazopyridine is given to clients with urinary tract infections. Propantheline is an anticholinergic. Tolterodine is an anticholinergic with smooth muscle relaxant properties.
4. A registered nurse (RN) cares for clients on a surgical unit. Which clients should the RN delegate to a licensed practical nurse (LPN)? (Select all that apply.)
a. A 32-year-old who had a radical neck dissection 6 hours ago
b. A 43-year-old diagnosed with cancer after a lung biopsy 2 days ago
c. A 55-year-old who needs discharge teaching after a laryngectomy
d. A 67-year-old who is awaiting preoperative teaching for laryngeal cancer
e. An 88-year-old with esophageal cancer who is awaiting gastric tube placement
ANS: B, E
The nurse can delegate stable clients to the LPN. The client who had a biopsy 2 days ago and the client who is awaiting gastric tube placement are stable. The client who is 6 hours post-surgery is not yet stable. The RN is the only one who can perform discharge and preoperative teaching; teaching cannot be delegated.
DIF: Applying/Application REF: 540
KEY: Interdisciplinary team| delegation MSC: Integrated Process: Teaching/Learning NOT: Client Needs Category: Safe and Effective Care Environment: Management of Care
The home health nurse is reviewing the patient's medication list and sees that new medications were added during a recent hospitalization. In addition, the patient reports he takes a daily low dose of aspirin, but aspirin is not on the final discharge list. Because of the aspirin usage, the nurse is most likely to call the prescribing health care provider for clarification of which type of medication?
a. Vitamin supplement
b. Platelet inhibitor
c. Antihypertensive
d. Erythrocyte stimulating agent
b
16. A nurse cares for a client who is recovering from a myocardial infarction. The client states, I will need to stop eating so much chili to keep that indigestion pain from returning. How should the nurse respond?
a. Chili is high in fat and calories; it would be a good idea to stop eating it.
b. The provider has prescribed an antacid for you to take every morning.
c. What do you understand about what happened to you?
d. When did you start experiencing this indigestion?
C
Clients who experience myocardial infarction often respond with denial, which is a defense mechanism. The nurse should ask the client what he or she thinks happened, or what the illness means to him or her. The other responses do not address the clients misconception about recent pain and the cause of that pain.
What effect can starting a dose of levothyroxine sodium (Synthroid) too high or increasing a dose too rapidly have on a client?
A. Bradycardia and decreased level of consciousness
B. Decreased respiratory rate
C. Hypotension and shock
D. Hypertension and heart failure
D.
Hypertension and heart failure are possible if the levothyroxine sodium dose is started too high or raised too rapidly, because levothyroxine would essentially put the client into a hyperthyroid state. The client would be tachycardic, not bradycardic. The client may have an increased respiratory rate. Shock may develop, but only as a late effect and as the result of "pump failure."
A client has been admitted to the medical intensive care unit with a diagnosis of diabetes insipidus (DI) secondary to lithium overdose. Which medication is used to treat the DI?
A. Desmopressin (DDAVP)
B. Dopamine hydrochloride (Intropin)
C. Prednisone
D. Tolvaptan (Samsca)
A.
Desmopressin is the drug of choice for treatment of severe DI. It may be administered orally, nasally, or by intramuscular or intravenous routes. Dopamine hydrochloride is a naturally occurring catecholamine and inotropic vasopressor; it would not be used to treat DI. Prednisone would not be used to treat DI. Tolvaptan is a selective competitive arginine vasopressin receptor 2 antagonist and is not used with DI.
The nurse is assessing a client with chest pain to evaluate whether the client is suffering from angina or myocardial infarction (MI). Which symptom is indicative of an MI?
a. Chest pain brought on by exertion or stress
b. Substernal chest discomfort occurring at rest
c. Substernal chest discomfort relieved by nitroglycerin or rest
d. Substernal chest pressure relieved only by opioids
d. Substernal chest pressure relieved only by opioids
Substernal chest pressure relieved only by opioids is typically indicative of MI. Substernal chest discomfort that occurs at rest is not necessarily indicative of MI; it could be a sign of unstable angina. Both chest pain brought on by exertion or stress and substernal chest discomfort relieved by nitroglycerin or rest are indicative of angina.
22. A patient has been taught how to care for his central venous catheter at home. Which statements by the patient indicate that further instruction is necessary? (Select all that apply.)
a. "I will flush the catheter with heparin three times a day."
b. "I will change the Luer-Lok cap on each catheter daily."
c. "I will tape the catheter to my skin."
d. "If the catheter lumen breaks or punctures, I will immediately clamp the catheter between myself and the opening."
e. "I will wash my hands before working with the catheter."
a, b
21. A nurse cares for a client who tests positive for alpha1-antitrypsin (AAT) deficiency. The client asks, "What does this mean?" How should the nurse respond?
a. "Your children will be at high risk for the development of chronic obstructive
pulmonary disease."
b. "I will contact a genetic counselor to discuss your condition."
c. "Your risk for chronic obstructive pulmonary disease is higher, especially if you
smoke."
d. "This is a recessive gene and should have no impact on your health."
ANS: C
The gene for AAT is a recessive gene. Clients with only one allele produce enough AAT to prevent chronic obstructive pulmonary disease (COPD) unless the client smokes. A client with two alleles is at high risk for COPD even if not exposed to smoke or other irritants. The client is a carrier, and children may or may not be at high risk depending on the partner's AAT levels. Contacting a genetic counselor may be helpful but does not address the client's current question.
DIF: Applying/Application REF: 558
KEY: Gene| allele| health screening| a1AT (alpha1-antitrypsin) gene
MSC: IntegratedProcess:Teaching/Learning
NOT: Client Needs Category: Safe and Effective Care Environment: Management of Care
11. Which personal action is most likely to cause the kidneys to produce and release erythropoietin?
a. Person moves to a low desert area where the humidity is very low.
b. Person moves to a high-altitude area where atmospheric oxygen is low.
c. Person drinks an excessive amount of fluid that results in fluid overload.
d.d Person eats a large high-protein meal after a rigorous exercise workout.
b
12. A client is taking furosemide (Lasix) 40 mg/day for management of chronic kidney disease (CKD). To detect the positive effect of the medication, what action of the nurse is best?
a. Obtain daily weights of the client.
b. Auscultate heart and breath sounds.
c. Palpate the client's abdomen. d. Assess the client's diet history.
ANS: A
Furosemide (Lasix) is a loop diuretic that helps reduce fluid overload and hypertension in clients with early stages of CKD. One kilogram of weight equals about 1 liter of fluid retained in the client, so daily weights are necessary to monitor the response of the client to the medication. Heart and breath sounds should be assessed if there is fluid retention, as in heart failure. Palpation of the client's abdomen is not necessary, but the nurse should check for edema. The diet history of the client would be helpful to assess electrolyte replacement since potassium is lost with this diuretic, but this does not assess the effect of the medication.
Question 5 of 41
The nurse assesses the client with which hematologic problem first?
a. A 32-year-old with pernicious anemia who needs a vitamin B12 injection
b. A 40-year-old with iron deficiency anemia who needs a Z-track iron injection
c. A 67-year-old with acute myelocytic leukemia with petechiae on both legs
d. An 81-year-old with thrombocytopenia and an increase in abdominal girth
d
An 81-year-old with thrombocytopenia and an increase in abdominal girth
An increase in abdominal girth in a client with thrombocytopenia indicates possible hemorrhage; this warrants further assessment immediately. The 32-year-old with pernicious anemia, the 40-year-old with iron deficiency anemia, and the 67-year-old with acute myelocytic leukemia do not indicate any acute complications, so the nurse can assess them after assessing the client with thrombocytopenia.
The nurse is assessing a client with chest pain to evaluate whether the client is suffering from angina or myocardial infarction (MI). Which symptom is indicative of an MI?
Chest pain brought on by exertion or stress
Substernal chest discomfort occurring at rest
Substernal chest discomfort relieved by nitroglycerin or rest
Substernal chest pressure relieved only by opioids
Substernal chest pressure relieved only by opioids
Substernal chest pressure relieved only by opioids is typically indicative of MI. Substernal chest discomfort that occurs at rest is not necessarily indicative of MI; it could be a sign of unstable angina. Both chest pain brought on by exertion or stress and substernal chest discomfort relieved by nitroglycerin or rest are indicative of angina.
8. The nurse is performing an assessment of an adult patient with new-onset acromegaly. What does the nurse expect to find?
a. Extremely long arms and legs
b. Thickened lips
c. Changes in menses with infertility
d. Rough, extremely dry skin
b
Thickened lips
18. A student nurse is preparing to administer enoxaparin (Lovenox) to a client. What action by the student requires immediate intervention by the supervising nurse?
a. Assessing the client's platelet count
b. Choosing an 18-gauge, 2-inch needle
c. Not aspirating prior to injection
d. Swabbing the injection site with alcohol
ANS: B
Enoxaparin is given subcutaneously, so the 18-gauge, 2-inch needle is too big. The other actions are appropriate.
16. A client has presented to the emergency department with an acute myocardial infarction (MI). What action by the nurse is best to meet The Joint Commission's Core Measures outcomes?
a. Obtain an electrocardiogram (ECG) now and in the morning.
b. Give the client an aspirin.
c. Notify the Rapid Response Team.
d. Prepare to administer thrombolytics.
ANS: B
The Joint Commission's Core Measures set for acute MI require that aspirin is administered when a client with MI presents to the emergency department or when an MI occurs in the hospital. A rapid ECG is vital, but getting another one in the morning is not part of the Core Measures set. The Rapid Response Team is not needed if an emergency department provider is available. Thrombolytics may or may not be needed.
DIF: Remembering/Knowledge REF: 766
KEY: Coronary artery disease| Core Measures| The Joint Commission
MSC: Integrated Process: Nursing Process: Implementation
NOT: Client Needs Category: Safe and Effective Care Environment: Management of Care
8. Which statements about hormones and the endocrine system are accurate? (Select all that apply.)
a. There are specific normal blood levels of each hormone.
b. Hormones exert their effects on specific target tissues.
c. Each hormone can bind with multiple receptor sites.
d. The endocrine system works independently to regulate homeostasis.
e. More than one hormone can be stimulated before the target tissue is affected.
a, b, e
There are specific normal blood levels of each hormone.
Hormones exert their effects on specific target tissues.
More than one hormone can be stimulated before the target tissue is affected.
A client has a sickle cell crisis with extreme lower extremity pain. What comfort measure does the nurse delegate to the unlicensed assistive personnel (UAP)?
a. Apply ice packs to the client's legs.
b. Elevate the client's legs on pillows.
c. Keep the lower extremities warm.
d. Place elastic bandage wraps on the client's legs.
c. Keep the lower extremities warm.
18. A client has the diagnosis of "valley fever" accompanied by myalgias and arthralgias. What treatment should the nurse educate the client on?
a. Intravenous amphotericin B
b. Long-term anti-inflammatories
c. No specific treatment
d. Oral fluconazole (Diflucan)
ANS: D
"Valley fever," or coccidioidomycosis, is a fungal infection. Many people do not need treatment and the disease resolves on its own. However, the presence of joint and muscle pain indicates a moderate infection that needs treatment with antifungal medications. IV amphotericin is reserved for pregnant women and those with severe infection. Anti-inflammatory medications may be used to treat muscle aches and pain but are not used long term.
12. A nurse assesses a client who is recovering from extracorporeal shock wave lithotripsy for renal calculi. The nurse notes an ecchymotic area on the client's right lower back. Which action should the nurse take?
a. Administer fresh-frozen plasma.
b. Apply an ice pack to the site.
c. Place the client in the prone position. d. Obtain serum coagulation test results.
ANS: B
The shock waves from lithotripsy can cause bleeding into the tissues through which the waves pass. Application of ice can reduce the extent and discomfort of the bruising. Although coagulation test results and fresh-frozen plasma are used to assess and treat bleeding disorders, ecchymosis after this procedure is not unusual and does not warrant a higher level of intervention. Changing the client's position will not decrease bleeding.
The nurse notices a visitor walking into the room of a client on airborne isolation with no protective gear. What does the nurse do?
A) Ensures that the client is wearing a mask
B) Tells the visitor that the client cannot receive visitors at this time
C) Provides a particulate air respirator to the visitor
D) Provides a mask to the visitor
(Chp. 31; elsevier resources)
D) Provides a mask to the visitor
(Chp. 31; elsevier resources)
A client admitted for sickle cell crisis is distraught after learning her child also has the disease. What response by the nurse is best?
a. "Both you and the father are equally responsible for passing it on."
b. "I can see you are upset. I can stay here with you a while if you like."
c. "It's not your fault; there is no way to know who will have this disease."
d. "There are many good treatments for sickle cell disease these days."
b. "I can see you are upset. I can stay here with you a while if you like."
17. After teaching a client who is recovering from a heart transplant to change positions slowly, the client asks, "Why is this important?" How should the nurse respond?
a. "Rapid position changes can create shear and friction forces, which can tear out
your internal vascular sutures."
b. "Your new vascular connections are more sensitive to position changes, leading to
increased intravascular pressure and dizziness."
c. "Your new heart is not connected to the nervous system and is unable to respond to
decreases in blood pressure caused by position changes."
d. "While your heart is recovering, blood flow is diverted away from the brain,
increasing the risk for stroke when you stand up."
ANS: C
Because the new heart is denervated, the baroreceptor and other mechanisms that compensate for blood pressure drops caused by position changes do not function. This allows orthostatic hypotension to persist in the postoperative period. The other options are false statements and do not correctly address the client's question.
DIF: Understanding/Comprehension REF: 703
KEY: Transplant| patient education
MSC: IntegratedProcess:NursingProcess:Implementation
NOT: Client Needs Category: Physiological Integrity: Physiological Adaptation
A nurse cares for a client who possibly has syndrome of inappropriate antidiuretic hormone (SIADH). The clients serum sodium level is 114 mEq/L. Which action should the nurse take first?
a. Consult with the dietitian about increased dietary sodium.
b. Restrict the clients fluid intake to 600 mL/day.
c. Handle the client gently by using turn sheets for re-positioning.
d. Instruct unlicensed assistive personnel to measure intake and output.
B (With SIADH, clients often have dilutional hyponatremia. The client needs a fluid restriction, sometimes to as little as 500 to 600 mL/24 hr. Adding sodium to the clients diet will not help if he or she is retaining fluid and diluting the sodium. The client is not at increased risk for fracture, so gentle handling is not an issue. The client should be on intake and output; however, this will monitor only the clients intake, so it is not the best answer. Reducing intake will help increase the clients sodium.)
12. A nurse assesses a client who is recovering from extracorporeal shock wave lithotripsy for renal calculi. The nurse notes an ecchymotic area on the clients right lower back. Which action should the nurse take?
a. Administer fresh-frozen plasma.
b. Apply an ice pack to the site.
c. Place the client in the prone position.
d. Obtain serum coagulation test results.
B
The shock waves from lithotripsy can cause bleeding into the tissues through which the waves pass. Application of ice can reduce the extent and discomfort of the bruising. Although coagulation test results and fresh-frozen plasma are used to assess and treat bleeding disorders, ecchymosis after this procedure is not unusual and does not warrant a higher level of intervention. Changing the clients position will not decrease bleeding.
12. A nurse provides health screening for a community health center with a large population of African-American clients. Which priority assessment should the nurse include when working with this population?
a. Measure height and weight.
b. Assess blood pressure.
c. Observe for any signs of abuse. d. Ask about medications.
ANS: B
All interventions are important for the visiting nurse to accomplish. However, African Americans have a high rate of hypertension leading to end-stage renal disease. Each encounter that the nurse has with an African-American client provides a chance to detect hypertension and treat it. If the client is already on antihypertensive medication, assessing blood pressure monitors therapy.
The nurse is preparing a client for discharge who has undergone percutaneous needle aspiration of a peritonsillar abscess. Which is most important to teach the client about follow-up care?
A) Completing the antibiotic medication regimen
B) Taking pain medications every 4 to 6 hours
C) Contacting the provider if the throat feels more swollen
D) Using warm saline gargles and irrigations
(Chp. 31; elsevier resources)
C) Contacting the provider if the throat feels more swollen
(Chp. 31; elsevier resources)
Question 6 of 41
A client who is receiving a blood transfusion suddenly exclaims to the nurse, "I don't feel right!" What does the nurse do next?
a. Call the Rapid Response Team.
b. Obtain vital signs and continue to monitor.
c. Slow the infusion rate of the transfusion.
d. Stop the transfusion.
d
Stop the transfusion.
The client may be experiencing a transfusion reaction; the nurse should stop the transfusion immediately. Calling the Rapid Response Team or obtaining vital signs is not the first thing that should be done. The nurse should not slow the infusion rate, but should stop it altogether.
18. A nurse prepares a client for coronary artery bypass graft surgery. The client states, I am afraid I might die. How should the nurse respond?
a. This is a routine surgery and the risk of death is very low.
b. Would you like to speak with a chaplain prior to surgery?
c. Tell me more about your concerns about the surgery.
d. What support systems do you have to assist you?
C
The nurse should discuss the clients feelings and concerns related to the surgery. The nurse should not provide false hope or push the clients concerns off on the chaplain. The nurse should address support systems after addressing the clients current issue.
The laboratory results for a 53-year-old patient indicate a low T3 level and elevated thyroidstimulating hormone (TSH). What do these results indicate?
a. Hyperthyroidism
b. Hypothyroidism
c. Malfunctioning pituitary gland
d. Normal laboratory values for this age
b
17. A nurse is caring for four clients. Which client should the nurse assess first? a. Client with an acute myocardial infarction, pulse 102 beats/min
b. Client who is 1 hour post angioplasty, has tongue swelling and anxiety c. Client who is post coronary artery bypass, chest tube drained 100 mL/hr d. Client who is post coronary artery bypass, potassium 4.2 mEq/L
ANS: B
The post-angioplasty client with tongue swelling and anxiety is exhibiting manifestations of an allergic reaction that could progress to anaphylaxis. The nurse should assess this client first. The client with a heart rate of 102 beats/min may have increased oxygen demands but is just over the normal limit for heart rate. The two post coronary artery bypass clients are stable.
DIF: Analyzing/Analysis REF: 774
KEY: Coronary artery disease| critical rescue| medical emergencies| hypersensitivities| allergic reaction MSC: Integrated Process: Nursing Process: Analysis
NOT: Client Needs Category: Safe and Effective Care Environment: Management of Care
A nurse teaches a client who has been prescribed a 24-hour urine collection to measure excreted hormones. The client asks, "Why do I need to collect urine for 24 hours instead of providing a random specimen?" How should the nurse respond?
a. "This test will assess for a hormone secreted on a circadian rhythm."
b. "The hormone is diluted in urine; therefore, we need a large volume."
c. "We are assessing when the hormone is secreted in large amounts."
d. "To collect the correct hormone, you need to urinate multiple times."
ANS: A
Some hormones are secreted in a pulsatile, or circadian, cycle. When testing for these substances, a collection that occurs over 24 hours will most accurately reflect hormone secretion. Dilution of hormones in urine, secretion of hormone amounts, and ability to collect the correct hormone are not reasons to complete a 24-hour urine test.
23. The nurse has instructed a patient at risk for bleeding about techniques to manage this condition. Which statements by the patient indicate that teaching has been successful? (Select all that apply.)
a. "I will take a stool softener to prevent straining during a bowel movement."
b. "I won't take aspirin or aspirin-containing products."
c. "I won't participate in any contact sports.."
d. "I will report a headache that is not responsive to acetaminophen."
e. "I will avoid bending over at the waist."
f. "If I am bumped, I will apply ice to the site for at least 10 minutes."
a, b, c, d, e
12. The nurse is teaching a man about how to prevent UTIs. What information does the nurse include?
a. "Have a minimal fluid intake of 5 L daily, unless contraindicated.
b. "Empty your bladder before and after sexual intercourse.
c. "Make sure that spermicides are used with condoms."
d. "Gently wash the genital area before intercourse."
d
19. A client is in the family medicine clinic reporting a dry, sore throat. The provider asks the nurse to assess for odynophagia. What assessment technique is most appropriate?
a. Ask the client what foods cause trouble swallowing.
b. Assess the client for pain when swallowing.
c. Determine if the client can swallow saliva. d. Palpate the client's jaw while swallowing.
ANS: B
Odynophagia is painful swallowing. The nurse should assess the client for this either by asking or by having the client attempt to drink water. It is not related to specific foods and is not assessed by palpating the jaw. Being unable to swallow saliva is not odynophagia, but it would be a serious situation.
12. Vitamin D is converted to its active from in the kidney. If this function fails, which electrolyte imbalance will occur?
a. Hyperkalemia
b. Hypocalcemia
c. Hypernatremia
d. Hypoglycemia
b
19. A client in the emergency department has several broken ribs. What care measure will best promote comfort?
a. Allowing the client to choose the position in bed
b. Humidifying the supplemental oxygen c. Offering frequent, small drinks of water d. Providing warmed blankets
ANS: A
Allow the client with respiratory problems to assume a position of comfort if it does not interfere with care. Often the client will choose a more upright position, which also improves oxygenation. The other options are less effective comfort measures.
A patient has a suspected hematologic problem. Which instruction is the nurse most likely to give to the UAP?
a. Record urine output for the shift.
b. Take the vital signs every 2 hours.
c. Assess the patient for fatigue after exertion.
d. Handle the patient gently to avoid bruising.
d
17. A nurse supervises an unlicensed assistive personnel (UAP) applying electrocardiographic monitoring. Which statement should the nurse provide to the UAP related to this procedure?
a. "Clean the skin and clip hairs if needed."
b. "Add gel to the electrodes prior to applying them."
c. "Place the electrodes on the posterior chest."
d. "Turn off oxygen prior to monitoring the client."
ANS: A
To ensure the best signal transmission, the skin should be clean and hairs clipped. Electrodes should be placed on the anterior chest, and no additional gel is needed. Oxygen has no impact on electrocardiographic monitoring.
DIF: Remembering/Knowledge REF: 652
KEY: Assessment/diagnostic examination| interdisciplinary team| unlicensed assistive personnel (UAP) MSC: Integrated Process: Communication and Documentation NOT: Client Needs Category: Safe and Effective Care Environment: Management of Care
Ch.62 p. 1276, Patient-Centered Care; Quality Improvement; Safety
The patient is a 32-year-old woman admitted to your unit after surgery for fractures of the left arm and leg resulting from a car crash. She is awake and able to verify her medical history of rheumatoid arthritis and her usual daily medications. These are 10 mg of prednisone, naproxen 800 mg twice daily, oral contraceptives, calcium 600 mg, and one multiple vitamin tablet. All of these are prescribed for her to receive during her hospitalization. She is concerned about pain management and how long the recovery will be for the fractures. She is friendly, somewhat anxious, asks many questions, and wants to do "her part" to ensure good recovery. Over the next 4 days, she has become quieter, mumbles that her head and stomach hurt, and now does not recognize the assistant who has been providing her daily care. When she receives her medications, she has difficulty picking them up. The nursing assistant remarks that taking her pulse is difficult because it is so slow and irregular. When you assess her, she is so weak that she is unable to lift her arm for a blood pressure check. Her blood pressure is 92/50, which is down from the 128/84 reading on admission. You also verify that her heart beat is slow and irregular.
4. What specifically would be the nurse's role in preventing this problem?
This type of adrenal insufficiency develops over a period of days. Monitoring trends for level of consciousness, blood pressure, and heart rate and rhythm should be something all nurses do on every patient. It is very likely that changes were present earlier and not recognized.
12. The nurse is caring for several patients on a medical-surgical unit. None of the patient currently has any acute or chronic kidney problems. Which patient has the greatest risk to develop AKI?
a. 73-year-old male who has hypertension and peripheral vascular disease
b. 32-year-old female who is pregnant and has gestational diabetes
c. 49-year-old male who is obese and has a history of skin cancer
d. 23-year-old female who has been treated for a urinary tract infection
a
22. A nurse cares for a female client who has a family history of cystic fibrosis. The client asks, "Will my children have cystic fibrosis?" How should the nurse respond?
a. "Since many of your family members are carriers, your children will also be
carriers of the gene."
b. "Cystic fibrosis is an autosomal recessive disorder. If you are a carrier, your
children will have the disorder."
c. "Since you have a family history of cystic fibrosis, I would encourage you and
your partner to be tested."
d. "Cystic fibrosis is caused by a protein that controls the movement of chloride.
Adjusting your diet will decrease the spread of this disorder."
ANS: C
Cystic fibrosis is an autosomal recessive disorder in which both gene alleles must be mutated for the disorder to be expressed. The nurse should encourage both the client and partner to be tested for the abnormal gene. The other statements are not true.
DIF: Applying/Application REF: 567
KEY: Gene| allele| health screening MSC: Integrated Process: Teaching/Learning NOT: Client Needs Category: Safe and Effective Care Environment: Management of Care
A client with sickle cell disease (SCD) takes hydroxyurea (Droxia). The client presents to the clinic reporting an increase in fatigue. What laboratory result should the nurse report immediately?
a. Hematocrit: 25%
b. Hemoglobin: 9.2 mg/dL
c. Potassium: 3.2 mEq/L
d. White blood cell count: 38,000/mm3
d. White blood cell count: 38,000/mm3
A client comes to the emergency department with chest discomfort. Which action does the nurse perform first?
Administers oxygen therapy
Obtains the client's description of the chest discomfort
Provides pain relief medication
Remains calm and stays with the client
Obtains the client's description of the chest discomfort
A description of the chest discomfort must be obtained first, before further action can be taken. Neither oxygen therapy nor pain medication is the first priority in this situation; an assessment is needed first. Remaining calm and staying with the client are important, but are not matters of highest priority.
A nurse teaches a client about self-monitoring of blood glucose levels. Which statement should the nurse include in this client's teaching to prevent bloodborne infections?
a. "Wash your hands after completing each test."
b. "Do not share your monitoring equipment."
c. "Blot excess blood from the strip with a cotton ball."
d. "Use gloves when monitoring your blood glucose."
ANS: B
Small particles of blood can adhere to the monitoring device, and infection can be transported from one user to another. Hepatitis B in particular can survive in a dried state for about a week. The client should be taught to avoid sharing any equipment, including the lancet holder. The client should be taught to wash his or her hands before testing. The client would not need to blot excess blood away from the strip or wear gloves.
A client comes to the emergency department with chest discomfort. Which action does the nurse perform first?
a. Administers oxygen therapy
b. Obtains the client's description of the chest discomfort
c. Provides pain relief medication
d. Remains calm and stays with the client
b. Obtains the client's description of the chest discomfort
A description of the chest discomfort must be obtained first, before further action can be taken. Neither oxygen therapy nor pain medication is the first priority in this situation; an assessment is needed first. Remaining calm and staying with the client are important, but are not matters of highest priority.
Ch.61 Question 8 of 16
Which gland releases catecholamines?
a. Adrenal
b. Pancreas
c. Parathyroid
d. Thyroid
a
Adrenal
The adrenal medulla releases catecholamines in response to stimulation of the sympathetic nervous system. The principal hormones of the pancreas are insulin, glucagon, and somatostatin. Parathyroid hormone is the principal hormone of the parathyroid gland. Triiodothyronine (T3), thyroxine (T4), and calcitonin are the principal hormones of the thyroid.
5. A nurse teaches a client who is being discharged after a fixed centric occlusion for a mandibular fracture. Which statements should the nurse include in this client's teaching? (Select all that apply.)
a. "You will need to cut the wires if you start vomiting."
b. "Eat six soft or liquid meals each day while recovering."
c. "Irrigate your mouth every 2 hours to prevent infection."
d. "Sleep in a semi-Fowler's position after the surgery."
e. "Gargle with mouthwash that contains Benadryl once a day."
ANS: A, B, C, D
The client needs to know how to cut the wires in case of emergency. If the client vomits, he or she may aspirate. The client should also be taught to eat soft or liquid meals multiple times a day, irrigate the mouth with a Waterpik to prevent infection, and sleep in a semi-Fowler's position to assist in avoiding aspiration. Mouthwash with Benadryl is used for clients who have mouth pain after radiation treatment; it is not used to treat pain in a client with a mandibular fracture.
DIF: Applying/Application REF: 534
KEY: Surgical care| aspiration precautions
MSC: IntegratedProcess:Teaching/Learning
NOT: Client Needs Category: Physiological Integrity: Reduction of Risk Potential
Question 7 of 41
The nurse is caring for a client with sickle cell disease. Which action is most effective in reducing the potential for sepsis in this client?
a. Administering prophylactic drug therapy
b. Frequent and thorough handwashing
c. Monitoring laboratory values to look for abnormalities
d. Taking vital signs every 4 hours, day and night
b
Frequent and thorough handwashing
Prevention and early detection strategies are used to protect the client in sickle cell crisis from infection. Frequent and thorough handwashing is of the utmost importance. Drug therapy is a major defense against infections that develop in the client with sickle cell disease, but is not the most effective way that the nurse can reduce the potential for sepsis. Continually assessing the client for infection and monitoring the daily complete blood count with differential white blood cell count is early detection, not prevention. Taking vital signs every 4 hours will help with early detection of infection, but is not prevention.
12. The nurse is reviewing laboratory results for a patient with PKD. Which laboratory abnormality indicates glomeruli involvement?
a. Low specific gravity of urine
b. Bacteria in urine
c. Proteinuria
d. Hematuria
c
12. A nurse reviews the allergy list of a client who is scheduled for an intravenous urography. Which client allergy should alert the nurse to urgently contact the health care provider?
a. Seafood
b. Penicillin
c. Bee stings
d. Red food dye
ANS: A
Clients with seafood allergies often have severe allergic reactions to the standard dyes used during intravenous urography. The other allergies have no impact on the client's safety during an intravenous urography.
6. A nurse is assessing clients on a rehabilitation unit. Which clients are at greatest risk for asphyxiation related to inspissated oral and nasopharyngeal secretions? (Select all that apply.)
a. A 24-year-old with a traumatic brain injury
b. A 36-year-old who fractured his left femur
c. A 58-year-old at risk for aspiration following radiation therapy
d. A 66-year-old who is a quadriplegic and has a sacral ulcer
e. An 80-year-old who is aphasic after a cerebral vascular accident
ANS: A, C, D, E
Risk for asphyxiation related to inspissated oral and nasopharyngeal secretions is caused by poor oral hygiene. Clients at risk include those with altered mental status and level of consciousness (traumatic brain injury), dehydration, an inability to communicate (aphasic) and cough effectively (quadriplegic), and a risk of aspiration (aspiration precautions). The client with a fractured femur is at risk for a pulmonary embolism.
DIF: Applying/Application REF: 536
KEY: Medical emergencies
MSC: IntegratedProcess:NursingProcess:Assessment
NOT: Client Needs Category: Safe and Effective Care Environment: Management of Care
24. The new registered nurse is giving a blood transfusion to a patient. Which statement by the new nurse indicates the need for action by the supervising nurse?
a. "I will complete red blood cell transfusion within 6 hours."
b. "I will check the patient verification with another registered nurse."
c. "I will use normal saline solution to dilute the blood."
d. "I will remain with the patient for the first 15 to 30 minutes of the infusion."
a
18. A nurse is caring for a client who is intubated and has an intra-aortic balloon pump. The client is restless and agitated. What action should the nurse perform first for comfort?
a. Allow family members to remain at the bedside.
b. Ask the family if the client would like a fan in the room.
c. Keep the television tuned to the client's favorite channel.
d. Speak loudly to the client in case of hearing problems.
ANS: A
Allowing the family to remain at the bedside can help calm the client with familiar voices (and faces if the client wakes up). A fan might be helpful but may also spread germs through air movement. The TV should not be kept on all the time to allow for rest. Speaking loudly may agitate the client more.
DIF: Applying/Application REF: 780
KEY: Intra-aortic balloon pump| nonpharmacologic comfort measures MSC: Integrated Process: Nursing Process: Implementation
NOT: Client Needs Category: Physiological Integrity: Basic Care and Comfort
8. The laboratory results for a 53-year-old patient indicate a low T3 level and elevated thyroid-stimulating hormone (TSH). What do these results indicate?
a. Hyperthyroidism
b. Hypothyroidism
c. Malfunctioning pituitary gland
d. Normal laboratory values for this age
b
13. A nurse cares for a client with diabetes mellitus who is prescribed metformin (Glucophage) and is scheduled for an intravenous urography. Which action should the nurse take first?
a. Contact the provider and recommend discontinuing the metformin.
b. Keep the client NPO for at least 6 hours prior to the examination.
c. Check the client's capillary artery blood glucose and administer prescribed insulin. d. Administer intravenous fluids to dilute and increase the excretion of dye.
ANS: A
Metformin can cause lactic acidosis and renal impairment as the result of an interaction with the dye. This drug must be discontinued for 48 hours before the procedure and not started again after the procedure until urine output is well established. The client's health care provider needs to provide alternative therapy for the client until the metformin can be resumed. Keeping the client NPO, checking the client's blood glucose, and administering intravenous fluids should be part of the client's plan of care, but are not the priority, as the examination should not occur while the client is still taking metformin.
23. A nurse administers medications to a client who has asthma. Which medication classification is paired correctly with its physiologic response to the medication?
a. Bronchodilator - Stabilizes the membranes of mast cells and prevents the release
of inflammatory mediators
b. Cholinergic antagonist - Causes bronchodilation by inhibiting the parasympathetic
nervous system
c. Corticosteroid - Relaxes bronchiolar smooth muscles by binding to and activating
pulmonary beta2 receptors
d. Cromone - Disrupts the production of pathways of inflammatory mediators
ANS: B
Cholinergic antagonist drugs cause bronchodilation by inhibiting the parasympathetic nervous system. This allows the sympathetic nervous system to dominate and release norepinephrine that actives beta2 receptors. Bronchodilators relax bronchiolar smooth muscles by binding to and activating pulmonary beta2 receptors. Corticosteroids disrupt the production of pathways of inflammatory mediators. Cromones stabilize the membranes of mast cells and prevent the release of inflammatory mediators.
DIF: Remembering/Knowledge REF: 554 KEY: Medications MSC: Integrated Process: Nursing Process: Analysis
NOT: Client Needs Category: Physiological Integrity: Pharmacological and Parenteral Therapies
A patient is diagnosed with iron deficiency anemia. Which assessment finding is the nurse most likely to observe in this patient?
a. Neck veins are distended and edema is present.
b. Lower extremities show signs of phlebitis.
c. Systolic blood pressure is lower than normal.
d. Palpation of ribs or sternum elicits tenderness.
c
Which statement by a client scheduled for a percutaneous transluminal coronary angioplasty (PTCA) indicates a need for further preoperative teaching?
"I will be awake during this procedure."
"I will have a balloon in my artery to widen it."
"I must lie still after the procedure."
"My angina will be gone for good."
"My angina will be gone for good."
Reocclusion is possible after PTCA. The client is typically awake, but drowsy, during this procedure. PTCA uses a balloon to widen the artery, and the client will have to lie still after the procedure because of the large-bore venous access. Time is necessary to allow the hole to heal and prevent hemorrhage.
8. A nurse cares for a client who has hypothyroidism as a result of Hashimoto's thyroiditis. The client asks, "How long will I need to take this thyroid medication?" How should the nurse respond?
a. "You will need to take the thyroid medication until the goiter is completely gone."
b. "Thyroiditis is cured with antibiotics. Then you won't need thyroid medication."
c. "You'll need thyroid pills for life because your thyroid won't start working again."
d. "When blood tests indicate normal thyroid function, you can stop the medication."
c. Hashimoto's thyroiditis results in a permanent loss of thyroid function. The client will need lifelong thyroid replacement therapy. The client will not be able to stop taking the medication.
A nurse teaches a client who has been prescribed a 24-hour urine collection to measure excreted hormones. The client asks, "Why do I need to collect urine for 24 hours instead of providing a random specimen?" How should the nurse respond?
a. "This test will assess for a hormone secreted on a circadian rhythm."
b. "The hormone is diluted in urine; therefore, we need a large volume."
c. "We are assessing when the hormone is secreted in large amounts."
d. "To collect the correct hormone, you need to urinate multiple times."
ANS: A
Some hormones are secreted in a pulsatile, or circadian, cycle. When testing for these substances, a collection that occurs over 24 hours will most accurately reflect hormone secretion. Dilution of hormones in urine, secretion of hormone amounts, and ability to collect the correct hormone are not reasons to complete a 24-hour urine test.
Which statement by a client scheduled for a percutaneous transluminal coronary angioplasty (PTCA) indicates a need for further preoperative teaching?
a. "I will be awake during this procedure."
b. "I will have a balloon in my artery to widen it."
c. "I must lie still after the procedure."
d. "My angina will be gone for good."
d. "My angina will be gone for good."
Reocclusion is possible after PTCA. The client is typically awake, but drowsy, during this procedure. PTCA uses a balloon to widen the artery, and the client will have to lie still after the procedure because of the large-bore venous access. Time is necessary to allow the hole to heal and prevent hemorrhage.
A nurse is caring for four clients with leukemia. After hand-off report, which client should the nurse see first?
a. Client who had two bloody diarrhea stools this morning
b. Client who has been premedicated for nausea prior to chemotherapy
c. Client with a respiratory rate change from 18 to 22 breaths/min
d. Client with an unchanged lesion to the lower right lateral malleolus
a. Client who had two bloody diarrhea stools this morning
20. A client has been diagnosed with a very large pulmonary embolism (PE) and has a dropping blood pressure. What medication should the nurse anticipate the client will need as the priority?
a. Alteplase (Activase)
b. Enoxaparin (Lovenox)
c. Unfractionated heparin
d. Warfarin sodium (Coumadin)
ANS: A
Activase is a "clot-busting" agent indicated in large PEs in the setting of hemodynamic instability. The nurse knows this drug is the priority, although heparin may be started initially. Enoxaparin and warfarin are not indicated in this setting.
13. A patient is suspected of having PKD. Which diagnostic study has minimal risk and can reveal PKD?
a. Kidneys-ureters-bladder (KUB) x-ray
b. Urography
c. Renal sonography
d. Renal angiography
c
20. A client is admitted with suspected pneumonia from the emergency department. The client went to the primary care provider a "few days ago" and shows the nurse the results of what the client calls "an allergy test," as shown below:
What action by the nurse takes priority?
a. Assess the client for possible items to which he or she is allergic. b. Call the primary care provider's office to request records.
c. Immediately place the client on Airborne Precautions.
d. Prepare to begin administration of intravenous antibiotics.
ANS: C
This "allergy test" is actually a positive tuberculosis test. The client should be placed on Airborne Precautions immediately. The other options do not take priority over preventing the spread of the disease.
Question 8 of 41
Which client does the nurse assign as a roommate for the client with aplastic anemia?
a. A 23-year-old with sickle cell disease who has two draining leg ulcers
b. A 28-year-old with glucose-6-phosphate dehydrogenase (G6PD) deficiency anemia who is receiving mannitol (Osmitrol)
c. A 30-year-old with leukemia who is receiving induction chemotherapy
d. A 34-year-old with idiopathic thrombocytopenia who is taking steroids
b
A 28-year-old with glucose-6-phosphate dehydrogenase (G6PD) deficiency anemia who is receiving mannitol (Osmitrol)
Because clients with aplastic anemia usually have low white blood cell counts that place them at high risk for infection, roommates such as the client with G6PD deficiency anemia should be free from infection or infection risk. The client with sickle cell disease has two draining leg ulcer infections that would threaten the diminished immune system of the client with aplastic anemia. The client with leukemia who is receiving induction chemotherapy and the client with idiopathic thrombocytopenia who is taking steroids are at risk for development of infection, which places the client with aplastic anemia at risk, too.
18. A nurse assesses a client's electrocardiogram (ECG) and observes the reading shown below:
How should the nurse document this client's ECG strip?
a. Ventricular tachycardia
b. V entricular fibrillation
c. Sinus rhythm with premature atrial contractions (PACs)
d. Sinus rhythm with premature ventricular contractions (PVCs)
ANS: D
Sinus rhythm with PVCs has an underlying regular sinus rhythm with ventricular depolarization that sometimes precede atrial depolarization. Ventricular tachycardia and ventricular fibrillation rhythms would not have sinus beats present. Premature atrial contractions are atrial contractions initiated from another region of the atria before the sinus node initiates atrial depolarization.
DIF: Applying/Application REF: 669
KEY: Cardiac electrical conduction| documentation
MSC: Integrated Process: Communication and Documentation
NOT: Client Needs Category: Physiological Integrity: Physiological Adaptation
Question 6 of 16
A client is hospitalized for pituitary function testing. Which nursing action included in the client's plan of care will be most appropriate for the RN to delegate to the LPN/LVN?
a. Assess the client for clinical manifestations of hypopituitarism.
b. Inject regular insulin for the growth hormone stimulation test.
c. Palpate the thyroid gland for size and firmness.
d. Teach the client about the adrenocorticotropic hormone stimulation test.
b
Inject regular insulin for the growth hormone stimulation test.
Injection of insulin is within the LPN/LVN scope of practice. Client assessment for clinical manifestations of hypopituitarism, palpating the thyroid gland, and client education are complex skills requiring training and expertise, and are best performed by an RN.
13. For a patient with AKI, the nurse would consider questioning the order for which diagnostic test?
a. Kidney biopsy
b. Ultrasonography
c. Computed tomography with contrast dye
d. Kidney, ureter, bladder (KUB) x-ray
c
18. A nurse teaches a client recovering from a heart transplant who is prescribed cyclosporine (Sandimmune). Which statement should the nurse include in this client's discharge teaching?
a. "Use a soft-bristled toothbrush and avoid flossing."
b. "Avoid large crowds and people who are sick."
c. "Change positions slowly to avoid hypotension."
d. "Check your heart rate before taking the medication."
ANS: B
These agents cause immune suppression, leaving the client more vulnerable to infection. The medication does not place the client at risk for bleeding, orthostatic hypotension, or a change in heart rate.
DIF: Applying/Application REF: 703 KEY: Transplant| immune suppressant
MSC: Integrated Process: Nursing Process: Implementation
NOT: Client Needs Category: Physiological Integrity: Pharmacological and Parenteral Therapies
A client is taking methimazole (Tapazole) for hyperthyroidism and would like to know how soon this medication will begin working. What is the nurse's best response?
A. "You should see effects of this medication immediately."
B. "You should see effects of this medication within 1 week."
C. "You should see full effects from this medication within 1 to 2 days."
D. "You should see some effects of this medication within 2 weeks."
D.
Methimazole is an iodine preparation that decreases blood flow through the thyroid gland. This action reduces the production and release of thyroid hormone. The client should see some effects within 2 weeks; however, it may take several more weeks before metabolism returns to normal. Although onset of action is 30 to 40 minutes after an oral dose, the client will not see effects immediately. Effects will take longer than 1 week to become apparent when methimazole is used. Methimazole needs to be taken every 8 hours for an extended period of time. Levels of triiodothyronine (T3) and thyroxine (T4) will be monitored and dosages adjusted as levels fall.
25. The new registered nurse is identifying a patient for blood transfusion. Which action by the new nurse warrants intervention by the supervising nurse?
a. Checks the health care provider's order before the blood transfusion.
b. Compares the hospital identification band name and number to those on the blood component tag
c. Uses the patient's room number as a form of identification
d. Examines blood bag tag and attached tag to ensure that the ABO and Rh types are compatible
c
A client is being discharged home with active tuberculosis. Which information does the nurse include in the discharge teaching plan?
A) "You are not contagious unless you stop taking your medication."
B) "You will not be contagious to the people you have been living with."
C) "You will have to take these medications for at least 1 year."
D) "Your sputum may turn a rust color as your condition gets better."
(Chp. 31; elsevier resources)
B) "You will not be contagious to the people you have been living with."
(Chp. 31; elsevier resources)
13. Patients who have central nervous system lesions from stroke, multiple sclerosis, or parasacral spinal cord lesions may have which type for urinary incontinence?
a. Detrusor hyperreflexia
b. Mixed
c. Stress
d. Functional
a
21. A client is brought to the emergency department after sustaining injuries in a severe car crash. The client's chest wall does not appear to be moving normally with respirations, oxygen saturation is 82%, and the client is cyanotic. What action by the nurse is the priority?
a. Administer oxygen and reassess.
b. Auscultate the client's lung sounds. c. Facilitate a portable chest x-ray.
d. Prepare to assist with intubation.
ANS: D
This client has manifestations of flail chest and, with the other signs, needs to be intubated and mechanically ventilated immediately. The nurse does not have time to administer oxygen and wait to reassess, or to listen to lung sounds. A chest x-ray will be taken after the client is intubated.
An experienced nurse is supervising a new nurse who is assessing a patient with a suspected hematologic problem. The experienced nurse would intervene if the new nurse performed which action?
a. Palpated the edge of the liver in the right upper
quadrant
b. Auscultated the heart for abnormal heart sounds or
irregular rhythms
c. Used the fingertips to firmly press over the ribs or
sternum
d. Palpated the left upper quadrant to locate an
enlarged spleen
d
19. An emergency department nurse triages clients who present with chest discomfort. Which client should the nurse plan to assess first?
a. A 42-year-old female who describes her pain as a dull ache with numbness in her fingers
b. A 49-year-old male who reports moderate pain that is worse on inspiration
c. A 53-year-old female who reports substernal pain that radiates to her abdomen
d. A 58-year-old male who describes his pain as intense stabbing that spreads across his chest
D
All clients who have chest pain should be assessed more thoroughly. To determine which client should be seen first, the nurse must understand common differences in pain descriptions. Intense stabbing, vise-like substernal pain that spreads through the clients chest, arms, jaw, back, or neck is indicative of a myocardial infarction.
The nurse should plan to see this client first to prevent cardiac cell death. A dull ache with numbness in the fingers is consistent with anxiety. Pain that gets worse with inspiration is usually related to a pleuropulmonary problem. Pain that spreads to the abdomen is often associated with an esophageal-gastric problem, especially when this pain is experienced by a male client. Female clients may experience abdominal discomfort with a myocardial event. Although clients with anxiety, pleuropulmonary, and esophageal-gastric problems should be seen, they are not a higher priority than myocardial infarction.
How does the drug desmopressin (DDAVP) decrease urine output in a client with diabetes insipidus (DI)?
A. Blocks reabsorption of sodium
B. Increases blood pressure
C. Increases cardiac output
D. Works as an antidiuretic hormone (ADH) in the kidneys
D.
Desmopressin is a synthetic form of ADH that binds to kidney receptors and enhances reabsorption of water, thus reducing urine output. Desmopressin does not have any effect on sodium reabsorption. It may cause a slight increase or a transient decrease in blood pressure, but this does not affect urine output. Desmopressin does not increase cardiac output.
13. Mastering voluntary micturition is a normal developmental task for which person?
a. A healthy 20-month-old toddler
b. A 56-year-old woman with stress incontinence
c. A healthy 8-year-old child
d. A 25-year-old with a spinal cord injury
a
13. A client is diagnosed with chronic kidney disease (CKD). What is an ideal goal of treatment set by the nurse in the care plan to reduce the risk of pulmonary edema?
a. Maintaining oxygen saturation of 89%
b. Minimal crackles and wheezes in lung sounds
c. Maintaining a balanced intake and output d. Limited shortness of breath upon exertion
ANS: C
With an optimal fluid balance, the client will be more able to eject blood from the left ventricle without increased pressure in the left ventricle and pulmonary vessels. Other ideal goals are oxygen saturations greater than 92%, no auscultated crackles or wheezes, and no demonstrated shortness of breath.
21. A nurse admits a client from the emergency department. Client data are listed below:
History
Physical Assessment
Laboratory Values:
70 years of age
History of diabetes
On insulin twice a day Reports new-onset dyspnea and productive cough
Crackles and rhonchi heard throughout the lungs
Dullness to percussion LLL Afebrile
Oriented to person only
WBC: 5,200/mm3 PaO2 on room air 65mm Hg
What action by the nurse is the priority?
a. Administer oxygen at 4 liters per nasal cannula. b. Begin broad-spectrum antibiotics.
c. Collect a sputum sample for culture.
d. Start an IV of normal saline at 50 mL/hr.
ANS: A
All actions are appropriate for this client who has manifestations of pneumonia. However, airway and breathing come first, so begin oxygen administration and titrate it to maintain saturations greater than 95%. Start the IV and collect a sputum culture, and then begin antibiotics.
26. A patient with lymphoma requires a hematopoietic stem cell transplant and a donor is being sought. which type of transplant is likely to yield the best results?
a. Synthetic human leukocyte antigen (HLA)
b. HLA-identical twin sibling
c. HLA-matched first-degree relative
d. HLA-matched stem cells from an umbilical cord of a related donor
b
19. A nurse cares for a client who is on a cardiac monitor. The monitor displayed the rhythm shown below:
Which action should the nurse take first?
a. Assess airway, breathing, and level of consciousness.
b. Administer an amiodarone bolus followed by a drip.
c. Cardiovert the client with a biphasic defibrillator.
d. Begin cardiopulmonary resuscitation (CPR).
ANS: A
Ventricular tachycardia occurs with repetitive firing of an irritable ventricular ectopic focus, usually at a rate of 140 to 180 beats/min or more. Ventricular tachycardia is a lethal dysrhythmia. The nurse should first assess if the client is alert and breathing. Then the nurse should call a Code Blue and begin CPR. If this client is pulseless, the treatment of choice is defibrillation. Amiodarone is the antidysrhythmic of choice, but it is not the first action.
DIF: Applying/Application REF: 670
KEY: Cardiac electrical conduction| medical emergency
MSC: Integrated Process: Nursing Process: Implementation
NOT: Client Needs Category: Physiological Integrity: Physiological Adaptation
9. When analyzing laboratory values, the nurse expects to find which value as a direct result of overproduction of GH?
a. Hyperglycemia
b. Hyperphosphatemia
c. Hypocalcemia
d. Hypercalcemia
a
Hyperglycemia
19. A nurse cares for a client with end-stage heart failure who is awaiting a transplant. The client appears depressed and states, "I know a transplant is my last chance, but I don't want to become a vegetable." How should the nurse respond?
a. "Would you like to speak with a priest or chaplain?"
b. "I will arrange for a psychiatrist to speak with you."
c. "Do you want to come off the transplant list?"
d. "Would you like information about advance directives?"
ANS: D
The client is verbalizing a real concern or fear about negative outcomes of the surgery. This anxiety itself can have a negative effect on the outcome of the surgery because of sympathetic stimulation. The best action is to allow the client to verbalize the concern and work toward a positive outcome without making the client feel as though he or she is crazy. The client needs to feel that he or she has some control over the future. The nurse personally provides care to address the client's concerns instead of pushing the client's issues off on a chaplain or psychiatrist. The nurse should not jump to conclusions and suggest taking the client off the transplant list, which is the best treatment option.
DIF: Applying/Application REF: 691 KEY: Transplant| psychosocial response| anxiety
MSC: Integrated Process: Nursing Process: Implementation NOT: Client Needs Category: Psychosocial Integrity
9. The binding of a hormone to a specific receptor site is an example of which endocrine process?
a. "Lock and key" manner
b. Negative feedback mechanism
c. Neuroendocrine regulation
d. "Fight-or-flight" response
a
"Lock and key" manner
Question 9 of 41
The nurse is mentoring a recent graduate RN about administering blood and blood products. What does the nurse include in the data?
a. Obtain the client's initial set of vital signs (VS) within the first 10 minutes of the infusion.
b. Remain with the client who is receiving the blood for the first 5 minutes of the infusion.
c. Use a 22-gauge needle to obtain venous access when starting the infusion.
d. Verify with another RN all of the data on blood products.
d
Verify with another RN all of the data on blood products.
All data are checked by two RNs. Human error is the most common cause of ABO incompatibilities in administering blood and blood products. Initial VS should be recorded before the start of infusion of blood, not after it has begun. The nurse remains with the client for the first 15 to 30 minutes (not 5) of the infusion. This is the period when any transfusion reactions are likely to happen. A 20-gauge needle (or a central line catheter) is used; the 22-gauge needle is too small.
Which method is the best way to prevent outbreaks of pandemic influenza?
A) Avoiding public gatherings at all times
B) Early recognition and quarantine
C) Vaccinating everyone with pneumonia vaccine
D) Widespread distribution of antiviral drugs
(Chp. 31; elsevier resources)
B) Early recognition and quarantine
(Chp. 31; elsevier resources)
13. After teaching a client with renal cancer who is prescribed temsirolimus (Torisel), the nurse assesses the client's understanding. Which statement made by the client indicates a correct understanding of the teaching?
a. "I will take this medication with food and plenty of water."
b. "I shall keep my appointment at the infusion center each week."
c. "I'll limit my intake of green leafy vegetables while on this medication." d. "I must not take this medication if I have an infection or am feeling ill."
ANS: B
Temsirolimus is administered as a weekly intravenous infusion. This medication blocks protein that is needed for cell division and therefore inhibits cell cycle progression. This medication is not taken orally, and clients do not need to follow a specific diet.
13. A nurse cares for a client admitted from a nursing home after several recent falls. What prescription should the nurse complete first?
a. Obtain urine sample for culture and sensitivity.
b. Administer intravenous antibiotics.
c. Encourage protein intake and additional fluids.
d. Consult physical therapy for gait training.
A
Although all interventions are or might be important, obtaining a urine sample for urinalysis takes priority. Often urinary tract infection (UTI) symptoms in older adults are atypical, and a UTI may present with new onset of confusion or falling. The urine sample should be obtained before starting antibiotics. Dietary
requirements and gait training should be implemented after obtaining the urine sample.
24. A nurse auscultates a client's lung fields. Which pathophysiologic process should the nurse associate with this breath sound?
(Click the media button to hear the audio clip.)
a. Inflammation of the pleura
b. Constriction of the bronchioles
c. Upper airway obstruction
d. Pulmonary vascular edema
ANS: A
A pleural friction rub can be heard when the pleura is inflamed and rubbing against the lung wall. The other pathophysiologic processes would not cause a pleural friction rub. Constriction of the bronchioles may be heard as a wheeze, upper airway obstruction may be heard as stridor, and pulmonary vascular edema may be heard as crackles.
DIF: Applying/Application REF: 575
KEY: Assessment/diagnostic examination
MSC: Integrated Process: Nursing Process: Implementation
NOT: Client Needs Category: Physiological Integrity: Physiological Adaptation
A client has frequent hospitalizations for leukemia and is worried about functioning as a parent to four small children. What action by the nurse would be most helpful?
a. Assist the client to make "sick day" plans for household responsibilities.
b. Determine if there are family members or friends who can help the client.
c. Help the client inform friends and family that they will have to help out.
d. Refer the client to a social worker in order to investigate respite child care.
a. Assist the client to make "sick day" plans for household responsibilities.
20. A nurse auscultated heart tones on an older adult client. Which action should the nurse take based on heart tones heard?
(Click the media button to hear the audio clip.)
a. Administer a diuretic.
b. Document the finding.
c. Decrease the IV flow rate.
d. Evaluate the clients medications.
B
The sound heard is an atrial gallop S4. An atrial gallop may be heard in older clients because of a stiffened ventricle. The nurse should document the finding, but no other intervention is needed at this time.
A nurse plans care for a client with Cushings disease. Which action should the nurse include in this clients plan of care to prevent injury?
a. Pad the siderails of the clients bed.
b. Assist the client to change positions slowly.
c. Use a lift sheet to change the clients position.
d. Keep suctioning equipment at the clients bedside.
C (Cushings syndrome or disease greatly increases the serum levels of cortisol, which contributes to excessive bone demineralization and increases the risk for pathologic bone fracture. Padding the siderails and assisting the client to change position may be effective, but these measures will not protect him or her as much as using a lift sheet. The client should not require suctioning.)
13. A nurse cares for a client admitted from a nursing home after several recent falls. What prescription should the nurse complete first?
a. Obtain urine sample for culture and sensitivity.
b. Administer intravenous antibiotics.
c. Encourage protein intake and additional fluids. d. Consult physical therapy for gait training.
ANS: A
Although all interventions are or might be important, obtaining a urine sample for urinalysis takes priority. Often urinary tract infection (UTI) symptoms in older adults are atypical, and a UTI may present with new onset of confusion or falling. The urine sample should be obtained before starting antibiotics. Dietary requirements and gait training should be implemented after obtaining the urine sample.
25. A nurse auscultates a client's lung fields. Which action should the nurse take based on the lung sounds?
(Click the media button to hear the audio clip.)
a. Assess for airway obstruction.
b. Initiate oxygen therapy.
c. Assess vital signs.
d. Elevate the client's head.
ANS: A
Stridor is the sound heard, and it indicates severe airway constriction. The nurse must administer a bronchodilator to get air into the lungs. Administering oxygen, assessing vital signs, and elevating the client's head will not help until the client's airways are open.
DIF: Applying/Application REF: 575
KEY: Assessment/diagnostic examination
MSC: Integrated Process: Nursing Process: Implementation
NOT: Client Needs Category: Safe and Effective Care Environment: Management of Care
A client has been treated for a deep vein thrombus and today presents to the clinic with petechiae. Laboratory results show a platelet count of 42,000/mm3. The nurse reviews the client's medication list to determine if the client is taking which drug?
a. Enoxaparin (Lovenox)
b. Salicylates (aspirin)
c. Unfractionated heparin
d. Warfarin (Coumadin)
c. Unfractionated heparin
After receiving change-of-shift report in the coronary care unit, which client does the nurse assess first?
a. The client with acute coronary syndrome who has a 3-pound weight gain and dyspnea
b. The client with percutaneous coronary angioplasty who has a dose of heparin scheduled
c. The client who had bradycardia after a myocardial infarction and now has a paced heart rate of 64 beats/min
d. A client who has first-degree heart block, rate 68 beats/min, after having an inferior myocardial infarction
a. The client with acute coronary syndrome who has a 3-pound weight gain and dyspnea
Dyspnea and weight gain are symptoms of left ventricular failure and pulmonary edema; this client needs prompt intervention. A scheduled heparin dose does not take priority over dyspnea; it can be administered after the client with dyspnea is taken care of. The client with a pacemaker and a normal heart rate is not in danger. First-degree heart block is rarely symptomatic, and the client has a normal heart rate; the client with dyspnea should be seen first.
The clinical manifestations of hyperthyroidism are known as which condition?
a. Thyrotoxicosis
b. Euthyroid function
c. Graves' disease
d. Hypermetabolism
a
A nurse plans care for an older adult who is admitted to the hospital for pneumonia. The client has no known drug allergies and no significant health history. Which action should the nurse include in this client's plan of care?
a. Initiate Airborne Precautions.
b. Offer fluids every hour or two.
c. Place an indwelling urinary catheter.
d. Palpate the client's thyroid gland.
A normal age-related endocrine change is decreased antidiuretic hormone (ADH) production. This results in a more diluted urine output, which can lead to dehydration. If no contraindications are known, the nurse should offer (or delegate) the client something to drink at least every 2 hours. A client with simple pneumonia would not require Airborne Precautions. Indwelling urinary catheterization is not necessary for this client and would increase the client's risk for infection. The nurse should plan a toileting schedule and assist the client to the bathroom if needed. Palpating the client's thyroid gland is a part of a comprehensive examination but is not specifically related to this client.
19. The nurse is caring for a client with a chest tube after a coronary artery bypass graft. The drainage slows significantly. What action by the nurse is most important?
a. Increase the setting on the suction.
b. Notify the provider immediately.
c. Re-position the chest tube.
d. Take the tubing apart to assess for clots.
ANS: B
If the drainage in the chest tube decreases significantly and dramatically, the tube may be blocked by a clot. This could lead to cardiac tamponade. The nurse should notify the provider immediately. The nurse should not independently increase the suction, re-position the chest tube, or take the tubing apart.
DIF: Applying/Application REF: 778
KEY: Coronary artery bypass graft| critical rescue| chest tubes| cardiovascular system MSC: Integrated Process: Communication and Documentation
NOT: Client Needs Category: Safe and Effective Care Environment: Management of Care
Question 10 of 41
Which would be an appropriate task to delegate to unlicensed assistive personnel (UAP) working on a medical-surgical unit?
a. Administering erythropoietin to a client with myelodysplastic syndrome
b. Assessing skin integrity on an anemic client who fell during ambulation
c. Assisting a client with folic acid deficiency in making diet choices
d. Obtaining vital signs on a client receiving a blood transfusion
d
Obtaining vital signs on a client receiving a blood transfusion
Obtaining vital signs on a client is within the scope of practice for UAP. Administering medication, assessing clients, and assisting with prescribed diet choices are complex actions that should be done by licensed nurses.
22. A student nurse asks for an explanation of "refractory hypoxemia." What answer by the nurse instructor is best?
a. "It is chronic hypoxemia that accompanies restrictive airway disease."
b. "It is hypoxemia from lung damage due to mechanical ventilation."
c. "It is hypoxemia that continues even after the client is weaned from oxygen." d. "It is hypoxemia that persists even with 100% oxygen administration."
ANS: D
Refractory hypoxemia is hypoxemia that persists even with the administration of 100% oxygen. It is a cardinal sign of acute respiratory distress syndrome. It does not accompany restrictive airway disease and is not caused by the use of mechanical ventilation or by being weaned from oxygen.
14. A client has a long history of hypertension. Which category of medications would the nurse expect to be ordered to avoid chronic kidney disease (CKD)?
a. Antibiotic
b. Histamine blocker
c. Bronchodilator
d. Angiotensin-converting enzyme (ACE) inhibitor
ANS: D
ACE inhibitors stop the conversion of angiotensin I to the vasoconstrictor angiotensin II. This category of medication also blocks bradykinin and prostaglandin, increases renin, and decreases aldosterone, which promotes vasodilation and perfusion to the kidney. Antibiotics fight infection, histamine blockers decrease inflammation, and bronchodilators increase the size of the bronchi; none of these medications helps slow the progression of CKD in clients with hypertension.
14. A nurse assesses clients on the medical-surgical unit. Which client is at greatest risk for bladder cancer?
a. A 25-year-old female with a history of sexually transmitted diseases
b. A 42-year-old male who has worked in a lumber yard for 10 years
c. A 55-year-old female who has had numerous episodes of bacterial cystitis d. An 86-year-old male with a 50-pack-year cigarette smoking history
ANS: D
The greatest risk factor for bladder cancer is a long history of tobacco use. The other factors would not necessarily contribute to the development of this specific type of cancer.
1. A nurse is providing pneumonia vaccinations in a community setting. Due to limited finances, the event organizers must limit giving the vaccination to priority groups. What clients would be considered a priority when administering the pneumonia vaccination? (Select all that apply.)
a. 22-year-old client with asthma
b. Client who had a cholecystectomy last year
c. Client with well-controlled diabetes
d. Healthy 72-year-old client
e. Client who is taking medication for hypertension
ANS: A, C, D, E
Clients over 65 years of age and any client (no matter what age) with a chronic health condition would be considered a priority for a pneumonia vaccination. Having a cholecystectomy a year ago does not qualify as a chronic health condition.
27. Patients with sickle cell disease are more susceptible to infections. Which actions help prevent infection? (Select all that apply.)
a. Perform consistent thorough handwashing
b. Encourage yearly flu vaccination
c. Administer twice-daily oral penicillin
d. Administer NSAIDs three times a day
e. Monitor CBC and differential white cell count
f. Assess vital signs at least every 4 hours
a, b, c, e, f
After receiving change-of-shift report in the coronary care unit, which client does the nurse assess first?
The client with acute coronary syndrome who has a 3-pound weight gain and dyspnea
The client with percutaneous coronary angioplasty who has a dose of heparin scheduled
The client who had bradycardia after a myocardial infarction and now has a paced heart rate of 64 beats/min
A client who has first-degree heart block, rate 68 beats/min, after having an inferior myocardial infarction
The client with acute coronary syndrome who has a 3-pound weight gain and dyspnea
Dyspnea and weight gain are symptoms of left ventricular failure and pulmonary edema; this client needs prompt intervention. A scheduled heparin dose does not take priority over dyspnea; it can be administered after the client with dyspnea is taken care of. The client with a pacemaker and a normal heart rate is not in danger. First-degree heart block is rarely symptomatic, and the client has a normal heart rate; the client with dyspnea should be seen first.
Ch.62 p. 1276, Patient-Centered Care; Quality Improvement; Safety
The patient is a 32-year-old woman admitted to your unit after surgery for fractures of the left arm and leg resulting from a car crash. She is awake and able to verify her medical history of rheumatoid arthritis and her usual daily medications. These are 10 mg of prednisone, naproxen 800 mg twice daily, oral contraceptives, calcium 600 mg, and one multiple vitamin tablet. All of these are prescribed for her to receive during her hospitalization. She is concerned about pain management and how long the recovery will be for the fractures. She is friendly, somewhat anxious, asks many questions, and wants to do "her part" to ensure good recovery. Over the next 4 days, she has become quieter, mumbles that her head and stomach hurt, and now does not recognize the assistant who has been providing her daily care. When she receives her medications, she has difficulty picking them up. The nursing assistant remarks that taking her pulse is difficult because it is so slow and irregular. When you assess her, she is so weak that she is unable to lift her arm for a blood pressure check. Her blood pressure is 92/50, which is down from the 128/84 reading on admission. You also verify that her heart beat is slow and irregular.
5. What could be done to prevent this problem from happening again?
Any patient who routinely takes a corticosteroid should be automatically evaluated on a daily basis for manifestations of adrenal insufficiency. Ideally, the person would receive additional corticosteroid therapy in advance of changes to prevent adrenal insufficiency. At the very least, assessing for early manifestations could have identified this problem earlier and prevented a near tragedy.
14. A nurse cares for a client who has pyelonephritis. The client states, "I am embarrassed to talk about my symptoms." How should the nurse respond?
a. "I am a professional. Your symptoms will be kept in confidence."
b. "I understand. Elimination is a private topic and shouldn't be discussed."
c. "Take your time. It is okay to use words that are familiar to you."
d. "You seem anxious. Would you like a nurse of the same gender to care for you?"
ANS: C
Clients may be uncomfortable discussing issues related to elimination and the genitourinary area. The nurse should encourage the client to use language that is familiar to the client. The nurse should not make promises that cannot be kept, like keeping the client's symptoms confidential. The nurse must assess the client and cannot take the time to stop the discussion or find another nurse to complete the assessment.
Community health nurses are tasked with providing education on prevention of respiratory infection for diseases such as the flu. Which target audience is given the highest priority?
A) Homeless people
B) Hospital staff
C) Politicians
D) Prison staff and inmates
(Chp. 31; elsevier resources)
D) Prison staff and inmates
(Chp. 31; elsevier resources)
The nurse assesses a client's oral cavity and makes the discovery shown in the photo below:
What action by the nurse is most appropriate?
a. Encourage the client to have genetic testing.
b. Instruct the client on high-fiber foods.
c. Place the client in protective precautions.
d. Teach the client about cobalamin therapy.
d. Teach the client about cobalamin therapy.
20. A nurse performs an admission assessment on a 75-year-old client with multiple chronic diseases. The client's blood pressure is 135/75 mm Hg and oxygen saturation is 94% on 2 liters per nasal cannula. The nurse assesses the client's rhythm on the cardiac monitor and observes the reading shown below:
Which action should the nurse take first?
a. Begin external temporary pacing.
b. Assess peripheral pulse strength.
c. Ask the client what medications he or she takes.
d. Administer 1 mg of atropine.
ANS: C
This client is stable and therefore does not require any intervention except to determine the cause of the bradycardia. Bradycardia is often caused by medications. Clients who have multiple chronic diseases are often on multiple medications that can interact with each other. The nurse should assess the client's current medications first.
DIF: Applying/Application REF: 658
KEY: Cardiac electrical conduction| medications| adverse effects
MSC: IntegratedProcess:NursingProcess:Assessment
NOT: Client Needs Category: Physiological Integrity: Pharmacological and Parenteral Therapies
A nurse teaches a client with type 2 diabetes mellitus who is prescribed glipizide (Glucotrol). Which statement should the nurse include in this client's teaching?
a. "Change positions slowly when you get out of bed."
b. "Avoid taking nonsteroidal anti-inflammatory drugs (NSAIDs)."
c. "If you miss a dose of this drug, you can double the next dose."
d. "Discontinue the medication if you develop a urinary infection."
ANS: B
NSAIDs potentiate the hypoglycemic effects of sulfonylurea agents. Glipizide is a sulfonylurea. The other statements are not applicable to glipizide.
Ch.61 Question 9 of 16
A client with an endocrine disorder says, "I can't, you know, satisfy my wife anymore." What is the nurse's best response?
a. "Can you please tell me more?"
b. "Don't worry. That is normal."
c. "How does she feel?"
d. "Should I make an appointment with a counselor?"
a
"Can you please tell me more?"
Asking the client to explain his concerns in an open-ended question allows the nurse to explore his feelings more thoroughly. Telling a client that something is "normal" is dismissive; this is new to the client and is a concern for him. The focus of the nurse's response needs to be on the client, not on the wife initially. Referring the client to a counselor is not an appropriate first step; this dismisses the client's concerns and does not allow him to express his frustrations at the moment.
20. A nurse assesses a client who has a history of heart failure. Which question should the nurse ask to assess the extent of the client's heart failure?
a. "Do you have trouble breathing or chest pain?"
b. "Are you able to walk upstairs without fatigue?"
c. "Do you awake with breathlessness during the night?"
d. "Do you have new-onset heaviness in your legs?"
ANS: B
Clients with a history of heart failure generally have negative findings, such as shortness of breath. The nurse needs to determine whether the client's activity is the same or worse, or whether the client identifies a decrease in activity level. Trouble breathing, chest pain, breathlessness at night, and peripheral edema are symptoms of heart failure, but do not provide data that can determine the extent of the client's heart failure.
DIF: Applying/Application REF: 682
KEY: Heart failure| functional ability| respiratory distress/failure
MSC: IntegratedProcess:NursingProcess:Assessment
NOT: Client Needs Category: Physiological Integrity: Physiological Adaptation
Question 11 of 41
A 56-year-old client admitted with a diagnosis of acute myelogenous leukemia (AML) is prescribed IV cytosine arabinoside for 7 days and an infusion of daunorubicin for the first 3 days. An infection develops. What knowledge does the nurse use to determine that the appropriate antibiotic has been prescribed for this client?
a. Evaluating the client's liver function tests (LFTs) and serum creatinine levels
b. Evaluating the client's white blood cell (WBC) count level
c. Checking the culture and sensitivity test results to be certain that the requested antibiotic is effective against the organism causing the infection
d. Recognizing that vancomycin (Vancocin) is the drug of choice used to treat all infections in clients with AML
c
Checking the culture and sensitivity test results to be certain that the requested antibiotic is effective against the organism causing the infection
Checking the culture and sensitivity test results to be certain that the requested antibiotic is effective against the organism causing the infection is the best action to take. Drug therapy is the main defense against infections that develop in clients undergoing therapy for AML. Agents used depend on the client's sensitivity to various antibiotics for the organism causing the infection. Although LFTs and kidney function tests may be influenced by antibiotics, these tests do not determine the effectiveness of the antibiotic. Although the WBC count is elevated in infection, this test does not influence which antibiotic will be effective in fighting the infection. Vancomycin may not be effective in all infections; culturing of the infection site and determining the organism's sensitivity to a cohort of drugs are needed, which will provide data on drugs that are capable of eradicating the infection in this client.
26. The nurse instructs a client on the steps needed to obtain a peak expiratory flow rate. In which order should these steps occur?
1. "Take as deep a breath as possible."
2. "Stand up (unless you have a physical disability)."
3. "Place the meter in your mouth, and close your lips around the mouthpiece." 4. "Make sure the device reads zero or is at base level."
5. "Blow out as hard and as fast as possible for 1 to 2 seconds."
6. "Write down the value obtained."
7. "Repeat the process two additional times, and record the highest number in your chart." a. 4, 2, 1, 3, 5, 6, 7
b. 3, 4, 1, 2, 5, 7, 6
c. 2, 1, 3, 4, 5, 6, 7
d. 1, 3, 2, 5, 6, 7, 4
ANS: A
The proper order for obtaining a peak expiratory flow rate is as follows. Make sure the device reads zero or is at base level. The client should stand up (unless he or she has a physical disability). The client should take as deep a breath as possible, place the meter in the mouth, and close the lips around the mouthpiece. The client should blow out as hard and as fast as possible for 1 to 2 seconds. The value obtained should be written down. The process should be repeated two more times, and the highest of the three numbers should be recorded in the client's chart.
DIF: Applying/Application REF: 552
KEY: Assessment/diagnostic examination
MSC: IntegratedProcess:NursingProcess:Assessment
NOT: Client Needs Category: Physiological Integrity: Reduction of Risk Potential
14. A nurse assesses clients on the medical-surgical unit. Which client is at greatest risk for bladder cancer?
a. A 25-year-old female with a history of sexually transmitted diseases
b. A 42-year-old male who has worked in a lumber yard for 10 years
c. A 55-year-old female who has had numerous episodes of bacterial cystitis
d. An 86-year-old male with a 50pack-year cigarette smoking history
D
The greatest risk factor for bladder cancer is a long history of tobacco use. The other factors would not necessarily contribute to the development of this specific type of cancer.
14. An elderly patient has been in bed for several days after a fall. The nurse encourage ambulation to stimulate the movement of urine through the ureter by what phenomenon?
a. Peristalsis
b. Gravity
c Pelvic pressure
d. Backflow
a
23. A nurse is caring for a client on the medical stepdown unit. The following data are related to this client:
What action by the nurse is most appropriate?
a. Call respiratory therapy for a breathing treatment. b. Facilitate a STAT pulmonary angiography.
c. Prepare for immediate endotracheal intubation.
d. Prepare to administer intravenous anticoagulants.
ANS: B
This client has manifestations of pulmonary embolism (PE); however, many conditions can cause the client's presentation. The gold standard for diagnosing a PE is pulmonary angiography. The nurse should facilitate this test as soon as possible. The client does not have wheezing, so a respiratory treatment is not needed. The client is not unstable enough to need intubation and mechanical ventilation. IV anticoagulants are not given without a diagnosis of PE.
28. The nurse is caring for a patient who has donated bone marrow. In addition to having the aspiration sites monitored, the nurse will anticipate the need for which intervention? (Select all that apply.)
a. Fluid for hydration
b. Pain management
c. Possible RBC infusion
d. Prophylactic antibiotic therapy
e. Assessment for complications of anesthesia
a, b, c, e
21. A nurse assesses a client who has aortic regurgitation. In which location in the illustration shown below should the nurse auscultate to best hear a cardiac murmur related to aortic regurgitation?
a. Location A
b. Location B
c. Location C
d. Location D
A
The aortic valve is auscultated in the second intercostal space just to the right of the sternum.
9. The clinical manifestations of hyperthyroidism are known as which condition?
a. Thyrotoxicosis
b. Euthyroid function
c. Graves' disease
d. Hypermetabolism
a
14. The nurse is caring for a postoperative patient and is evaluating the patient's intake and output as a measure to prevent AKI. The patient weighs 60 kilograms and has produced 180 mL of urine in the past 4 hours. What should the nurse do?
a. Perform other assessments related to fluid status and record the output.
b. Call the health care provider and obtain an order for a fluid bolus.
c. Encourage the patient to drink more fluid, so that the output is increased.
d. Compare the patient's weight to baseline to determine fluid retention.
a
2. A hospital nurse is participating in a drill during which many clients with inhalation anthrax are being admitted. What drugs should the nurse anticipate administering? (Select all that apply.)
a. Amoxicillin (Amoxil)
b. Ciprofloxacin (Cipro)
c. Doxycycline (Vibramycin)
d. Ethambutol (Myambutol)
e. Sulfamethoxazole-trimethoprim (SMX-TMP) (Septra)
ANS: A, B, C
Amoxicillin, ciprofloxacin, and doxycycline are all possible treatments for inhalation anthrax. Ethambutol is used for tuberculosis. SMX-TMP is commonly used for urinary tract infections and other common infections.
29. When caring for a patient after bone marrow stem cell transplantation, when does the nurse expect engraftment (the settling in of stem cells and the start of producing new cells) to occur?
a. 8 to 12 hours after infusion
b. 7 days after infusion
c. 21 days after infusion
d. 6 weeks after infusion
c
An LPN/LVN is scheduled to work on the inpatient "stepdown" cardiac unit. Which client does the charge nurse assign to the LPN/LVN?
a. A 60-year-old who was admitted today for pacemaker insertion because of third-degree heart block and who is now reporting chest pain
b. A 62-year-old who underwent open heart surgery 4 days ago for mitral valve replacement and who has a temperature of 38.2° C
c. A 66-year-old who has a prescription for a nitroglycerin (Nitro-Dur) patch and is scheduled for discharge to a group home later today
d. A 69-year-old who had a stent placed 2 hours ago in the left anterior descending artery and who has bursts of ventricular tachycardia
c. A 66-year-old who has a prescription for a nitroglycerin (Nitro-Dur) patch and is scheduled for discharge to a group home later today
The LPN/LVN scope of practice includes administration of medications to stable clients. Third-degree heart block is characterized by a very low heart rate and usually by required pacemaker insertion; the skills of the RN are needed to care for this client. Fever after surgery requires collaboration with the health care provider, which is more consistent with the role of the RN. The client with a recent stent placement and having bursts of ventricular tachycardia is unstable and is showing ventricular irritability; he will need medications and monitoring beyond the scope of practice of the LPN/LVN.
9. A nurse assesses clients for potential endocrine disorders. Which client is at greatest risk for hyperparathyroidism?
a. A 29-year-old female with pregnancy-induced hypertension
b. A 41-year-old male receiving dialysis for end-stage kidney disease
c. A 66-year-old female with moderate heart failure
d. A 72-year-old male who is prescribed home oxygen therapy
b. Clients who have chronic kidney disease do not completely activate vitamin D and poorly absorb calcium from the GI tract. They are chronically hypocalcemic, and this triggers overstimulation of the parathyroid glands. Pregnancy-induced hypertension, moderate heart failure, and home oxygen therapy do not place a client at higher risk for hyperparathyroidism.
An LPN/LVN is scheduled to work on the inpatient "stepdown" cardiac unit. Which client does the charge nurse assign to the LPN/LVN?
A 60-year-old who was admitted today for pacemaker insertion because of third-degree heart block and who is now reporting chest pain
A 62-year-old who underwent open heart surgery 4 days ago for mitral valve replacement and who has a temperature of 38.2° C
A 66-year-old who has a prescription for a nitroglycerin (Nitro-Dur) patch and is scheduled for discharge to a group home later today
A 69-year-old who had a stent placed 2 hours ago in the left anterior descending artery and who has bursts of ventricular tachycardia
A 66-year-old who has a prescription for a nitroglycerin (Nitro-Dur) patch and is scheduled for discharge to a group home later today
The LPN/LVN scope of practice includes administration of medications to stable clients. Third-degree heart block is characterized by a very low heart rate and usually by required pacemaker insertion; the skills of the RN are needed to care for this client. Fever after surgery requires collaboration with the health care provider, which is more consistent with the role of the RN. The client with a recent stent placement and having bursts of ventricular tachycardia is unstable and is showing ventricular irritability; he will need medications and monitoring beyond the scope of practice of the LPN/LVN.
A nurse plans care for an older adult who is admitted to the hospital for pneumonia. The client has no known drug allergies and no significant health history. Which action should the nurse include in this client's plan of care?
a. Initiate Airborne Precautions.
b. Offer fluids every hour or two.
c. Place an indwelling urinary catheter.
d. Palpate the client's thyroid gland.
A normal age-related endocrine change is decreased antidiuretic hormone (ADH) production. This results in a more diluted urine output, which can lead to dehydration. If no contraindications are known, the nurse should offer (or delegate) the client something to drink at least every 2 hours. A client with simple pneumonia would not require Airborne Precautions. Indwelling urinary catheterization is not necessary for this client and would increase the client's risk for infection. The nurse should plan a toileting schedule and assist the client to the bathroom if needed. Palpating the client's thyroid gland is a part of a comprehensive examination but is not specifically related to this client.
Question 12 of 41
What are the typical clinical manifestations of anemia?
Select all that apply.
a. Decreased breath sounds
b. Dyspnea on exertion
c. Elevated temperature
d. Fatigue
e. Pallor
f. Tachycardia
b, d, e, f
Dyspnea on exertion, Fatigue, Pallor, Tachycardia
Difficulty breathing—especially with activity—is common with anemia. Lower levels of hemoglobin carry less O2 to the cells of the body. Fatigue is a classic symptom of anemia; lowered O2 levels contribute to a faster pulse (i.e., cardiac rate) and tend to "wear out" a client's energy. Lowered O2 levels deliver less oxygen to all cells, making clients with anemia pale—especially their ears, nail beds, palms, and conjunctivae and around the mouth. Respiratory problems with anemia do not include changes in breath sounds; dyspnea and decreased oxygen saturation levels are present. Skin is cool to the touch, and an intolerance to cold is noted; elevated temperature would signify something additional, such as infection
A 70-year-old client has a complicated medical history, including chronic obstructive pulmonary disease. Which client statement indicates the need for further teaching about the disease?
A) "I am here to receive the yearly pneumonia shot again."
B) "I am here to get my yearly flu shot again."
C) "I should avoid large gatherings during cold and flu season."
D) "I should cough into my upper sleeve instead of my hand."
(Chp. 31; elsevier resources)
A) "I am here to receive the yearly pneumonia shot again."
(Chp. 31; elsevier resources)
14. A patient reports intense urgency, freuqnecy, and bladder pain. Urinalysis results show white blood cells (WBCs) and red blood cells (RBCs) and urine culture results are negative for infection. How does the nurse interpret these findings?
a. Interstitial cystitis
b. Urethritis
c. Bacteriuria
d. Infectious cystitis
a
14. Which pain management strategy does the nurse teach a patient who has pain from infected kidney cysts of PKD?
a. Take nothing by mouth.
b. Increase the dose of NSAIDs.
c. Assume a high-Fowler's position.
d. Apply dry heat to the abdomen or flank.
d
3. A client in the emergency department is taking rifampin (Rifadin) for tuberculosis. The client reports yellowing of the sclera and skin and bleeding after minor trauma. What laboratory results correlate to this condition? (Select all that apply.)
a. Blood urea nitrogen (BUN): 19 mg/dL
b. International normalized ratio (INR): 6.3
c. Prothrombin time: 35 seconds
d. Serum sodium: 130 mEq/L 3 e. White blood cell (WBC) count: 72,000/mm
ANS: B, C
Rifampin can cause liver damage, evidenced by the client's high INR and prothrombin time. The BUN and WBC count are normal. The sodium level is low, but that is not related to this client's problem.
A nurse working with clients with sickle cell disease (SCD) teaches about self-management to prevent exacerbations and sickle cell crises. What factors should clients be taught to avoid? (Select all that apply.)
a. Dehydration
b. Exercise
c. Extreme stress
d. High altitudes
e. Pregnancy
a. Dehydration
c. Extreme stress
d. High altitudes
e. Pregnancy
20. A home health care nurse is visiting an older client who lives alone after being discharged from the hospital after a coronary artery bypass graft. What finding in the home most causes the nurse to consider additional referrals?
a. Dirty carpets in need of vacuuming
b. Expired food in the refrigerator
c. Old medications in the kitchen
d. Several cats present in the home
ANS: B
Expired food in the refrigerator demonstrates a safety concern for the client and a possible lack of money to buy food. The nurse can consider a referral to Meals on Wheels or another home-based food program. Dirty carpets may indicate the client has no household help and is waiting for clearance to vacuum. Old medications can be managed by the home health care nurse and the client working collaboratively. Having pets is not a cause for concern.
DIF: Applying/Application REF: 781
KEY: Home safety| referrals| coronary artery bypass graft
MSC: Integrated Process: Communication and Documentation
NOT: Client Needs Category: Safe and Effective Care Environment: Safety and Infection Control
Question 7 of 16
The nurse is instructing a client who will undergo a suppression test. Which statement by the client indicates that teaching was effective?
a."I am being tested to see whether my hormone glands are hyperactive."
b. "I am being tested to see whether my hormone glands are hypoactive."
c. "I am being tested to see whether my kidneys work at all."
d. "I will be given more hormones as a trigger."
a
"I am being tested to see whether my hormone glands are hyperactive."
Suppression tests are used when hormone levels are high or in the upper range of normal. Failure of suppression of hormone production during testing indicates hyperfunction. A stimulation test assesses whether hormone glands are hypoactive. The adrenal glands are endocrine glands that are located on the kidneys; a suppression test does not measure kidney function. Hormones are given as a trigger in a stimulation test.
14. A nurse teaches a client who is recovering from a urography. Which instruction should the nurse include in this client's discharge teaching?
a. "Avoid direct contact with your urine for 24 hours until the radioisotope clears."
b. "You may have some dribbling of urine for several weeks after this procedure."
c. "Be sure to drink at least 3 liters of fluids today to help eliminate the dye faster." d. "Your skin may become slightly yellow from the dye used in this procedure."
ANS: C
Dyes used in urography are potentially nephrotoxic. A large fluid intake will help the client eliminate the dye rapidly. Dyes used in urography are not radioactive, the client should not experience any dribbling of urine, and the dye should not change the color of the client's skin.
A client who has recently traveled to Vietnam comes to the emergency department with fatigue, lethargy, night sweats, and a low-grade fever. What is the nurse's first action?
A) Contact the health care provider for tuberculosis (TB) medications.
B) Perform a TB skin test.
C) Place a respiratory mask on the client.
D) Test all family members for TB.
(Chp. 31; elsevier resources)
C) Place a respiratory mask on the client.
(Chp. 31; elsevier resources)
A student studying leukemias learns the risk factors for developing this disorder. Which risk factors does this include? (Select all that apply.)
a. Chemical exposure
b. Genetically modified foods
c. Ionizing radiation exposure
d. Vaccinations
e. Viral infections
a. Chemical exposure
c. Ionizing radiation exposure
e. Viral infections
21. A client is on a dopamine infusion via a peripheral line. What action by the nurse takes priority for safety?
a. Assess the IV site hourly.
b. Monitor the pedal pulses.
c. Monitor the client's vital signs.
d. Obtain consent for a central line.
ANS: A
Dopamine should be infused through a central line to prevent extravasation and necrosis of tissue. If it needs to be run peripherally, the nurse assesses the site hourly for problems. When the client is getting the central line, ensuring informed consent is on the chart is a priority. But at this point, the client has only a peripheral line, so caution must be taken to preserve the integrity of the client's integumentary system. Monitoring pedal pulses and vital signs give indications as to how well the drug is working.
DIF: Applying/Application REF: 773
KEY: Inotropic agents| adverse effects| medication safety
MSC: IntegratedProcess:NursingProcess:Assessment
NOT: Client Needs Category: Physiological Integrity: Pharmacological and Parenteral Therapies
A client with hypothyroidism is being discharged. Which environmental change may the client experience in the home?
A. Frequent home care
B. Handrails in the bath
C. Increased thermostat setting
D. Strict infection-control measures
C.
Manifestations of hypothyroidism include cold intolerance. Increased thermostat settings or additional clothing may be necessary. A client with a diagnosis of hypothyroidism can be safely managed at home with adequate discharge teaching regarding medications and instructions on when to notify the health care provider or home health nurse. In general, hypothyroidism does not cause mobility issues. Activity intolerance and fatigue may be an issue, however. A client with hypothyroidism is not immune-compromised or contagious, so no environmental changes need to be made to the home.
Which characteristics place women at high risk for myocardial infarction (MI)? (Select all that apply.)
Premenopausal
Increasing age
Family history
Abdominal obesity
Breast cancer
Increasing age
Family history
Abdominal obesity
Increasing age is a risk factor, especially after 70 years. Family history is a significant risk factor in both men and women. A large waist size and/or abdominal obesity are risk factors for both metabolic syndrome and MI. Premenopausal women are not at higher risk for MI, and breast cancer is not a risk factor for MI.
A client presents to the emergency department with a history of adrenal insufficiency. The following laboratory values are obtained: Na+ 130 mEq/L, K+ 5.6 mEq/L, and glucose 72 mg/dL. Which is the first request that the nurse anticipates?
A. Administer insulin and dextrose in normal saline to shift potassium into cells.
B. Give spironolactone (Aldactone) 100 mg orally.
C. Initiate histamine2 (H2) blocker therapy with ranitidine for ulcer prophylaxis.
D. Obtain arterial blood gases to assess for peaked T waves.
A.
This client is hyperkalemic. The nurse should anticipate a request to administer 20 to 50 units of insulin with 20 to 50 mg of dextrose in normal saline as an IV infusion to shift potassium into the cells. Spironolactone is a potassium-sparing diuretic that helps the body keep potassium, which the client does not need. Although H2 blocker therapy would be appropriate for this client, it is not the first priority. Arterial blood gases are not used to assess for peaked T waves associated with hyperkalemia; an electrocardiogram needs to be obtained instead.
15. A 70-kg adult with chronic renal failure is on a 40-g protein diet. The client has a reduced glomerular filtration rate and is not undergoing dialysis. Which result would give the nurse the most concern?
a. Albumin level of 2.5 g/dL
b. Phosphorus level of 5 mg/dL c. Sodium level of 135 mmol/L d. Potassium level of 5.5 mmol/L
ANS: A
Protein restriction is necessary with chronic renal failure due to the buildup of waste products from protein breakdown. The nurse would be concerned with the low albumin level since this indicates that the protein in the diet is not enough for the client's metabolic needs. The electrolyte values are not related to the protein-restricted diet.
Question 13 of 41
The nurse is teaching a client with newly diagnosed anemia about conserving energy. What does the nurse tell the client?
Select all that apply.
a. "Allow others to perform your care during periods of extreme fatigue."
b. "Drink small quantities of protein shakes and nutritional supplements daily."
c. "Perform a complete bath daily to reduce your chance of getting an infection."
d. "Provide yourself with four to six small, easy-to-eat meals daily."
e. "Perform your care activities in groups to conserve your energy."
f. "Stop activity when shortness of breath or palpitations are present."
a, b, d, f
"Allow others to perform your care during periods of extreme fatigue.", "Drink small quantities of protein shakes and nutritional supplements daily.", "Provide yourself with four to six small, easy-to-eat meals daily.", "Stop activity when shortness of breath or palpitations are present."
It is critical to have others help the anemic client who is extremely tired. Although it may be difficult for him or her to ask for help, this practice should be stressed to the client. Drinking small protein or nutritional supplements will help rebuild the client's nutritional status. Having four to six small meals daily is preferred over three large meals; this practice conserves the body's expenditure of energy used in digestion and assimilation of nutrients. Stopping activities when strain on the cardiac or respiratory system is noted is critical. A complete bath should be performed only every other day; on days in between, the client can be taught to take a "mini" sponge bath, which will conserve energy and still be safe in preventing the risks for infection. Care activities should be spaced every hour or so rather than in groups to conserve energy; the time just before and after meals should be avoided.
27. The nurse instructs a client on how to correctly use an inhaler with a spacer. In which order should these steps occur?
1. 2. 3. 4. 5.
"Press down firmly on the canister to release one dose of medication."
"Breathe in slowly and deeply."
"Shake the whole unit vigorously three or four times."
"Insert the mouthpiece of the inhaler into the nonmouthpiece end of the spacer." "Place the mouthpiece into your mouth, over the tongue, and seal your lips tightly
around the mouthpiece."
...
15. A nurse assesses a client with bladder cancer who is recovering from a complete cystectomy with ileal conduit. Which assessment finding should alert the nurse to urgently contact the health care provider?
a. The ileostomy is draining blood-tinged urine.
b. There is serous sanguineous drainage present on the surgical dressing. c. The ileostomy stoma is pale and cyanotic in appearance.
d. Oxygen saturations are 92% on room air.
ANS: C
A pale or cyanotic stoma indicates impaired circulation to the stoma and must be treated to prevent necrosis. Blood-tinged urine and serous sanguineous drainage are expected after this type of surgery. Oxygen saturation of 92% on room air is at the low limit of normal.
1. A nurse is caring for five clients. For which clients would the nurse assess a high risk for developing a pulmonary embolism (PE)? (Select all that apply.)
a. Client who had a reaction to contrast dye yesterday
b. Client with a new spinal cord injury on a rotating bed
c. Middle-aged man with an exacerbation of asthma
d. Older client who is 1-day post hip replacement surgery e. Young obese client with a fractured femur
ANS: B, D, E
Conditions that place clients at higher risk of developing PE include prolonged immobility, central venous catheters, surgery, obesity, advancing age, conditions that increase blood clotting, history of thromboembolism, smoking, pregnancy, estrogen therapy, heart failure, stroke, cancer (particularly lung or prostate), and trauma. A contrast dye reaction and asthma pose no risk for PE.
30. A patient is at high risk for the development of venoocclusive disease (VOD). What assessments does the nurse perform for early detection of this disorder? (Select all that apply.)
a. Jaundice
b. Weight loss
c. Hepatomegaly
d. Right upper quadrant abdominal pain
e. Ascites
a, c, d, e
Which characteristics place women at high risk for myocardial infarction (MI)? (Select all that apply.)
a. Premenopausal
b. Increasing age
c. Family history
d. Abdominal obesity
e. Breast cancer
b. Increasing age
c. Family history
d. Abdominal obesity
Increasing age is a risk factor, especially after 70 years. Family history is a significant risk factor in both men and women. A large waist size and/or abdominal obesity are risk factors for both metabolic syndrome and MI. Premenopausal women are not at higher risk for MI, and breast cancer is not a risk factor for MI.
10. In caring for a patient with hyperpituirarism, which symptoms does the nurse expect the patient to report?
a. Joint pain
b. Visual disturbances
c. Changes in menstruation
d. Increase libido
e. Headache
f. Fatigue
a, b, c, e, f
Joint pain, Visual disturbances, Changes in menstruation, Headache, Fatigue
15. A nurse assesses a client with bladder cancer who is recovering from a complete cystectomy with ileal conduit. Which assessment finding should alert the nurse to urgently contact the health care provider?
a. The ileostomy is draining blood-tinged urine.
b. There is serous sanguineous drainage present on the surgical dressing.
c. The ileostomy stoma is pale and cyanotic in appearance.
d. Oxygen saturations are 92% on room air.
C
A pale or cyanotic stoma indicates impaired circulation to the stoma and must be treated to prevent necrosis. Blood-tinged urine and serous sanguineous drainage are expected after this type of surgery. Oxygen saturation of 92% on room air is at the low limit of normal.
A client comes to the emergency department with a sore throat. Examination reveals redness and swelling of the pharyngeal mucous membranes. Which diagnostic test does the nurse expect will be requested first?
A) Chest x-ray
B) Complete blood count (CBC)
C) Tuberculosis (TB) skin test
D) Throat culture
(Chp. 31; elsevier resources)
D) Throat culture
(Chp. 31; elsevier resources)
A client has Hodgkin's lymphoma, Ann Arbor stage Ib. For what manifestations should the nurse assess the client? (Select all that apply.)
a. Headaches
b. Night sweats
c. Persistent fever
d. Urinary frequency
e. Weight loss
b. Night sweats
c. Persistent fever
e. Weight loss
21. The nurse is caring for a client on the medical-surgical unit who suddenly becomes unresponsive and has no pulse. The cardiac monitor shows the rhythm below:
After calling for assistance and a defibrillator, which action should the nurse take next?
a. Perform a pericardial thump.
b. Initiate cardiopulmonary resuscitation (CPR).
c. Start an 18-gauge intravenous line.
d. Ask the client's family about code status.
ANS: B
The client's rhythm is ventricular fibrillation. This is a lethal rhythm that is best treated with immediate defibrillation. While the nurse is waiting for the defibrillator to arrive, the nurse should start CPR. A pericardial thump is not a treatment for ventricular fibrillation. If the client does not already have an IV, other members of the team can insert one after defibrillation. The client's code status should already be known by the nurse prior to this event.
DIF: Applying/Application REF: 671
KEY: Cardiac electrical conduction| medical emergency
MSC: IntegratedProcess:NursingProcess:Assessment
NOT: Client Needs Category: Physiological Integrity: Physiological Adaptation
10. What are tropic hormones?
a. Hormones that rigger female and male sex characteristics.
b. Hormones that have a direct effect on final target tissues.
c. Hormones produced by the anterior pituitary gland that stimulate other endocrine glands.
d. Hormones that are synthesized in the hypothalamus and stored in the posterior pituitary gland
c
Hormones produced by the anterior pituitary gland that stimulate other endocrine glands.
21. A nurse cares for an older adult client with heart failure. The client states, "I don't know what to do. I don't want to be a burden to my daughter, but I can't do it alone. Maybe I should die." How should the nurse respond?
a. "Would you like to talk more about this?"
b. "You are lucky to have such a devoted daughter."
c. "It is normal to feel as though you are a burden."
d. "Would you like to meet with the chaplain?"
ANS: A
Depression can occur in clients with heart failure, especially older adults. Having the client talk about his or her feelings will help the nurse focus on the actual problem. Open-ended statements allow the client to respond safely and honestly. The other options minimize the client's concerns and do not allow the nurse to obtain more information to provide client-centered care.
DIF: Applying/Application REF: 683 KEY: Heart failure| support| psychosocial response MSC: IntegratedProcess:Caring
NOT: Client Needs Category: Psychosocial Integrity
A nurse is caring for a client who was prescribed high-dose corticosteroid therapy for 1 month to treat a severe inflammatory condition. The clients symptoms have now resolved and the client asks, When can I stop taking these medications? How should the nurse respond?
a. It is possible for the inflammation to recur if you stop the medication.
b. Once you start corticosteroids, you have to be weaned off them.
c. You must decrease the dose slowly so your hormones will work again.
d. The drug suppresses your immune system, which must be built back up.
B (One of the most common causes of adrenal insufficiency, a life-threatening problem, is the sudden cessation of long-term, high-dose corticosteroid therapy. This therapy suppresses the hypothalamic-pituitary-adrenal axis and must be withdrawn gradually to allow for pituitary production of adrenocorticotropic hormone and adrenal production of cortisol. Decreasing hormone therapy slowly ensures self-production of hormone, not hormone effectiveness. Building the clients immune system and rebound inflammation are not concerns related to stopping high-dose corticosteroids.)
1. A nurse assesses a client who has a family history of polycystic kidney disease (PKD). For which clinical manifestations should the nurse assess? (Select all that apply.)
a. Nocturia
b. Flank pain
c. Increased abdominal girth d. Dysuria
e. Hematuria
f. Diarrhea
ANS: B, C, E
Clients with PKD experience abdominal distention that manifests as flank pain and increased abdominal girth. Bloody urine is also present with tissue damage secondary to PKD. Clients with PKD often experience constipation, but would not report nocturia or dysuria.
Question 14 of 41
A 56-year-old client admitted with a diagnosis of acute myelogenous leukemia is prescribed IV cytosine arabinoside for 7 days and an infusion of daunorubicin for the first 3 days. What is the major side effect of this therapy?
a. Bone marrow suppression
b. Liver toxicity
c. Nausea
d. Stomatitis
a
Bone marrow suppression
Intravenous cytosine arabinoside and daunorubicin are a commonly prescribed course of aggressive chemotherapy, and bone marrow suppression is a major side effect. The client is even more at risk for infection than before treatment began. Liver toxicity, nausea, and stomatitis are not the major problems with this therapy.
6. "Remove the mouthpiece from your mouth, keep your lips closed, and hold your breath for at least 10 seconds."
a. 2, 3, 4, 5, 6, 1
b. 3, 4, 5, 1, 6, 2
c. 4, 3, 5, 1, 2, 6
d. 5, 3, 6, 1, 2, 4
ANS: C
The proper order for correctly using an inhaler with a spacer is as follows. Insert the mouthpiece of the inhaler into the nonmouthpiece end of the spacer. Shake the whole unit vigorously three or four times. Place the mouthpiece into the mouth, over the tongue, and seal the lips tightly around it. Press down firmly on the canister of the inhaler to release one dose of medication into the spacer. Breathe in slowly and deeply. Remove the mouthpiece from the mouth, and, keeping the lips closed, hold the breath for at least 10 seconds. Then breathe out slowly. Wait at least 1 minute between puffs.
DIF: Applying/Application REF:
MSC: IntegratedProcess:Teaching/Learning
NOT: Client Needs Category: Physiological Integrity: Pharmacological and Parenteral Therapies
15. A nurse cares for a client who is recovering from a closed percutaneous kidney biopsy. The client states, "My pain has suddenly increased from a 3 to a 10 on a scale of 0 to 10." Which action should the nurse take first?
a. Reposition the client on the operative side.
b. Administer the prescribed opioid analgesic. c. Assess the pulse rate and blood pressure.
d. Examine the color of the client's urine.
ANS: C
An increase in the intensity of pain after a percutaneous kidney biopsy is a symptom of internal hemorrhage. A change in vital signs can indicate that hemorrhage is occurring. Before other actions, the nurse must assess the client's hemodynamic status.
2. When working with women who are taking hormonal birth control, what health promotion measures should the nurse teach to prevent possible pulmonary embolism (PE)? (Select all that apply.)
a. Avoid drinking alcohol.
b. Eat more omega-3 fatty acids. c. Exercise on a regular basis.
d. Maintain a healthy weight.
e. Stop smoking cigarettes.
ANS: C, D, E
Health promotion measures for clients to prevent thromboembolic events such as PE include maintaining a healthy weight, exercising on a regular basis, and not smoking. Avoiding alcohol and eating more foods containing omega-3 fatty acids are heart-healthy actions but do not relate to the prevention of PE.
31. While being interviewed for admission, a patient tells the nurse that he has Christmas disease. What does the nurse document this as?
a. Hemophilia A
b. Hemophilia B
c. Thrombocytopenia
d. Sickle cell disease
b
1. A nurse is caring for a client with a history of renal insufficiency who is scheduled for a cardiac catheterization. Which actions should the nurse take prior to the catheterization? (Select all that apply.)
a. Assess for allergies to iodine.
b. Administer intravenous fluids.
c. Assess blood urea nitrogen (BUN) and creatinine results.
d. Insert a Foley catheter.
e. Administer a prophylactic antibiotic.
f. Insert a central venous catheter.
A, B, C
If the client has kidney disease (as indicated by BUN and creatinine results), fluids and Mucomyst may be given 12 to 24 hours before the procedure for renal protection. The client should be assessed for allergies to iodine, including shellfish; the contrast medium used during the catheterization contains iodine. A Foley catheter and central venous catheter are not required for the procedure and would only increase the clients risk for infection. Prophylactic antibiotics are not administered prior to a cardiac catheterization.
What is the most common cause of hyperthyroidism?
a. Radiation to thyroid
b. Graves' disease
c. Thyroid cancer
d. Thyroiditis
b
A nurse cares for a client who is prescribed a 24-hour urine collection. The unlicensed assistive personnel (UAP) reports that, while pouring urine into the collection container, some urine splashed his hand. Which action should the nurse take next?
a. Ask the UAP if he washed his hands afterward.
b. Have the UAP fill out an incident report.
c. Ask the laboratory if the container has preservative in it.
d. Send the UAP to Employee Health right away.
ANS: A
For safety, the nurse should find out if the UAP washed his or her hands. The UAP should do this for two reasons. First, it is part of Standard Precautions to wash hands after client care. Second, if the container did have preservative in it, this would wash it away. The preservative may be caustic to the skin. The nurse can call the laboratory while the UAP is washing hands, if needed. The UAP would then need to fill out an incident or exposure report and may or may not need to go to Employee Health. The UAP also needs further education on Standard Precautions, which include wearing gloves.
2. An emergency room nurse assesses a female client. Which assessment findings should alert the nurse to request a prescription for an electrocardiogram? (Select all that apply.)
a. Hypertension
b. Fatigue despite adequate rest
c. Indigestion
d. Abdominal pain
e. Shortness of breath
B, C, E
Women may not have chest pain with myocardial infarction, but may feel discomfort or indigestion. They often present with a triad of symptomsindigestion or feeling of abdominal fullness, feeling of chronic fatigue despite adequate rest, and feeling unable to catch their breath. Frequently, women are not diagnosed and therefore are not treated adequately. Hypertension and abdominal pain are not associated with acute coronary syndrome.
4. A client has been diagnosed with an empyema. What interventions should the nurse anticipate providing to this client? (Select all that apply.)
a. Assisting with chest tube insertion
b. Facilitating pleural fluid sampling
c. Performing frequent respiratory assessment d. Providing antipyretics as needed
e. Suctioning deeply every 4 hours
ANS: A, B, C, D
The client with an empyema is often treated with chest tube insertion, which facilitates obtaining samples of the pleural fluid for analysis and re-expands the lungs. The nurse should perform frequent respiratory system assessments. Antipyretic medications are also used. Suction is only used when needed and is not done deeply to prevent tissue injury.
32. Which hematologic disorder is most likely to cause the patient to have joint problems?
a. Thrombocytopenia
b. Aplastic anemia
c. Hemophilia
d. Warm antibody anemia
c
15. Which renal change associated with aging does the nurse expect an older adult patient to report?
a. Nocturnal polyuria
b. Micturition
c. Hematuria
d. Dysuria
a
15. The nurse is caring for a patient receiving gentamicin. Because this drug has potential for nephrotoxicity, which laboratory results does the nurse monitor? (Select all that apply.)
a. Blood urea nitrogen
b. Creatinine
c. Drug peak and trough levels
d. Prothrombin time (PT)
e. Platelet count
f. Hemoglobin and hematocrit
a, b, c
28. A nurse evaluates the following arterial blood gas and vital sign results for a client with chronic obstructive pulmonary disease (COPD):
Which action should the nurse take first?
a. Administer a short-acting beta2 agonist inhaler.
b. Document the findings as normal for a client with COPD.
c. Teach the client diaphragmatic breathing techniques.
d. Initiate oxygenation therapy to increase saturation to 92%.
ANS: D
Oxygen should be administered to a client who is hypoxic even if the client has COPD and is a carbon dioxide retainer. The other interventions do not address the client's hypoxia, which is the priority.
DIF: Applying/Application REF: 563
KEY: Oxygen therapy| respiratory distress/failure
MSC: Integrated Process: Nursing Process: Implementation
NOT: Client Needs Category: Safe and Effective Care Environment: Management of Care
MULTIPLE RESPONSE
A client has a platelet count of 25,000/mm3. What actions does the nurse delegate to the unlicensed assistive personnel (UAP)? (Select all that apply.)
a. Assist with oral hygiene using a firm toothbrush.
b. Give the client an enema if he or she is constipated.
c. Help the client choose soft foods from the menu.
d. Shave the male client with an electric razor.
e. Use a lift sheet when needed to re-position the client.
c. Help the client choose soft foods from the menu.
d. Shave the male client with an electric razor.
e. Use a lift sheet when needed to re-position the client.
22. A nurse teaches a client with heart failure about energy conservation. Which statement should the nurse include in this client's teaching?
a. "Walk until you become short of breath, and then walk back home."
b. "Gather everything you need for a chore before you begin."
c. "Pull rather than push or carry items heavier than 5 pounds."
d. "Take a walk after dinner every day to build up your strength."
ANS: B
A client who has heart failure should be taught to conserve energy. Gathering all supplies needed for a chore at one time decreases the amount of energy needed. The client should not walk until becoming short of breath because he or she may not make it back home. Pushing a cart takes less energy than pulling or lifting. Although walking after dinner may help the client, the nurse should teach the client to complete activities when he or she has the most energy. This is usually in the morning.
DIF: Applying/Application REF: 696
KEY: Heart failure| functional ability| patient education
MSC: IntegratedProcess:Teaching/Learning
NOT: Client Needs Category: Health Promotion and Maintenance
Ch.62 p. 1280, Health Promotion and Maintenance
The client who is about to have a unilateral adrenalectomy for an adenoma that is causing hypercortisolism asks the nurse if she will have to continue the severe sodium restriction after surgery. What is the nurse's best response?
A. "No, once the tumor has been removed and your cortisol levels have normalized, you will not retain excess sodium anymore."
B. "No, after surgery you will have to take oral cortisol, which can easily be controlled so that your sodium levels do not rise."
C. Yes, the fact that you are retaining sodium and have high blood pressure is related to your age and lifestyle, not the tumor."
D. "Yes, sodium is very bad for people and everyone needs to eliminate sodium completely from their diets for the rest of their lives."
Answer: A
Rationale: A tumor secreting excessive amounts of cortisol is this patient's reason for needing to severely restrict her sodium. After the tumor is removed, she will not have hypercortisolism but may have to take oral cortisol until the remaining adrenal gland begins to secrete sufficient cortisol. She will no longer experience severe sodium retention. Although people in North America tend to have high-sodium diets and many could stand to reduce their sodium intake, sodium is an essential element and cannot be eliminated from the diet.
15. A patient with PKD usually experiences constipation. What does the nurse recommend?
a. Increased dietary fiber and increased fluids
b. Decreased dietary fiber and laxatives
c. Daily laxatives and increased exercise
e. Ta-water enemas and fiber supplements
a
3. A client with a new pulmonary embolism (PE) is anxious. What nursing actions are most appropriate? (Select all that apply.)
a. Acknowledge the frightening nature of the illness.
b. Delegate a back rub to the unlicensed assistive personnel (UAP).
c. Give simple explanations of what is happening.
d. Request a prescription for antianxiety medication.
e. Stay with the client and speak in a quiet, calm voice.
ANS: A, B, C, E
Clients with PEs are often anxious. The nurse can acknowledge the client's fears, delegate comfort measures, give simple explanations the client will understand, and stay with the client. Using a calm, quiet voice is also reassuring. Sedatives and antianxiety medications are not used routinely because they can contribute to hypoxia. If the client's anxiety is interfering with diagnostic testing or treatment, they can be used, but there is no evidence that this is the case.
Question 15 of 41
The nurse is transfusing a unit of whole blood to a client when the health care provider requests the following: "Furosemide (Lasix) 20 mg IV push." What does the nurse do?
a. Add furosemide to the normal saline that is infusing with the blood.
b. Administer furosemide to the client intramuscularly (IM).
c. Piggyback furosemide into the infusing blood.
d. Wait until the transfusion has been completed to administer furosemide.
d
Wait until the transfusion has been completed to administer furosemide.
Completing the transfusion before administering furosemide is the best course of action in this scenario. Drugs are not to be administered with infusing blood products; they can interact with the blood, causing risks for the client. Stopping the infusing blood to administer the drug and then restarting it is also not the best decision. Changing the admission route is not a nursing decision.
The nurse is preparing to teach a client that metabolic syndrome can increase the risk for myocardial infarction (MI). Which signs of metabolic syndrome should the nurse include in the discussion? (Select all that apply.)
Truncal obesity
Hypercholesterolemia
Elevated homocysteine levels
Glucose intolerance
Client taking losartan (Cozaar)
Truncal obesity
Hypercholesterolemia
Glucose intolerance
Client taking losartan (Cozaar)
A large waist size (excessive abdominal fat causing central obesity)—40 inches (102 cm) or greater for men, 35 inches (88 cm) or greater for women—is a sign of metabolic syndrome. Decreased high-density lipoprotein cholesterol (HDL-C) (usually with high low-density lipoprotein cholesterol)—HDL-C less than 40 mg/dL for men or less than 50 mg/dL for women—or taking an anticholesterol drug is a sign of metabolic syndrome. Increased fasting blood glucose (caused by diabetes, glucose intolerance, or insulin resistance) is included in the constellation of metabolic syndrome. Blood pressure greater than 130/85 mm Hg or taking antihypertensive medication indicates metabolic syndrome. Although elevated homocysteine levels may predispose to atherosclerosis, they are not part of metabolic syndrome.
After teaching a client with type 2 diabetes mellitus who is prescribed nateglinide (Starlix), the nurse assesses the client's understanding. Which statement made by the client indicates a correct understanding of the prescribed therapy?
a. "I'll take this medicine during each of my meals."
b. "I must take this medicine in the morning when I wake."
c. "I will take this medicine before I go to bed."
d. "I will take this medicine immediately before I eat."
ANS: D
Nateglinide is an insulin secretagogue that is designed to increase meal-related insulin secretion. It should be taken immediately before each meal. The medication should not be taken without eating as it will decrease the client's blood glucose levels. The medication should be taken before meals instead of during meals.
22. After assessing a client who is receiving an amiodarone intravenous infusion for unstable ventricular tachycardia, the nurse documents the findings and compares these with the previous assessment findings:
Vital Signs
Nursing Assessment
Time: 0800
Temperature: 98° F
Heart rate: 68 beats/min
Blood pressure: 135/60 mm Hg Respiratory rate: 14 breaths/min Oxygen saturation: 96% Oxygen therapy: 2 L nasal
cannula
Time: 1000
Temperature: 98.2° F
Heart rate: 50 beats/min
Blood pressure: 132/57 mm Hg Respiratory rate: 16 breaths/min Oxygen saturation: 95% Oxygen therapy: 2 L nasal
cannula
Time: 0800
Client alert and oriented.
Cardiac rhythm: normal sinus rhythm. Skin: warm, dry, and appropriate for
race.
Respirations equal and unlabored. Client denies shortness of breath and
chest pain.
Time: 1000
Client alert and oriented.
Cardiac rhythm: sinus bradycardia. Skin: warm, dry, and appropriate for
race.
Respirations equal and unlabored. Client denies shortness of breath and
chest pain.
Client voids 420 mL of clear yellow
urine.
Based on the assessments, which action should the nurse take?
a. Stop the infusion and flush the IV.
b. Slow the amiodarone infusion rate.
c. Administer IV normal saline.
d. Ask the client to cough and deep breathe.
ANS: B
IV administration of amiodarone may cause bradycardia and atrioventricular (AV) block. The correct action for the nurse to take at this time is to slow the infusion, because the client is asymptomatic and no evidence reveals AV block that might require pacing. Abruptly ceasing the medication could allow fatal dysrhythmias to occur. The administration of IV fluids and encouragement of coughing and deep breathing exercises are not indicated, and will not increase the client's heart rate.
DIF: Applying/Application REF: 660
KEY: Cardiac electrical conduction| medication
MSC: Integrated Process: Nursing Process: Implementation
NOT: Client Needs Category: Physiological Integrity: Pharmacological and Parenteral Therapies
Question 8 of 16
Which gland releases catecholamines?
a. Adrenal
b. Pancreas
c. Parathyroid
d. Thyroid
a
Adrenal
The adrenal medulla releases catecholamines in response to stimulation of the sympathetic nervous system. The principal hormones of the pancreas are insulin, glucagon, and somatostatin. Parathyroid hormone is the principal hormone of the parathyroid gland. Triiodothyronine (T3), thyroxine (T4), and calcitonin are the principal hormones of the thyroid.
A student nurse is helping a registered nurse with a blood transfusion. Which actions by the student are most appropriate? (Select all that apply.)
a. Hanging the blood product using normal saline and a filtered tubing set
b. Taking a full set of vital signs prior to starting the blood transfusion
c. Telling the client someone will remain at the bedside for the first 5 minutes
d. Using gloves to start the client's IV if needed and to handle the blood product
e. Verifying the client's identity, and checking blood compatibility and expiration time
a. Hanging the blood product using normal saline and a filtered tubing set
b. Taking a full set of vital signs prior to starting the blood transfusion
d. Using gloves to start the client's IV if needed and to handle the blood product
Which symptom of pneumonia may present differently in the older adult than in the younger adult?
A) Crackles on auscultation
B) Fever
C) Headache
D) Wheezing
(Chp. 31; elsevier resources)
B) Fever
(Chp. 31; elsevier resources)
15. A patient has been started on oxybutynin (Ditropan) for urinary incontinence. What is the major action of this medication?
a. Increases blood flow to the urethra
b. Blocks acetylcholine receptors
c. Causes slight numbing of the bladder
d. Relaxes bladder muscles
d
16. The nurse is teaching a client with chronic kidney disease (CKD) about the sodium restriction needed in the diet to prevent edema and hypertension. Which statement by the client indicates more teaching is needed?
a. "I am thrilled that I can continue to eat fast food."
b. "I will cut out bacon with my eggs every morning."
c. "My cooking style will change by not adding salt."
d. "I will probably lose weight by cutting out potato chips."
ANS: A
Fast food restaurants usually serve food that is high in sodium. This statement indicates that more teaching needs to occur. The other statements show a correct understanding of the teaching.
1. A nurse assesses a client with asthma and notes bilateral wheezing, decreased pulse oxygen saturation, and suprasternal retraction on inhalation. Which actions should the nurse take? (Select all that apply.)
555 KEY: Medication safety
Arterial Blood Gas Results
Vital Signs
pH = 7.32
PaCO2 = 62 mm Hg PaO2 = 46 mm Hg HCO3- = 28 mEq/L
Heart rate = 110 beats/min Respiratory rate = 12 breaths/min Blood pressure = 145/65 mm Hg Oxygen saturation = 76%
a. Administer prescribed salmeterol (Serevent) inhaler.
b. Assess the client for a tracheal deviation.
c. Administer oxygen to keep saturations greater than 94%.
d. Perform peak expiratory flow readings.
e. Administer prescribed albuterol (Proventil) inhaler.
ANS: C, E
Suprasternal retraction caused by inhalation usually indicates that the client is using accessory muscles and is having difficulty moving air into the respiratory passages because of airway narrowing. Wheezing indicates a narrowed airway; a decreased pulse oxygen saturation also supports this finding. The asthma is not responding to the medication, and intervention is needed. Administration of a rescue inhaler is indicated, probably along with administration of oxygen. The nurse would not do a peak flow reading at this time, nor would a code be called. Midline trachea is a normal and expected finding.
DIF: Applying/Application REF: 575
KEY: Respiratory distress/failure
MSC: Integrated Process: Nursing Process: Implementation
NOT: Client Needs Category: Physiological Integrity: Pharmacological and Parenteral Therapies
Question 16 of 41
The nurse is to administer packed red blood cells to a client. How does the nurse ensure proper client identification?
a. Asks the client's name
b. Checks the client's armband
c. Reviews all information with another registered nurse
d. Verifies the client's room number
c
Reviews all information with another registered nurse
With another registered nurse, verify the client by name and number, check blood compatibility, and note expiration time. Human error is the most common cause of ABO incompatibility reactions, even for experienced nurses. Asking the client's name and checking the client's armband are not adequate for identifying the client before transfusion therapy. Using the room number to verify client identification is never appropriate.
22. A client had an acute myocardial infarction. What assessment finding indicates to the nurse that a significant complication has occurred?
a. Blood pressure that is 20 mm Hg below baseline
b. Oxygen saturation of 94% on room air
c. Poor peripheral pulses and cool skin
d. Urine output of 1.2 mL/kg/hr for 4 hours
ANS: C
Poor peripheral pulses and cool skin may be signs of impending cardiogenic shock and should be reported immediately. A blood pressure drop of 20 mm Hg is not worrisome. An oxygen saturation of 94% is just slightly below normal. A urine output of 1.2 mL/kg/hr for 4 hours is normal.
DIF: Remembering/Knowledge REF: 772
KEY: Coronary artery disease| critical rescue| nursing assessment
MSC: Integrated Process: Nursing Process: Analysis
NOT: Client Needs Category: Physiological Integrity: Reduction of Risk Potential
Ch.61 Question 10 of 16
The nurse should encourage fluids every 2 hours for older adult clients because of a decrease in which factor?
a. Antidiuretic hormone (ADH) production
b. General metabolism
c. Glucose tolerance
d. Ovarian production of estrogen
a
Antidiuretic hormone (ADH) production
A decrease in ADH production causes urine to be more dilute, so urine might not concentrate when fluid intake is low. The older adult is at greater risk for dehydration as a result of urine loss. A decrease in general metabolism causes decreased tolerance to cold, decreased appetite, and decreased heart rate and blood pressure; it is not related to fluid intake or hydration. A decrease in glucose tolerance does not affect fluid intake or hydration. A decrease in estrogen production causes a decrease in bone density and is not related to fluid intake and hydration.
16. A nurse obtains the health history of a client with a suspected diagnosis of bladder cancer. Which question should the nurse ask when determining this client's risk factors?
a. "Do you smoke cigarettes?"
b. "Do you use any alcohol?"
c. "Do you use recreational drugs?"
d. "Do you take any prescription drugs?"
ANS: A
Smoking is known to be a factor that greatly increases the risk of bladder cancer. Alcohol use, recreational drug use, and prescription drug use (except medications that contain phenacetin) are not known to increase the risk of developing bladder cancer.
. The nurse caring for mechanically ventilated clients uses best practices to prevent ventilator-associated pneumonia. What actions are included in this practice? (Select all that apply.)
a. Adherence to proper hand hygiene
b. Administering anti-ulcer medication
c. Elevating the head of the bed
d. Providing oral care per protocol
e. Suctioning the client on a regular schedule
ANS: A, B, C, D
The "ventilator bundle" is a group of care measures to prevent ventilator-associated pneumonia. Actions in the bundle include using proper hand hygiene, giving anti-ulcer medications, elevating the head of the bed, providing frequent oral care per policy, preventing aspiration, and providing pulmonary hygiene measures. Suctioning is done as needed.
33. The nurse is inwerting an intravenous needle into an older patient for the purpose of administering a blood transfusion. Which size needle should the nurse select?
a. 22-gauge needle
b. 20-gauge needle
c. 19-gauge needle
d. 23-gauge butterfly needle
b
The nurse is preparing to teach a client that metabolic syndrome can increase the risk for myocardial infarction (MI). Which signs of metabolic syndrome should the nurse include in the discussion? (Select all that apply.)
a. Truncal obesity
b. Hypercholesterolemia
c. Elevated homocysteine levels
d. Glucose intolerance
e. Client taking losartan (Cozaar)
a. Truncal obesity
b. Hypercholesterolemia
d. Glucose intolerance
e. Client taking losartan (Cozaar)
A large waist size (excessive abdominal fat causing central obesity)—40 inches (102 cm) or greater for men, 35 inches (88 cm) or greater for women—is a sign of metabolic syndrome. Decreased high-density lipoprotein cholesterol (HDL-C) (usually with high low-density lipoprotein cholesterol)—HDL-C less than 40 mg/dL for men or less than 50 mg/dL for women—or taking an anticholesterol drug is a sign of metabolic syndrome. Increased fasting blood glucose (caused by diabetes, glucose intolerance, or insulin resistance) is included in the constellation of metabolic syndrome. Blood pressure greater than 130/85 mm Hg or taking antihypertensive medication indicates metabolic syndrome. Although elevated homocysteine levels may predispose to atherosclerosis, they are not part of metabolic syndrome.
A client has hyperparathyroidism. Which incident witnessed by the nurse requires the nurse's intervention?
A. The client eating a morning meal of cereal and fruit
B. The physical therapist walking with the client in the hallway
C. Unlicensed assistive personnel pulling the client up in bed by the shoulders
D. Visitors talking with the client about going home
C.
The client with hyperparathyroidism is at risk for pathologic fracture. All members of the health care team must move the client carefully. A lift sheet should be used to re-position the client. The client with hyperparathyroidism is not restricted from eating and should maintain a balanced diet. The client can benefit from moderate exercise and physical therapy, and is not restricted from having visitors.
MULTIPLE RESPONSE
1. A nurse cares for a client with congestive heart failure who has a regular cardiac rhythm of 128 beats/min. For which physiologic alterations should the nurse assess? (Select all that apply.)
a. Decrease in cardiac output
b. Increase in cardiac output
c. Decrease in blood pressure
d. Increase in blood pressure
e. Decrease in urine output
f. Increase in urine output
ANS: A, D, E
Elevated heart rates in a healthy client initially cause blood pressure and cardiac output to increase. However, in a client who has congestive heart failure or a client with long-term tachycardia, ventricular filling time, cardiac output, and blood pressure eventually decrease. As cardiac output and blood pressure decrease, urine output will fall.
DIF: Applying/Application REF: 657
KEY: Cardiac electrical conduction| heart failure
MSC: IntegratedProcess:NursingProcess:Assessment
NOT: Client Needs Category: Physiological Integrity: Physiological Adaptation
A nurse cares for a client who is prescribed a 24-hour urine collection. The unlicensed assistive personnel (UAP) reports that, while pouring urine into the collection container, some urine splashed his hand. Which action should the nurse take next?
a. Ask the UAP if he washed his hands afterward.
b. Have the UAP fill out an incident report.
c. Ask the laboratory if the container has preservative in it.
d. Send the UAP to Employee Health right away.
ANS: A
For safety, the nurse should find out if the UAP washed his or her hands. The UAP should do this for two reasons. First, it is part of Standard Precautions to wash hands after client care. Second, if the container did have preservative in it, this would wash it away. The preservative may be caustic to the skin. The nurse can call the laboratory while the UAP is washing hands, if needed. The UAP would then need to fill out an incident or exposure report and may or may not need to go to Employee Health. The UAP also needs further education on Standard Precautions, which include wearing gloves.
16. A nurse obtains the health history of a client with a suspected diagnosis of bladder cancer. Which question should the nurse ask when determining this clients risk factors?
a. Do you smoke cigarettes?
b. Do you use any alcohol?
c. Do you use recreational drugs?
d. Do you take any prescription drugs?
A
Smoking is known to be a factor that greatly increases the risk of bladder cancer. Alcohol use, recreational drug use, and prescription drug use (except medications that contain phenacetin) are not known to increase the risk of developing bladder cancer.
A student nurse is learning about blood transfusion compatibilities. What information does this include? (Select all that apply.)
a. Donor blood type A can donate to recipient blood type AB.
b. Donor blood type B can donate to recipient blood type O.
c. Donor blood type AB can donate to anyone.
d. Donor blood type O can donate to anyone.
e. Donor blood type A can donate to recipient blood type B.
a. Donor blood type A can donate to recipient blood type AB.
d. Donor blood type O can donate to anyone.
The nurse is preparing to admit an adult client with pertussis. Which symptom does the nurse anticipate finding in this client?
A) "Whooping" after a cough
B) Hemoptysis
C) Mild cold-like symptoms
D) Post-cough emesis
(Chp. 31; elsevier resources)
D) Post-cough emesis
(Chp. 31; elsevier resources)
2. A nurse assesses a client with nephrotic syndrome. For which clinical manifestations should the nurse assess? (Select all that apply.)
a. Proteinuria
b. Hypoalbuminemia
c. Dehydration
d. Lipiduria
e. Dysuria
f. Costovertebral angle (CV A) tenderness
ANS: A, B, D
Nephrotic syndrome is caused by glomerular damage and is characterized by proteinuria (protein level higher than 3.5 g/24 hr), hypoalbuminemia, edema, and lipiduria. Fluid overload leading to edema and hypertension is common with nephrotic syndrome; dehydration does not occur. Dysuria is present with cystitis. CVA tenderness is present with inflammatory changes in the kidney.
5. A nurse is caring for a client who is on mechanical ventilation. What actions will promote comfort in this client? (Select all that apply.)
a. Allow visitors at the client's bedside.
b. Ensure the client can communicate if awake.
c. Keep the television tuned to a favorite channel. d. Provide back and hand massages when turning. e. Turn the client every 2 hours or more.
ANS: A, B, D, E
There are many basic care measures that can be employed for the client who is on a ventilator. Allowing visitation, providing a means of communication, massaging the client's skin, and routinely turning and repositioning the client are some of them. Keeping the TV on will interfere with sleep and rest.
Question 17 of 41
The nurse is caring for a client with neutropenia who has a suspected infection. Which intervention does the nurse implement first?
a. Hydrate the client with 1000 mL of IV normal saline.
b. Initiate the administration of prescribed antibiotics.
c. Obtain requested cultures.
d. Place the client on Bleeding Precautions.
c
Obtain requested cultures.
Obtaining cultures to identify the infectious agent correctly is the priority for this client. Hydrating the client is not the priority. Administering antibiotics is important, but antibiotics should always be started after cultures are obtained. Placing the client on Bleeding Precautions is unnecessary.
23. A nurse is caring for a client with acute pericarditis who reports substernal precordial pain that radiates to the left side of the neck. Which nonpharmacologic comfort measure should the nurse implement?
a. Apply an ice pack to the client's chest.
b. Provide a neck rub, especially on the left side.
c. Allow the client to lie in bed with the lights down.
d. Sit the client up with a pillow to lean forward on.
ANS: D
Pain from acute pericarditis may worsen when the client lays supine. The nurse should position the client in a comfortable position, which usually is upright and leaning slightly forward. Pain is decreased by using gravity to take pressure off the heart muscle. An ice pack and neck rub will not relieve this pain.
DIF: Applying/Application REF: 699
KEY: Nonpharmacologic pain management
MSC: Integrated Process: Nursing Process: Implementation
NOT: Client Needs Category: Physiological Integrity: Basic Care and Comfort
10. What is the most common cause of hyperthyroidism?
a. Radiation to thyroid
b. Graves' disease
c. Thyroid cancer
d. Thyroiditis
b
16. An older adult male patient has a history of an enlarged prostate. The patient is most likely to report which symptom associated with this condition?
a. Inability to sense the urge to void
b. Difficulty starting the urine stream
c. Excreting large amounts of very dilute urine
d. Burning sensation when urinating
b
2. A nurse assesses a client who has a mediastinal chest tube. Which symptoms require the nurse's immediate intervention? (Select all that apply.)
a. Production of pink sputum
b. Tracheal deviation
c. Pain at insertion site
d. Sudden onset of shortness of breath
e. Drainage greater than 70 mL/hr
f. Disconnection at Y site
ANS: B, D, E, F
Immediate intervention is warranted if the client has tracheal deviation because this could indicate a tension pneumothorax. Sudden shortness of breath could indicate dislodgment of the tube, occlusion of the tube, or pneumothorax. Drainage greater than 70 mL/hr could indicate hemorrhage. Disconnection at the Y site could result in air entering the tubing. Production of pink sputum, oxygen saturation less than 95%, and pain at the insertion site are not signs/symptoms that would require immediate intervention.
DIF: Applying/Application REF: 579
KEY: Drain| respiratory distress/failure
MSC: IntegratedProcess:NursingProcess:Assessment
NOT: Client Needs Category: Physiological Integrity: Physiological Adaptation
34. A patient is receiving a blood transfusion. Which solution does the nurse administer with the blood?
a. Ringer's lactate
b. Normal saline
c. Dextrose in water
d. Dextrose in saline
b
3. A nurse assesses a client who is recovering after a coronary catheterization. Which assessment findings in the first few hours after the procedure require immediate action by the nurse? (Select all that apply.)
a. Blood pressure of 140/88 mm Hg
b. Serum potassium of 2.9 mEq/L
c. Warmth and redness at the site
d. Expanding groin hematoma
e. Rhythm changes on the cardiac monitor
B, D, E
In the first few hours postprocedure, the nurse monitors for complications such as bleeding from the insertion site, hypotension, acute closure of the vessel, dye reaction, hypokalemia, and dysrhythmias. The clients blood pressure is slightly elevated but does not need immediate action. Warmth and redness at the site would indicate an infection, but this would not be present in the first few hours.
10. A nurse plans care for a client with hyperparathyroidism. Which intervention should the nurse include in this client's plan of care?
a. Ask the client to ambulate in the hallway twice a day.
b. Use a lift sheet to assist the client with position changes.
c. Provide the client with a soft-bristled toothbrush for oral care.
d. Instruct the unlicensed assistive personnel to strain the client's urine for stones.
b. Hyperparathyroidism causes increased resorption of calcium from the bones, increasing the risk for pathologic fractures. Using a lift sheet when moving or positioning the client, instead of pulling on the client, reduces the risk of bone injury. Hyperparathyroidism can cause kidney stones, but not every client will need to have urine strained. The priority is preventing injury. Ambulating in the hall and using a soft toothbrush are not specific interventions for this client.
Which atypical symptoms may be present in a female client experiencing myocardial infarction (MI)? (Select all that apply.)
a. Sharp, inspiratory chest pain
b. Dyspnea
c. Dizziness
d. Extreme fatigue
e. Anorexia
b. Dyspnea
c. Dizziness
d. Extreme fatigue
Many women who experience an MI present with dyspnea, light-headedness, and fatigue. Sharp, pleuritic pain is more consistent with pericarditis or pulmonary embolism. Anorexia is neither a typical nor an atypical sign of MI.
A nurse cares for a client who is prescribed pioglitazone (Actos). After 6 months of therapy, the client reports that his urine has become darker since starting the medication. Which action should the nurse take?
a. Assess for pain or burning with urination.
b. Review the client's liver function study results.
c. Instruct the client to increase water intake.
d. Test a sample of urine for occult blood.
ANS: B
Thiazolidinediones (including pioglitazone) can affect liver function; liver function should be assessed at the start of therapy and at regular intervals while the client continues to take these drugs. Dark urine is one indicator of liver impairment because bilirubin is increased in the blood and is excreted in the urine. The nurse should check the client's most recent liver function studies. The nurse does not need to assess for pain or burning with urination and does not need to check the urine for occult blood. The client does not need to be told to increase water intake.
16. A young female patient reports experiencing burning with urination. What question does the nurse ask to differentiate between a vaginal infection and a urinary infection?
a. "Have you noticed any blood in the urine?"
b. "Have you had recent sexual intercourse?"
c. "Have you noticed any vaginal discharge?"
d. "Have you had fever or chills?"
c
A client with chronic anemia has had many blood transfusions. What medications does the nurse anticipate teaching the client about adding to the regimen? (Select all that apply.)
a. Azacitidine (Vidaza)
b. Darbepoetin alfa (Aranesp)
c. Decitabine (Dacogen)
d. Epoetin alfa (Epogen)
e. Methylprednisolone (Solu-Medrol)
b. Darbepoetin alfa (Aranesp)
d. Epoetin alfa (Epogen)
A client with tuberculosis (TB) who is homeless and has been living in shelters for the past 6 months asks the nurse why he must take so many medications. What information will the nurse provide in answering this question? Select all that apply.
A) Combination drug therapy is effective in preventing transmission.
B) Combination drug therapy is the most effective method of treating TB.
C) Combination drug therapy will decrease the length of required treatment to 2 months.
D) Multiple drug regimens destroy organisms as quickly as possible.
E) The use of multiple drugs reduces the emergence of drug-resistant organisms.
(Chp. 31; elsevier resources)
A) Combination drug therapy is effective in preventing transmission.
B) Combination drug therapy is the most effective method of treating TB.
D) Multiple drug regimens destroy organisms as quickly as possible.
E) The use of multiple drugs reduces the emergence of drug-resistant organisms.
(Chp. 31; elsevier resources)
16. According to the RIFLE classification (Risk, Injury, Failure, Loss, End-stage kidney failure). How would the nurse interpret the following data? Serum creatinine increased x 1.5 or glomerular filtration rate (GFR) decrease > 25 %; Urine output is <0.5 mL/kg/hr for ≥6 hours.
a. Risk stage
b. Injury stage
c. Failure stage
d. End-stage kidney disease (ESKD)
a
3. A nurse teaches a client who has chronic obstructive pulmonary disease. Which statements related to nutrition should the nurse include in this client's teaching? (Select all that apply.) a. "Avoid drinking fluids just before and during meals."
b. "Rest before meals if you have dyspnea."
c. "Have about six small meals a day."
d. "Eat high-fiber foods to promote gastric emptying."
e. "Increase carbohydrate intake for energy."
ANS: A, B, C
Fluids can make a client feel bloated and should be avoided with meals. Resting before the meal will help a client with dyspnea. Six small meals a day also will help to decrease bloating. Fibrous foods can produce gas, which can cause abdominal bloating and can increase shortness of breath. The client should increase calorie and protein intake to prevent malnourishment. The client should not increase carbohydrate intake as this will increase carbon dioxide production and increase the client's risk of for acidosis.
DIF: Applying/Application REF: 565
KEY: Nutrition| patient education MSC: Integrated Process: Teaching/Learning NOT: Client Needs Category: Health Promotion and Maintenance
Question 18 of 41
An RN from pediatrics has "floated" to the medical-surgical unit. Which client is assigned to the float nurse?
a. A 42-year-old with sickle cell disease receiving a transfusion of packed red blood cells
b. A 50-year-old with pancytopenia needing assessment of risk factors for aplastic anemia
c. A 55-year-old with folic acid deficiency anemia caused by alcohol abuse who needs counseling
d. A 60-year-old with newly diagnosed polycythemia vera who needs teaching about the disease
a
A 42-year-old with sickle cell disease receiving a transfusion of packed red blood cells
Because sickle cell disease is commonly diagnosed during childhood, the pediatric nurse will be familiar with the disease and with red blood cell transfusion; therefore, he or she should be assigned to the client with sickle cell disease. Aplastic anemia, folic acid deficiency, and polycythemia vera are problems more commonly seen in adult clients who should be cared for by nurses who are more experienced in caring for adults.
Which atypical symptoms may be present in a female client experiencing myocardial infarction (MI)? (Select all that apply.)
Sharp, inspiratory chest pain
Dyspnea
Dizziness
Extreme fatigue
Anorexia
Dyspnea
Dizziness
Extreme fatigue
Many women who experience an MI present with dyspnea, light-headedness, and fatigue. Sharp, pleuritic pain is more consistent with pericarditis or pulmonary embolism. Anorexia is neither a typical nor an atypical sign of MI.
A client with diabetes insipidus (DI) has dry lips and mucous membranes and poor skin turgor. Which intervention does the nurse provide first?
A. Force fluids
B. Offer lip balm
C. Perform a 24-hour urine test
D. Withhold desmopressin acetate (DDAVP)
A.
Dry lips and mucous membranes and poor skin turgor are indications of dehydration, which can occur with DI. This is a serious condition that must be treated rapidly. Encouraging fluids is the initial step, provided the client is able to tolerate oral intake. Lip balm may make the client more comfortable, but does not address the problem of dehydration. A 24-hour urine test will identify loss of electrolytes and adrenal androgen metabolites, but will not correct the dehydration that this client is experiencing. Desmopressin acetate is a synthetic form of antidiuretic hormone that is given to reduce urine production; it improves DI and should not be withheld.
11. A deficiency of which anterior pituitary hormones is considered life-threatening? (Select all that apply.)
a. GH
b. Melanocyte-stimulating hormone (MSH)
c. PRL
d. Thyroid-stimulating hormone (TSH)
e. ACTH
d, e
Thyroid-stimulating hormone (TSH), ACTH
23. A client presents to the emergency department with an acute myocardial infarction (MI) at 1500 (3:00 PM). The facility has 24-hour catheterization laboratory abilities. To meet The Joint Commission's Core Measures set, by what time should the client have a percutaneous coronary intervention performed?
a. 1530 (3:30 PM)
b. 1600 (4:00 PM)
c. 1630 (4:30 PM)
d. 1700 (5:00 PM)
ANS: C
The Joint Commission's Core Measures set for MI includes percutaneous coronary intervention within 90 minutes of diagnosis of myocardial infarction. Therefore, the client should have a percutaneous coronary intervention performed no later than 1630 (4:30 PM).
DIF: Remembering/Knowledge REF: 774
KEY: Coronary artery disease| Core Measures| The Joint Commission
MSC: Integrated Process: Communication and Documentation
NOT: Client Needs Category: Safe and Effective Care Environment: Management of Care
35. A nursing student asks the registered nurse why D5W is contraindicated when transfusing blood. How does the nurse respond?
a. "It causes hemolysis of blood cells."
b. "It dilutes the cells."
c. "It shrinks the blood cells."
d. "It is in the procedure manual."
a
6. The nurse caring for mechanically ventilated clients knows that older adults are at higher risk for weaning failure. What age-related changes contribute to this? (Select all that apply.)
a. Chest wall stiffness
b. Decreased muscle strength c. Inability to cooperate
d. Less lung elasticity
e. Poor vision and hearing
ANS: A, B, D
Age-related changes that increase the difficulty of weaning older adults from mechanical ventilation include increased stiffness of the chest wall, decreased muscle strength, and less elasticity of lung tissue. Not all older adults have an inability to cooperate or poor sensory acuity.
16. A patient with PKD has nocturia. What does the nurse encourage the patient to do?
a. Drink at least 2 liters of fluid daily.
b. Restrict fluid in the evening.
c. Drink 1000 mL early in the morning
d. Add a pinch of salt to water in the evenings.
a
16. A nurse obtains a sterile urine specimen from a client's Foley catheter. After applying a clamp to the drainage tubing distal to the injection port, which action should the nurse take next?
a. Clamp another section of the tube to create a fixed sample section for retrieval.
b. Insert a syringe into the injection port and aspirate the quantity of urine required. c. Clean the injection port cap of the drainage tubing with povidone-iodine solution. d. Withdraw 10 mL of urine and discard it; then withdraw a fresh sample of urine.
ANS: C
It is important to clean the injection port cap of the catheter drainage tubing with an appropriate antiseptic, such as povidone-iodine solution or alcohol. This will help prevent surface contamination before injection of the syringe. The urine sample should be collected directly from the catheter; therefore, a second clamp to create a sample section would not be appropriate. Every sample from the catheter is usable; there is the need to discard the first sample.
A client has recently been released from prison and has just tested positive for tuberculosis (TB). What teaching points does the community health nurse want to stress for this client regarding medications? Select all that apply.
A) Not taking the medication could lead to an infection that is difficult to treat or to total drug resistance.
B) The medications may cause nausea. The client should take them at bedtime.
C) The client is generally not contagious after 2 to 3 consecutive weeks of treatment.
D) These medications must be taken for 2 years.
E) These medications may cause kidney failure.
(Chp. 31; elsevier resources)
A) Not taking the medication could lead to an infection that is difficult to treat or to total drug resistance.
B) The medications may cause nausea. The client should take them at bedtime.
(Chp. 31; elsevier resources)
A nurse is preparing to administer a blood transfusion to an older adult. Understanding age-related changes, what alterations in the usual protocol are necessary for the nurse to implement? (Select all that apply.)
a. Assess vital signs more often.
b. Hold other IV fluids running.
c. Premedicate to prevent reactions.
d. Transfuse smaller bags of blood.
e. Transfuse each unit over 8 hours.
a. Assess vital signs more often.
b. Hold other IV fluids running.
24. The provider requests the nurse start an infusion of an inotropic agent on a client. How does the nurse explain the action of these drugs to the client and spouse?
a. "It constricts vessels, improving blood flow."
b. "It dilates vessels, which lessens the work of the heart."
c. "It increases the force of the heart's contractions."
d. "It slows the heart rate down for better filling."
ANS: C
A positive inotrope is a medication that increases the strength of the heart's contractions. The other options are not correct.
DIF: Remembering/Knowledge REF: 772
KEY: Coronary artery disease| inotropic agents| patient education
MSC: IntegratedProcess:Teaching/Learning
NOT: Client Needs Category: Physiological Integrity: Pharmacological and Parenteral Therapies
11. Which hormone is directly suppressed when circulating levels of cortisol are above normal?
a. Corticotropin-releasing hormone (CRH)
b. ADH
c. Adrenocorticotropic hormone (ACTH)
d. Growth hormone-releasing hormone (GH-RH)
a
Corticotropin-releasing hormone (CRH)
17. A client is placed on fluid restrictions because of chronic kidney disease (CKD). Which assessment finding would alert the nurse that the client's fluid balance is stable at this time? a. Decreased calcium levels
b. Increased phosphorus levels
c. No adventitious sounds in the lungs
d. Increased edema in the legs
ANS: C
The absence of adventitious sounds upon auscultation of the lungs indicates a lack of fluid overload and fluid balance in the client's body. Decreased calcium levels and increased phosphorus levels are common findings with CKD. Edema would indicate a fluid imbalance.
4. A nurse assesses a client with chronic obstructive pulmonary disease. Which questions should the nurse ask to determine the client's activity tolerance? (Select all that apply.) a. "What color is your sputum?"
b. "Do you have any difficulty sleeping?"
c. "How long does it take to perform your morning routine?"
d. "Do you walk upstairs every day?"
e. "Have you lost any weight lately?"
ANS: B, C, E
Difficulty sleeping could indicate worsening breathlessness, as could taking longer to perform activities of daily living. Weight loss could mean increased dyspnea as the client becomes too fatigued to eat. The color of the client's sputum would not assist in determining activity tolerance. Asking whether the client walks upstairs every day is not as pertinent as determining if the client becomes short of breath on walking upstairs, or if the client goes upstairs less often than previously.
DIF: Applying/Application REF: 559 KEY: Functional ability MSC: IntegratedProcess:NursingProcess:Assessment
NOT: Client Needs Category: Physiological Integrity: Physiological Adaptation
Question 19 of 41
The nurse is educating a group of young women who have sickle cell disease (SCD). Which comment from a class member requires correction?
a. "Frequent handwashing is an important habit for me to develop."
b. "Getting an annual 'flu shot' would be dangerous for me."
c. "I must take my penicillin pills as prescribed, all the time."
d. "The pneumonia vaccine is protection that I need."
b
"Getting an annual 'flu shot' would be dangerous for me."
The client with SCD should receive annual influenza and pneumonia vaccinations; this helps prevent the development of these infections, which could cause a sickle cell crisis. Handwashing is a very important habit for the client with SCD to develop because it reduces the risk for infection. Prophylactic penicillin is given to clients with SCD orally twice a day to prevent the development of infection.
When planning care for a client in the emergency department, which interventions are needed in the acute phase of myocardial infarction? (Select all that apply.)
Morphine sulfate
Oxygen
Nitroglycerin
Naloxone
Acetaminophen
Verapamil (Calan, Isoptin)
Morphine sulfate
Oxygen
Nitroglycerin
Morphine is needed to reduce oxygen demand, preload, pain, and anxiety, and nitroglycerin is used to reduce preload and chest pain. Administering oxygen will increase available oxygen for the ischemic myocardium. Naloxone is a narcotic antagonist that is used for overdosage of opiates, not for MI. Acetaminophen may be used for headache related to nitroglycerin. Because of negative inotropic action, calcium channel blockers such as verapamil are used for angina, not for MI.
A nurse cares for a client with adrenal hyperfunction. The client screams at her husband, bursts into tears, and throws her water pitcher against the wall. She then tells the nurse, I feel like I am going crazy. How should the nurse respond?
a. I will ask your doctor to order a psychiatric consult for you.
b. You feel this way because of your hormone levels.
c. Can I bring you information about support groups?
d. I will close the door to your room and restrict visitors.
B (Hypercortisolism can cause the client to show neurotic or psychotic behavior. The client needs to know that these behavior changes do not reflect a true psychiatric disorder and will resolve when therapy results in lower and steadier blood cortisol levels. The client needs to understand this effect and does not need a psychiatrist, support groups, or restricted visitors at this time.)
When planning care for a client in the emergency department, which interventions are needed in the acute phase of myocardial infarction? (Select all that apply.)
a. Morphine sulfate
b. Oxygen
c. Nitroglycerin
d. Naloxone
e. Acetaminophen
f. Verapamil (Calan, Isoptin)
a. Morphine sulfate
b. Oxygen
c. Nitroglycerin
Morphine is needed to reduce oxygen demand, preload, pain, and anxiety, and nitroglycerin is used to reduce preload and chest pain. Administering oxygen will increase available oxygen for the ischemic myocardium. Naloxone is a narcotic antagonist that is used for overdosage of opiates, not for MI. Acetaminophen may be used for headache related to nitroglycerin. Because of negative inotropic action, calcium channel blockers such as verapamil are used for angina, not for MI.
A nurse evaluates laboratory results for a male client who reports fluid secretion from his breasts. Which hormone value should the nurse assess first?
a. Posterior pituitary hormones
b. Adrenal medulla hormones
c. Anterior pituitary hormones
d. Parathyroid hormone
ANS: C
Breast fluid and milk production are induced by the presence of prolactin, secreted from the anterior pituitary gland. The other hormones would not cause fluid secretion from the client's breast.
36. A patient is receiving a blood transfusion through a single-lumen peripherally inserted central catheter. The patient has two other peripheral IVs: one is capped and the other has D5/.45 NS running at a rate of 50 mL/hr. What can be given concurrently through the line that is selected for the blood product?
a. Normal saline
b. Piggyback of 10 mEq potassium chloride
c. Total parenteral nutrition
d. Furosemide (Lasix) 5 mg IV push
a
1. A 242-pound client is being mechanically ventilated. To prevent lung injury, what setting should the nurse anticipate for tidal volume? (Record your answer using a whole number.) ___ mL
ANS:
660 mL
A low tidal volume of 6 mL/kg is used to prevent lung injury. 242 pounds = 110 kg.
110 kg × 6 mL/kg = 660 mL.
17. A nurse cares for a client who is scheduled for the surgical creation of an ileal conduit. The client states, "I am anxious about having an ileal conduit. What is it like to have this drainage tube?" How should the nurse respond?
a. "I will ask the provider to prescribe you an antianxiety medication."
b. "Would you like to discuss the procedure with your doctor once more?" c. "I think it would be nice to not have to worry about finding a bathroom." d. "Would you like to speak with someone who has an ileal conduit?"
ANS: D
The goal for the client who is scheduled to undergo a procedure such as an ileal conduit is to have a positive self-image and a positive attitude about his or her body. Discussing the procedure candidly with someone who has undergone the same procedure will foster such feelings, especially when the current client has an opportunity to ask questions and voice concerns to someone with first-hand knowledge. Medications for anxiety will not promote a positive self-image and a positive attitude, nor will discussing the procedure once more with the physician or hearing the nurse's opinion.
17. A nurse cares for a client who is scheduled for the surgical creation of an ileal conduit. The client states, I am anxious about having an ileal conduit. What is it like to have this drainage tube? How should the nurse respond?
a. I will ask the provider to prescribe you an antianxiety medication.
b. Would you like to discuss the procedure with your doctor once more?
c. I think it would be nice to not have to worry about finding a bathroom.
d. Would you like to speak with someone who has an ileal conduit?
D
The goal for the client who is scheduled to undergo a procedure such as an ileal conduit is to have a positive self-image and a positive attitude about his or her body. Discussing the procedure candidly with someone who has undergone the same procedure will foster such feelings, especially when the current client has an opportunity to ask questions and voice concerns to someone with first-hand knowledge. Medications for anxiety will not promote a positive self-image and a positive attitude, nor will discussing the procedure once more with the physician or hearing the nurses opinion.
5. A nurse assesses a client who has a chest tube. For which manifestations should the nurse immediately intervene? (Select all that apply.)
a. Production of pink sputum
b. Tracheal deviation
c. Sudden onset of shortness of breath
d. Pain at insertion site
e. Drainage of 75 mL/hr
ANS: B, C
Tracheal deviation and sudden onset of shortness of breath are manifestations of a tension pneumothorax. The nurse must intervene immediately for this emergency situation. Pink sputum is associated with pulmonary edema and is not a complication of a chest tube. Pain at the insertion site and drainage of 75 mL/hr are normal findings with a chest tube.
DIF: Applying/Application REF: 579 KEY: Drain| respiratory distress/failure
MSC: IntegratedProcess:NursingProcess:Assessment
NOT: Client Needs Category: Physiological Integrity: Reduction of Risk Potential
37. To avoid transfusion reaction, the nurse is carefully monitoring the patient during a blood transfusion. When are hemolytic reactions to blood transfusion most likely to occur?
a. 1 mL is sufficient
b. 5 mL is typical
c. Within the first 50 mL
d. Occurs after 100 mL
c
2. A nurse teaches a client with a new permanent pacemaker. Which instructions should the nurse include in this client's teaching? (Select all that apply.)
a. "Until your incision is healed, do not submerge your pacemaker. Only take
showers."
b. "Report any pulse rates lower than your pacemaker settings."
c. "If you feel weak, apply pressure over your generator."
d. "Have your pacemaker turned off before having magnetic resonance imaging
(MRI)."
e. "Do not lift your left arm above the level of your shoulder for 8 weeks."
ANS: A, B, E
The client should not submerge in water until the site has healed; after the incision is healed, the client may take showers or baths without concern for the pacemaker. The client should be instructed to report changes in heart rate or rhythm, such as rates lower than the pacemaker setting or greater than 100 beats/min. The client should be advised of restrictions on physical activity for 8 weeks to allow the pacemaker to settle in place. The client should never apply pressure over the generator and should avoid tight clothing. The client should never have MRI because, whether turned on or off, the pacemaker contains metal. The client should be advised to inform all health care providers that he or she has a pacemaker.
DIF: Applying/Application REF: 675
KEY: Cardiac electrical conduction| patient education
MSC: IntegratedProcess:Teaching/Learning
NOT: Client Needs Category: Health Promotion and Maintenance
The nurse assessing a patient palpates enlargement of the thyroid gland, along with noticeable swelling of the neck. How does the nurse interpret this finding?
a. Globe lag
b. Myxedema
c. Exophthalmos
d. Goiter
d
3. A nurse reviews laboratory results for a client with glomerulonephritis. The client's glomerular filtration rate (GFR) is 40 mL/min as measured by a 24-hour creatinine clearance. How should the nurse interpret this finding? (Select all that apply.)
a. Excessive GFR
b. Normal GFR
c. Reduced GFR
d. Potential for fluid overload e. Potential for dehydration
ANS: C, D
The GFR refers to the initial amount of urine that the kidneys filter from the blood. In the healthy adult, the normal GFR ranges between 100 and 120 mL/min, most of which is reabsorbed in the kidney tubules. A GFR of 40 mL/min is drastically reduced, with the client experiencing fluid retention and risks for hypertension and pulmonary edema as a result of excess vascular fluid.
6. A nurse plans care for a client who has chronic obstructive pulmonary disease and thick, tenacious secretions. Which interventions should the nurse include in this client's plan of care? (Select all that apply.)
a. Ask the client to drink 2 liters of fluids daily.
b. Add humidity to the prescribed oxygen.
c. Suction the client every 2 to 3 hours.
d. Use a vibrating positive expiratory pressure device.
e. Encourage diaphragmatic breathing.
ANS: A, B, D
Interventions to decrease thick tenacious secretions include maintaining adequate hydration and providing humidified oxygen. These actions will help to thin secretions, making them easier to remove by coughing. The use of a vibrating positive expiratory pressure device can also help clients remove thick secretions. Although suctioning may assist with the removal of secretions, frequent suctioning can cause airway trauma and does not support the client's ability to successfully remove secretions through normal coughing. Diaphragmatic breathing is not used to improve the removal of thick secretions.
DIF: Applying/Application REF: 564
KEY: Respiratory distress/failure
MSC: IntegratedProcess:NursingProcess:Implementation
NOT: Client Needs Category: Physiological Integrity: Reduction of Risk Potential
Question 20 of 41
What are serious side effects of antiviral agents prescribed for a client with acute myelogenous leukemia?
Select all that apply.
a. Cardiomyopathy
b. Nephrotoxicity
c. Ototoxicity
d. Stroke
e. Diarrhea
b, c
Nephrotoxicity, Ototoxicity
Antiviral agents, although helpful in combating severe infection, have serious side effects, especially nephrotoxicity and ototoxicity. Cardiomyopathy and stroke are not serious side effects of antiviral agents. Diarrhea is a mild side effect associated with antibiotic therapy.
24. A nurse assesses a client who has mitral valve regurgitation. For which cardiac dysrhythmia should the nurse assess?
a. Preventricular contractions
b. Atrial fibrillation
c. Symptomatic bradycardia
d. Sinus tachycardia
ANS: B
Atrial fibrillation is a clinical manifestation of mitral valve regurgitation and stenosis. Preventricular contractions and bradycardia are not associated with valvular problems. These are usually identified in clients with electrolyte imbalances, myocardial infarction, and sinus node problems. Sinus tachycardia is a manifestation of aortic regurgitation due to a decrease in cardiac output.
DIF: Understanding/Comprehension REF: 692
KEY: Valve disorder| cardiac dysrhythmia
MSC: IntegratedProcess:NursingProcess:Assessment
NOT: Client Needs Category: Physiological Integrity: Reduction of Risk Potential
Ch.63 p. 1288, Physiological Integrity
Which manifestations are most often seen in general hyperthyroidism? Select all that apply.
A. Increased appetite
B. Cold intolerance
C. Constipation
D. Increased sweating
E. Insomnia
F. Palpitations
G. Tremors
H. Weight gain
Answer: A, D, E, F, G
Rationale: The person with any type of hyperthyroidism has increased metabolism, which causes an increased appetite, increased sweating, increased nervous system stimulation (tremors and insomnia), and increased cardiovascular responses (palpitations). In most people with hyperthyroidism, all other systems are also stimulated, causing increased bowel movements (not constipation) and an elevated body temperature (not cold intolerance). Even though appetite is increased, most people lose weight.
4. A nurse reviews a clients laboratory results. Which findings should alert the nurse to the possibility of atherosclerosis? (Select all that apply.)
a. Total cholesterol: 280 mg/dL
b. High-density lipoprotein cholesterol: 50 mg/dL
c. Triglycerides: 200 mg/dL
d. Serum albumin: 4 g/dL
e. Low-density lipoprotein cholesterol: 160 mg/dL
A, C, E
A lipid panel is often used to screen for cardiovascular risk. Total cholesterol, triglycerides, and low-density lipoprotein cholesterol levels are all high, indicating higher risk for cardiovascular disease. High-density lipoprotein cholesterol is within the normal range for both males and females. Serum albumin is not assessed for atherosclerosis.
Ch.61 Question 11 of 16
The nurse is teaching a client about the correct procedure for a 24-hour urine test for creatinine clearance. Which statement by the client indicates a need for further teaching?
a. "I should keep the urine container cool in a separate refrigerator or cooler."
b. "I should not eat any protein when I am collecting urine for this test."
c. "I won't save the first urine sample."
d. "To end the collection, I must empty my bladder, adding it to the collection."
b
"I should not eat any protein when I am collecting urine for this test."
Eating protein does not interfere with collection or testing of the urine sample. Because the specimen must be kept cool, it can be placed in an inexpensive cooler with ice; the client should not keep the specimen container with food or beverages. The timing of the 24-hour collection begins after the initial void. To end a 24-hour urine specimen, emptying the bladder and adding it to the collection is the proper procedure.
17. A nurse cares for a client who is having trouble voiding. The client states, "I cannot urinate in public places." How should the nurse respond?
a. "I will turn on the faucet in the bathroom to help stimulate your urination."
b. "I can recommend a prescription for a diuretic to improve your urine output."
c. "I'll move you to a room with a private bathroom to increase your comfort." d. "I will close the curtain to provide you with as much privacy as possible."
ANS: D
The nurse should provide privacy to clients who may be uncomfortable or have issues related to elimination or the urogenital area. Turning on the faucet and administering a diuretic will not address the client's concern. Although moving the client to a private room with a private bathroom would be nice, this is not realistic. The nurse needs to provide as much privacy as possible within the client's current room.
A client has heparin-induced thrombocytopenia (HIT). The student nurse asks how this is treated. About what drugs does the nurse instructor teach? (Select all that apply.)
a. Argatroban (Argatroban)
b. Bivalirudin (Angiomax)
c. Clopidogrel (Plavix)
d. Lepirudin (Refludan)
e. Methylprednisolone (Solu-Medrol)
a. Argatroban (Argatroban)
b. Bivalirudin (Angiomax)
d. Lepirudin (Refludan)
7. A nurse cares for a client who is prescribed an intravenous prostacyclin agent. Which actions should the nurse take to ensure the client's safety while on this medication? (Select all that apply.)
a. Keep an intravenous line dedicated strictly to the infusion.
b. Teach the client that this medication increases pulmonary pressures.
c. Ensure that there is always a backup drug cassette available.
d. Start a large-bore peripheral intravenous line.
e. Use strict aseptic technique when using the drug delivery system.
ANS: A, C, E
Intravenous prostacyclin agents should be administered in a central venous catheter with a dedicated intravenous line for this medication. Death has been reported when the drug delivery system is interrupted; therefore, a backup drug cassette should also be available. The nurse should use strict aseptic technique when using the drug delivery system. The nurse should teach the client that this medication decreases pulmonary pressures and increases lung blood flow.
DIF: Understanding/Comprehension REF: 571
KEY: Medication administration| safety
MSC: Integrated Process: Nursing Process: Implementation
NOT: Client Needs Category: Physiological Integrity: Pharmacological and Parenteral Therapies
17. Impairment in the thirst mechanism associated with aging makes an older adult patient more vulnerable to which disorder?
a. Hypernatremia
b. Hypocalcemia
c. Hyperkalemia
d. Hypoglycemia
a
49. The nurse is taking the social history from a
client diagnosed with small cell carcinoma of the
lung. Which information is significant for this
disease?
1. The client worked with asbestos for a short
time many years ago.
2. The client has no family history for this type
of lung cancer.
3. The client has numerous tattoos covering
both upper and lower arms.
4. The client has smoked two (2) packs of
cigarettes a day for 20 years.
4. Smoking is the number-one risk factor
for developing cancer of the lung. More
than 85% of lung cancers are attributable
to inhalation of chemicals. There are
more than 400 chemicals in each puff of
cigarette smoke, 17 of which are known to
cause cancer.
TEST-TAKING HINT: If the test taker did not
know this information, option "3" has no anatomical
connection to the lungs and could
be eliminated. This information has been
widely disseminated in the media for more
than 40 years since the Surgeon General's office
first warned about the dangers of smoking
in the early 1960s.
A client has received a bone marrow transplant and is waiting for engraftment. What actions by the nurse are most appropriate? (Select all that apply.)
a. Not allowing any visitors until engraftment
b. Limiting the protein in the client's diet
c. Placing the client in protective precautions
d. Teaching visitors appropriate hand hygiene
e. Telling visitors not to bring live flowers or plants
c. Placing the client in protective precautions
d. Teaching visitors appropriate hand hygiene
e. Telling visitors not to bring live flowers or plants
5. A nurse prepares a client for a pharmacologic stress echocardiogram. Which actions should the nurse take when preparing this client for the procedure? (Select all that apply.)
a. Assist the provider to place a central venous access device.
b. Prepare for continuous blood pressure and pulse monitoring.
c. Administer the clients prescribed beta blocker.
d. Give the client nothing by mouth 3 to 6 hours before the procedure.
e. Explain to the client that dobutamine will simulate exercise for this examination.
B, D, E
Clients receiving a pharmacologic stress echocardiogram will need peripheral venous access and continuous blood pressure and pulse monitoring. The client must be NPO 3 to 6 hours prior to the procedure. Education about dobutamine, which will be administered during the procedure, should be performed. Beta blockers are often held prior to the procedure.
11. The nurse assessing a patient palpates enlargement of the thyroid gland, along with noticeable swelling of the neck. How does the nurse interpret this finding?
a. Globe lag
b. Myxedema
c. Exophthalmos
d. Goiter
d
17. A patient reports symptoms indicating a UTI. Results from which diagnostic test will verify a UTI?
a. Urinalysis to test for leukocyte esterase and nitrate
b. Urinalysis for glucose and red blood cells
c. Urinalysis to test for ketones and protein
d. Urinalysis for pH and specific gravity
a
85. The client is admitted to the emergency
department with chest trauma. Which signs/
symptoms indicate to the nurse the diagnosis
of pneumothorax?
1. Bronchovesicular lung sounds and bradypnea.
2. Unequal lung expansion and dyspnea.
3. Frothy, bloody sputum and consolidation.
4. Barrel chest and polycythemia.
2. Unequal lung expansion and dyspnea
indicate a pneumothorax.
TEST-TAKING HINT: The test taker can use
"chest trauma" or "pneumothorax" to help
select the correct answer. Both of these terms
should cause the test taker to select option
"2" because unequal chest expansion would
result from trauma.
Question 21 of 41
An 82-year-old client with anemia is requested to receive 2 units of whole blood. Which assessment findings cause the nurse to discontinue the transfusion because it is unsafe for the client?
Select all that apply.
a. Capillary refill less than 3 seconds
b. Decreased pallor
c. Flattened superficial veins
d. Hypertension
e. Hypotension
f. Rapid, bounding pulse
d, e, f
Hypertension, Hypotension, Rapid, bounding pulse
In an older adult receiving a transfusion, hypertension is a sign of overload, low blood pressure is a sign of a transfusion reaction, and a rapid and bounding pulse is a sign of fluid overload. In this scenario, 2 units, or about a liter of fluid, could be problematic. Capillary refill time that is less than 3 seconds is considered to be normal and would not pose a problem. Increased (not decreased) pallor and cyanosis are signs of a transfusion reaction, while swollen (not flattened) superficial veins are present in fluid overload in older adult clients receiving transfusions.
MULTIPLE RESPONSE
1. A nurse is assessing a client with left-sided heart failure. For which clinical manifestations should the nurse assess? (Select all that apply.)
a. Pulmonary crackles
b. Confusion, restlessness
c. Pulmonary hypertension
d. Dependent edema
e. Cough that worsens at night
ANS: A, B, E
Left-sided heart failure occurs with a decrease in contractility of the heart or an increase in afterload. Most of the signs will be noted in the respiratory system. Right-sided heart failure occurs with problems from the pulmonary vasculature onward including pulmonary hypertension. Signs will be noted before the right atrium or ventricle including dependent edema.
DIF: Remembering/Knowledge REF: 682
KEY: Heart failure| assessment/diagnostic examination
MSC: IntegratedProcess:NursingProcess:Assessment
NOT: Client Needs Category: Physiological Integrity: Physiological Adaptation
11. A nurse cares for a client who is recovering from a parathyroidectomy. When taking the client's blood pressure, the nurse notes that the client's hand has gone into flexion contractions. Which laboratory result does the nurse correlate with this condition?
a. Serum potassium: 2.9 mEq/L
b. Serum magnesium: 1.7 mEq/L
c. Serum sodium: 122 mEq/L
d. Serum calcium: 6.9 mg/dL
D. Hypocalcemia destabilizes excitable membranes and can lead to muscle twitches, spasms, and tetany. This effect of hypocalcemia is enhanced in the presence of tissue hypoxia. The flexion contractions (Trousseau's sign) that occur during blood pressure measurement are indicative of hypocalcemia, not the other electrolyte imbalances, which include hypokalemia, hyponatremia, and hypomagnesemia.
3. A nurse is teaching a client with premature ectopic beats. Which education should the nurse include in this client's teaching? (Select all that apply.)
a. Smoking cessation
b. Stress reduction and management
c. Avoiding vagal stimulation
d. Adverse effects of medications
e. Foods high in potassium
ANS: A, B, D
A client who has premature beats or ectopic rhythms should be taught to stop smoking, manage stress, take medications as prescribed, and report adverse effects of medications. Clients with premature beats are not at risk for vasovagal attacks or potassium imbalances.
DIF: Remembering/Knowledge REF: 673 KEY: Patient education MSC: IntegratedProcess:NursingProcess:Implementation
NOT: Client Needs Category: Health Promotion and Maintenance
A nurse evaluates laboratory results for a male client who reports fluid secretion from his breasts. Which hormone value should the nurse assess first?
a. Posterior pituitary hormones
b. Adrenal medulla hormones
c. Anterior pituitary hormones
d. Parathyroid hormone
ANS: C
Breast fluid and milk production are induced by the presence of prolactin, secreted from the anterior pituitary gland. The other hormones would not cause fluid secretion from the client's breast.
17. A patient has been diagnosed with AKI, but the cause is uncertain. The nurse prepares patient educational material about which diagnostic test?
a. Flat plate of the abdomen
b. Renal ultrasonography
c. Computed tomography
d. Kidney biopsy
d
38. Which type of medication is used for patients receiving a platelet transfusion as premedications to prevent a reaction?
a. Vitamin K and a diuretic
b. Aspirin and hydroxyurea
c. Diphenhydramine and acetaminophen
d. Hydroxortisone and an antihypertensive
c
73. The client is diagnosed with a pulmonary
embolus (PE) and is receiving a heparin drip.
The bag hanging is 20,000 units/500 mL
of D5W infusing at 22 mL/hr. How many
units of heparin is the client receiving each
hour? ________
880 units.
If there are 20,000 units of heparin in 500
mL of D5W, there are 40 units in each mL:
20,000 ÷ 500 = 40 units
If 22 mL are infused per hour, then 880 units
of heparin are infused each hour:
40 × 22 = 880
TEST-TAKING HINT: The test taker must know
how to calculate heparin drips from two aspects:
the question may give the mL/hr and
the test taker has to determine units/hr, or
the question may give units/hr and the test
taker has to determine mL/hr. Remember to
learn how to use the drop-down calculator
on the computer. During the NCLEX-RN,
the test taker can request an erase slate.
17. For the patient with PKD, which antihypertensive medication may be used because it helps control the cell growth aspects of PKD and reduce microalbuminuria?
a. Angiotensin-converting enzyme inhibitors
b. Beta blockers
c. Calcium channel blockers
d. Vasodilators
a
1. A nurse working in a geriatric clinic sees clients with "cold" symptoms and rhinitis. Which drug would be appropriate to teach these clients to take for their symptoms?
a. Chlorpheniramine (Chlor-Trimeton)
b. Diphenhydramine (Benadryl)
c. Fexofenadine (Allegra)
d. Hydroxyzine (Vistaril)
ANS: C
First-generation antihistamines are not appropriate for use in the older population. These drugs include chlorpheniramine, diphenhydramine, and hydroxyzine. Fexofenadine is a second-generation antihistamine.
Question 22 of 41
The nurse is teaching a client with vitamin B12 deficiency anemia about dietary intake. Which type of food does the nurse encourage the client to eat?
a. Dairy products
b. Grains
c. Leafy vegetables
d. Starchy vegetables
a
Dairy products
Dairy products such as milk, cheese, and eggs will provide the vitamin B12 that the client needs. Grains, leafy vegetables, and starchy vegetables are not a source of vitamin B12.
The nurse is concerned that a client who had myocardial infarction (MI) has developed cardiogenic shock. Which findings indicate shock? (Select all that apply.)
a. Bradycardia
b. Cool, diaphoretic skin
c. Crackles in the lung fields
d. Respiratory rate of 12 breaths/min
e. Anxiety and restlessness
f. Temperature of 100.4° F
b. Cool, diaphoretic skin
c. Crackles in the lung fields
e. Anxiety and restlessness
The client with shock has cool, moist skin. Because of extensive tissue necrosis, the left ventricle cannot forward blood adequately, resulting in pulmonary congestion and crackles. Because of poor tissue perfusion, a change in mental status, anxiety, and restlessness are expected. All types of shock (except neurogenic) present with tachycardia, not bradycardia. Due to pulmonary congestion, a client with cardiogenic shock typically has tachypnea. Cardiogenic shock does not present with low-grade fever; this would be more likely to occur in pericarditis.
Question 9 of 16
A client with an endocrine disorder says, "I can't, you know, satisfy my wife anymore." What is the nurse's best response?
a. "Can you please tell me more?"
b. "Don't worry. That is normal."
c. "How does she feel?"
d. "Should I make an appointment with a counselor?"
a
"Can you please tell me more?"
Asking the client to explain his concerns in an open-ended question allows the nurse to explore his feelings more thoroughly. Telling a client that something is "normal" is dismissive; this is new to the client and is a concern for him. The focus of the nurse's response needs to be on the client, not on the wife initially. Referring the client to a counselor is not an appropriate first step; this dismisses the client's concerns and does not allow him to express his frustrations at the moment.
A client is being discharged with propylthiouracil (PTU). Which statement by the client indicates a need for further teaching by the nurse?
A. "I can return to my job at the nursing home."
B. "I must call if my urine is dark."
C. "I must faithfully take the drug every 8 hours."
D. "I need to report weight gain."
A.
The client should avoid large crowds and people who are ill because PTU reduces blood cell counts and the immune response, which increases the risk for infection. The client does not, however, need to remain completely at home. Dark urine may indicate liver toxicity or failure, and the client must notify the provider immediately. Taking PTU regularly at the same time each day provides better drug levels and ensures better drug action. The client must notify the provider of weight gain because this may indicate hypothyroidism; a lower drug dose may be required.
The nurse is concerned that a client who had myocardial infarction (MI) has developed cardiogenic shock. Which findings indicate shock? (Select all that apply.)
Bradycardia
Cool, diaphoretic skin
Crackles in the lung fields
Respiratory rate of 12 breaths/min
Anxiety and restlessness
Temperature of 100.4° F
Cool, diaphoretic skin
Crackles in the lung fields
Anxiety and restlessness
The client with shock has cool, moist skin. Because of extensive tissue necrosis, the left ventricle cannot forward blood adequately, resulting in pulmonary congestion and crackles. Because of poor tissue perfusion, a change in mental status, anxiety, and restlessness are expected. All types of shock (except neurogenic) present with tachycardia, not bradycardia. Due to pulmonary congestion, a client with cardiogenic shock typically has tachypnea. Cardiogenic shock does not present with low-grade fever; this would be more likely to occur in pericarditis.
39. An older patient has been receiving frequent blood transfusions without any complications or adverse reactions; however, the nurse carefully monitors the patient during the current transfusion. Which signs/symptoms suggest that the patient is experiencing circulatory overload?
a. Hypertension, bounding pulse, and distended neck veins
b. Fever, chills, and tachycardia
c. Urticaria, itching, and bronchospasm
d. Headache, chest pain, and hemoglobinuria
a
50. The nurse writes a problem of "impaired gas
exchange" for a client diagnosed with cancer
of the lung. Which interventions should be
included in the plan of care? Select all that
apply.
1. Apply O2 via nasal cannula.
2. Have the dietitian plan for six (6) small meals
per day.
3. Place the client in respiratory isolation.
4. Assess vital signs for fever.
5. Listen to lung sounds every shift.
1. Respiratory distress is a common finding
in clients diagnosed with lung cancer. As
the tumor grows and takes up more space
or blocks air movement, the client may
need to be taught positioning for lung
expansion. The administration of oxygen
will help